You are on page 1of 92

SELF-ASSESSMENT

Self-assessment
Questions f. Short-chain acyl-coenzyme A dehydrogenase deficiency
g. Hyperammonaemic hyperinsulinism
Case 1
h. Factitious induced illness
Sophie is a 36 hour old baby who presented to the Paediatric
During her admission Sophie’s blood and CSF cultures are
Assessment Unit yesterday with 9% weight loss and poor
reported as negative and her CRP returns to 15. A hypo-
feeding. Antenatal scans suggested Sophie was small for
glycaemia screen is completed: results are pending. Her
gestational age but she weighed 3.6 kg at birth and is the
glucose requirements continue to rise and IV access becomes
largest baby when compared to mother’s four previous chil-
increasingly difficult. Specialist opinion is sought.
dren. Pregnancy was otherwise uneventful and mother did not
(iii) What is Sophie’s glucose flow rate in mg/kg/hour based
receive a glucose tolerance test. She was born by normal
on an infusion of 15% dextrose at 150 ml/kg/day and a
vaginal delivery in hospital at full term and discharged home 6
current weight of 3.3 kg?
hours later; her mother noted that she struggled to take the
(iv) What specific drug treatment did the specialist centre
bottle for feeds prior to discharge home. There are no risk
recommend? (Choose ONE only)
factors for sepsis. Family history is unremarkable apart from
a. Mannose supplementation
Type 1 Diabetes Mellitus in the father and several of his family
b. Immediate transfer to specialist centre for treatment
members. On examination, Sophie has a soft fontanelle,
with sodium benzoate
normal cardiovascular, respiratory and abdominal examina-
c. Diazoxide in combination with a thiazide diuretic
tions. Both femoral pulses are palpable. She is not dysmor-
d. Diazoxide in combination with furosemide
phic. Blood gas is normal except the blood sugar, which is 2.0
e. No further treatment recommended, advised that So-
mmol/litre. A feed is immediately given by nasogastric tube as
phie’s condition would self-resolve over the next 5e7
Sophie will not take the bottle. The feed is an appropriate
days
volume based on 90 ml/kg/day every 3 hours. A blood sugar
is repeated 1 hour after this feed and is found to be 1.4. Case 2
(i) What is the next most appropriate course of action? A 3 month old baby girl, Sheila, is seen in the ED with her dad.
(Choose ONE only) He reports a 5 hour history of crying which has become
a. Perform a hypoglycaemia screen inconsolable, associated with coughing, choking, and “stop-
b. Give a 3 hourly volume at 90 ml/kg/day of formula via ping breathing” without colour change. The single episode of
nasogastric tube apnoea resolved spontaneously and there have been no
c. Give a 1 hourly volume at 90 ml/kg/day of formula via further respiratory symptoms. Sheila was born at 38 weeks
nasogastric tube gestation but elective caesarian. The birth weight was 3.4 kg
d. Give a 0.45% saline and 5% dextrose bolus at 2.5 ml/kg and she has been maintaining centile growth as documented
e. Start a 10% dextrose drip at 90 ml/kg/day in her Red Book. Sheila is the 3rd child to unmarried non-
f. Give a 10% dextrose bolus at 2.0 ml/kg medical healthcare professionals. Her older siblings are well.
g. Give a 10% dextrose bolus at 2.5 ml/kg She received her 2nd set of immunisations 4 days previously.
h. Wait 30 minutes and repeat blood sugar There was no history of cough or coryza. Dad reports that she
A full septic screen is performed and CRP is 50. Intravenous has been feeding “normally” and has passed three wet nappies
benzylpenicillin and gentamicin antibiotic therapy are initi- today, two of which contained bowel motions. She is formula
ated. Over the next 3 days Sophie’s blood sugar is difficult to fed and has no history of previous feeding issues.
keep above 2.6 mmol/litre: an infusion of 10% dextrose at 150 Her admission observations while crying were:
ml/kg/day is increased to 12.5% dextrose and finally 15%
dextrose. Despite this blood sugars remain at the lower end of
normal between 2.6 and 3.0 mmol/litre.
HR 140 beats/minute (120e140)
(ii) What is the most likely cause of Sophie’s persistent
RR 40 breaths/minute (30e50)
hypoglycaemia? (Choose ONE only)
Temperature 36.4  C (axillary)
a. Sepsis
BP 90/60 mmHg (80-50-115/80)
b. Transient Neonatal Hyperinsulinism
O2 saturations 99% in air (96e99)
c. BeckwitheWiedemann syndrome
Capillary refill time <2 seconds centrally and
d. Insulinoma
peripherally (<3)
e. Congenital disorder of glycosylation

Peter Heinz, State Exam Med, Dr med, MRCP, FRCPCH Addenbrooke’s When offered a bottle Sheila settles adequately for a new
Hospital, Department of Paediatrics, Cambridge, United Kingdom. senior house officer (SHO) to examine her. She has normal

PAEDIATRICS AND CHILD HEALTH 29:1 42 Ó 2018 Elsevier Ltd. All rights reserved.
SELF-ASSESSMENT

heart sounds, no respiratory distress, and a soft abdomen but you that Lucy is known to have mild asthma and has never
the history of inconsolable crying is convincing. been hospitalised with wheeze before. Lucy has a salbutamol
(i) What would the next appropriate investigation be? inhaler that she occasionally uses. She suffers from eczema
a. A chest X-ray to exclude lung pathology. and has multiple food allergies.
b. A catheter urine sample to exclude a UTI. On respiratory examination, Lucy has poor bi-basal air
c. A skeletal survey to exclude a fracture. entry and you can hear transmitted inspiratory upper airway
d. A septic screen prior to commencing IV antibiotics. noises with diffuse wheeze throughout her upper chest. The
e. Repeat the observations while the child is feeding. rest of her examination is unremarkable.
f. None required e this is explained by the recent (i) What is the next appropriate step in Lucy’s management?
immunisation. (Choose ONE only)
The crying abates after 7 hours and the baby is admitted to a. Continue inhaled salbutamol
the paediatric assessment unit for observation. Dad accom- b. Add ipratropium bromide inhalers alongside
panies. The examining SHO seeks senior advice regarding salbutamol
whether dad is able to consent to examination, investigations, c. IV salbutamol and magnesium sulphate
and treatment. d. IV aminophylline
(ii) What should she be told? e. Chlorphenamine 10 mg IM
a. Consent is not required again because the child has f. Hydrocortisone 200 mg IM
been examined once. g. Adrenaline 0.5 mg IM
b. Since the parents are unmarried, only mum can h. Call anaesthetics for emergency intubation and transfer
consent. to PICU
c. Regardless, she may act in the child’s best interests. i. Reassure and encourage breathing exercises
d. More information from dad is required. Lucy recovers well with appropriate treatment and is seen 4
e. The birth certificate should be sought. months later in the children’s outpatient combined clinic for
A detailed examination by a paediatric registrar identifies a Respiratory and Allergic conditions. Her asthma is well
‘thread tourniquet’ (Figure 1) on the second toe of the left foot. controlled and she is participating in lots of sports. Lucy has
The thread is tightly wound round the toe multiple times and had skin prick tests performed which show she has Type I
has circumferentially broken the skin. The toe tip is dusky but Hypersensitivity reactions to multiple pollens, milk, egg and
not necrotic and becomes immediately pink when the tourni- kiwi.
quet is removed by cutting. Dad is unable to provide any in- (ii) Which of the following leads directly to leukocyte adhe-
formation regarding how this has occurred and at this point sion in IgE hypersensitivity reactions? (Choose ONE only)
mum arrives in an extremely anxious state. a. Histamine
(iii) Does the discovery of this tourniquet warrant a safe- b. Platelet activating factor
guarding investigation? c. Leukotriene C4
a. Yes, and the patient should remain in hospital. d. Leukotriene D4
b. Yes, though the patient may be discharged into mum’s e. Leukotriene E4
care. f. Prostaglandin D2
c. No, not if the origin of the thread can be traced. g. Tumour necrosis factor
d. No, not unless mum requests one. During the clinic appointment, Lucy tells you that she is
e. No, only if there are further concerns. regularly using Doublebase ointment for the eczema on her
neck and face. The eczema in these locations has flared up
Case 3
over the past week and she has asked if you are able to pro-
You are asked to review a 14 year old girl called Lucy whose
vide her with any alternative treatments. On examination you
school have sent her in to the Emergency Department via
see that she has areas of erythematous dry skin across her face
ambulance with an acute asthma attack. The emergency
and neck with excoriation marks and some localised skin
department has called you because she has not responded to
thickening, particularly on the neck.
four salbutamol nebulisers back-to-back.
(iii) Which of the following is in line with NICE recommen-
On arrival she has a patent airway and looks warm and
dations for treatment of Lucy’s face and neck eczema?
well perfused. Her respiratory rate is elevated at 28 and there
(Choose ONE only)
is some mild respiratory distress. Heart rate is 120, saturations
a. 3e5 days Hydrocortisone acetate 0.1e2.5%
are 96% in air and capillary refill time is less than 2 seconds.
b. 7 days Hydrocortisone acetate 0.1e2.5%
Lucy can talk in full sentences but gasps to catch her breath at
c. 3e5 days Clobetasone butyrate 0.05%
the end of sentences. She has a biphasic noise that is audible
d. 7 days Clobetasone butyrate 0.05%
from the end of the bed, but you feel that the inspiratory phase
e. 3e5 days Beclometasone diproprionate 0.05%
is loudest. There is no rash. Lucy tells you that she acutely
f. 7 days Beclometasone diproprionate 0.05%
developed difficulty breathing at break-time; she did not eat
g. 3e5 days Clobetasol proprionate 0.05%
anything, but went outside with her friend for a walk. She is
h. 7 days Clobetasol proprionate 0.05%
otherwise well in herself. Her parents have arrived and inform

PAEDIATRICS AND CHILD HEALTH 29:1 43 Ó 2018 Elsevier Ltd. All rights reserved.
SELF-ASSESSMENT

central access due to the viscosity of high percentage dextrose


infusions. The oral medication most commonly used is Diaz-
oxide in combination with Chlorothiazide. Total fluid intake
must be no greater than 150 ml/kg/day on introduction of
Diazoxide due to its fluid retaining properties, hence the
administration with a thiazide diuretic. Diazoxide reduces
insulin levels by activating potassium channels which inhibit
insulin release, however it also triggers vasodilation via the
same mechanism and can cause low blood pressure. Moni-
toring during the administration of Diazoxide with Chlorothi-
azide includes daily electrolytes, weight and blood pressure
measurements. Cessation of these medications depends on the
cause of the hyperinsulinism; in transient cases the blood
sugars may become very elevated indicating resolution of the
hyperinsulinism, whereas genetic causes will require long
term treatment.
All cases of hyperinsulinism need to be discussed with
tertiary specialist centres and have follow-up with them.

FURTHER READING
1. Great Ormond Street Hospital http://www.gosh.nhs.uk/medical-
information-0/search-medical-conditions/hyperinsulinism.
2. National Organization for Rare Disorders http://rarediseases.org/
rare-diseases/congenital-hyperinsulinism/.
Figure 1 3. The Children’s Hospital of Philadelphia http://www.chop.edu/
centers-programs/congenital-hyperinsulinism-center#.
Vz388ZErK01.

Case 2
Answers b. A catheter urine sample for UTI
Case 1 d. More information from dad is required
(i) g. Give a 10% dextrose bolus at 2.5 ml/kg e. No, only if there are further concerns
with a 10% dextrose bolus at 2.0 ml/kg. Infection such as UTI may commonly explain inconsolable
(ii) b. Transient Neonatal Hyperinsulinism crying and the infant may be apyrexial at assessment. Without
(iii) a. 15.6 mg/kg/minute respiratory symptoms, a chest X-ray is unlikely to be infor-
(iv) c. Diazoxide in combination with a thiazide diuretic mative. Persistent infantile crying can often result in episodes
Newborn babies are treated for hypoglycaemia with a 10% of coughing and parents may report choking concerns
dextrose bolus at 2.5 ml/kg, whereas older babies and children accordingly. A detailed examination will identify the vast
are treated. majority of fractures and septic arthritides without the need to
Hyperinsulinism in the neonate is defined by continuous perform a skeletal survey. A convincing history of inconsol-
release of insulin from pancreatic beta cells without homeo- able crying should be investigated and would not be explained
static feedback from blood glucose levels; this causes hyper- by immunisations 4 days previously.
insulinaemic hypoglycaemia. In clinical practice, neonates Consent is required for non-emergency treatment from a
present with blood glucose levels that are resistant to high person with parental responsibility. If the parents have pre-
quantities of intravenous dextrose. A hypoglycaemic screen viously been married or if dad is named on the child’s birth
will include an insulin level: insulin levels during hypo- certificate then he holds parental responsibility. If this is not
glycaemia should typically be 20e30 mlU/L, therefore higher the case then consent from mum/another with parental re-
levels are suggestive of hyperinsulinism. Congenital hyperin- sponsibility should be sought. The doctor would be able to act
sulinism can be inherited by either autosomal dominant or without parental consent in the best interests of the patient in
recessive genes, but can also be transient and secondary to a an emergency situation.
variety of factors: diabetic mother, perinatal hypoxic injury, A thread or hair tourniquet is a rare but known presenta-
intrauterine growth restriction, BeckwitheWiedemann syn- tion of inconsolable crying and is believed to occur as a result
drome or can occur in isolation of these factors. of innocent movements of the infant. Commonly the source is
Once blood glucose levels have stabilised at greater than an adult hair or a blanket thread but the source need not be
3.5 mmol/litre using dextrose infusions at volumes no greater identified. Though non-accidental injury (NAI) should always
than 150 ml/kg/day, oral medications can be administered to be considered, this diagnosis in isolation may not be a safe-
control blood sugars. At this stage patients often require guarding concern. However, a detailed history and

PAEDIATRICS AND CHILD HEALTH 29:1 44 Ó 2018 Elsevier Ltd. All rights reserved.
SELF-ASSESSMENT

examination should always be undertaken to exclude other hypovolaemic state requiring intravenous fluids and in the
concerns. Recurrent presentations should also be considered. worst case scenario inotropic support. Salbutamol and oxygen
are key therapies but H1 antagonists and corticosteroids,
FURTHER READING although they can be useful should not be prioritised over
1. https://www.rcpch.ac.uk/resources/about-child-protection- other life-saving treatments. In-hospital doctors should ensure
companion. individuals are referred to a specialist paediatric allergy clinic
2. General Medical Council (2011) Protecting children and young following discharge and have an adrenaline auto-injector with
people: The responsibilities of all doctors. appropriate training on use for both the patient and care-
givers.

FURTHER READING
Case 3
1. Anaphylaxis and Allergy Foundation of America http://www.aafa.
(i) g. Adrenaline 0.5 mg IM
org/page/anaphylaxis-severe-allergic-reaction.aspx
(ii) g. Tumour necrosis factor
2. The European Academy of Allergy and Clinical Immunology
(iii) b. 7 days Hydrocortisone acetate 0.1e2.5%
guideline on the management of anaphylaxis in childhood http://
NICE guidelines recommend the use of mild potency topical
www.eaaci.org/attachments/881_Position%20Paper%
corticosteroids for the face and neck, except for short-term (3
20Anaphylaxis%20in%20childhood.pdf
e5 days) use of moderate potency topical corticosteroids for
severe flares. This is a moderate flare requiring mild potency
topical corticosteroids, therefore 7 days Hydrocortisone ace-
tate 0.1e2.5% is an appropriate choice.
This case describes a difficult presentation of anaphylaxis Practice points
masquerading as an acute asthma attack. Although anaphy-
laxis is most likely when there is acute onset of respiratory C Inconsolable crying has a wide differential that should always
distress (including wheeze and stridor) combined with skin be addressed with a thorough history and examination. UTI
and mucosal tissue involvement, however it can occur in and hair/thread tourniquets are among the differentials easily
isolation of these additional factors. Children who have missed. Full exposure is key.
asthma have a higher risk of allergic reactions and therefore C The difference between emergency care provided in a child’s
anaphylaxis. Advice to follow if you are unsure is to give best interests and the specifics of legal parental re-
adrenaline first followed by salbutamol inhalers; prompt in- sponsibility can prove challenging and the RCPCH offers clear
jection of intramuscular adrenaline is the ‘cornerstone of guidance on this.
therapy both in the hospital and in the community’ according C In any ED presentation NAI should be considered. Hair/thread
to the Anaphylaxis Working Group. Adrenaline is given as tourniquets provide a potentially misleading example.
0.01 ml/kg of 1:1000 (1 mg/ml) adrenaline via intramuscular Importantly, all staff members should feel able to raise
injection. In severe anaphylaxis vasodilation leads to a safeguarding concerns if they have them.

PAEDIATRICS AND CHILD HEALTH 29:1 45 Ó 2018 Elsevier Ltd. All rights reserved.
SELF-ASSESSMENT

Self-assessment
Questions b) Desmosomes
c) Hypodermis
Case 1
d) Connective tissue
A 1 year old boy was seen in general paediatrics outpatient
e) Basement membrane
clinic for a follow up appointment. He was born at term with
an unremarkable antenatal history and normal vaginal de- Case 2
livery. At birth he had marked skin loss of his left outer calf, A 31 year old lady attends asthma clinic with her 6 year old
and right elbow. He was admitted to the neonatal unit, he had daughter. She reports that she is 11 weeks pregnant. Two
no risk factors for sepsis, and remained a febrile. In light of the years ago she had a son that was born at 35 weeks gestation
skin loss, he was started on antibiotics. An urgent dermatology via normal vaginal delivery. She had premature rupture of
review was requested, the team who recommended caution membranes at 32 weeks, but otherwise was a normal preg-
with handling and skin care measures. A skin biopsy nancy. She reports he was extremely poorly when he was born
confirmed his diagnosis. He had no temperatures and was with lots of problems with his salt management and died in
discharged home day 5 of life following negative blood cul- the neonatal unit when he was only a few weeks old. She
tures. These lesions healed slowly with non-stick dressings. reports he was born with a hard layer of skin over the whole of
He continues to have skin fragility with easy blistering and a his body, and an abnormal appearance to his eyes and lips. He
delayed wound healing. There is a family history of paternal was diagnosed with Harlequin ichthyosis following genetic
eczema and hay fever. The patient continues follow-up with testing. There is no other family history of this condition. The
the dermatology team. The parents have attended clinic today mother and her husband are first cousins and were found to be
for general paediatric review. He appears happy with a good both carriers of the affected gene. She also has a 4 year old
head of straight brown hair. He is growing along the 25th daughter who is well apart from eczema. The mother does not
centile for weight and height, and reaching all his develop- feel this pregnancy is any different to her previous pregnan-
mental milestones. The parents are requesting the outcome of cies. She is concerned that this baby may also be affected and
the genetic bloods which were sent while he was the neonatal would like to discuss possible options available to her.
unit. The patient’s genetic results showed a homozygous base What tests would be available for discussion with this woman?
pair deletion in LAMB3 on chromosome 1. The mother is a) Chorionic villus sampling
heterozygous for this mutation; however the father is wild b) Fetal blood sampling
type for this mutation. Microsatellite markers were not c) Amniocentesis before 15 weeks gestation
consistent with isodisomy. d) Diagnosis from antenatal ultrasound scans
What is the most likely underlying diagnosis? e) None
a) Bullous pemphigoid What is the Pathophysiology of harlequin ichthyosis?
b) Epidermolysis bullosa a) Thin stratum corneum
c) Epidermolytic ichthyosis b) Breakdown of Dermaleepidermal junction
d) Friction blisters c) Increased hair follicles
e) Linear IgA dermatosis d) Thickening of the keratin layer
Which of the following is the most likely explanation for the e) Decreased water loss in transepidermal layer
genetic results? Following the birth of an affected baby with a collodion
a) Uniparental disomy membrane, what important single neonatal management
b) Dominance inheritance measures should be put in place?
c) Non paternity event a) Do not resuscitate order as it is a palliative condition
d) Sporadic mutation b) Careful fluid management
e) X-link recessive c) Surgical removal of collodion membrane
Which layer of the skin would a mutation in LAMB3 (laminin d) Early intubation as airway involvement is common
subunit beta 3) result in fragility? e) High salt feeds
a) Epidermis
Case 3
A 7 year-old boy attends A þ E with a history of intermittent
headaches and joint pains. He has had a worsening productive
Cassandra R. McDonald MBBCh is a registrar at Great Ormond cough and fever. He developed a non-itchy purple rash on his
Street Hospital, London, UK. face, chest and soles a few days ago which had a target-like
appearance. Areas of his upper left chest, lower back, and
Lee Hudson MBChB MTCPCH FRACP is a Consultant General upper arms bilaterally have blistered and pealed. His mum
Paediatrician and Hon Senior Lecturer at Great Ormond Street fears areas on his legs appear to be following the same pattern.
Hospital, London, UK. He reports sore eyes and pain on eating. There are visible

PAEDIATRICS AND CHILD HEALTH 29:2 95 Ó 2018 Published by Elsevier Ltd.


SELF-ASSESSMENT

blistering and ulcerations in his oral mucous membrane. He There is no X-link inheritance of epidermolysis bullosa
has a past medical history of epilepsy and was recently subtypes.
reviewed 14 days ago in the epilepsy outpatient clinic. He Part 3: E.
returned to the UK 3 weeks ago following a 7 day family Skin fragility in epidermolysis bullosa (EB) occurs in
holiday to Spain. His mother had a melanoma removed in her different areas within the skin depending on the subtype of EB.
early 20’s from her arm but reports she did not require any Epidermolysis bullosa simplex (EBS) is due to errors within
further treatment. There was no further family history of the epidermis or uppermost layer of skin cells (keratinocytes).
dermatological problems and the rest of the family have been Dystrophic epidermolysis bullosa (DEB) has errors within the
well since returning from holiday. On examination his total Lamina densa and upper dermis. Junctional epidermolysis
percentage skin area affected was estimated to be around bullosa (JEB) is secondary to mutations in Lamina 332 within
35%. Nikolsky’s sign was elicited on his leg. the basement membrane zone. Desomosomes provide adhe-
What is his likely underlying diagnosis? sion between cells, errors here result in skin fragility, but
a) Junctional epidermolysis bullosa patients also have woolly hair abnormalities. There are no
b) Toxic epidermal necrolysis connective tissue or hypodermis abnormalities which result in
c) Staphylococcal scalded skin syndrome skin fragility in EB.
d) Stevens Johnson syndrome
Answers case 2
e) Erythema multiforme
Part 1: A.
What is the likely cause of this condition?
Chorionic villus sampling (CVS) is a small sample of the
a) Antiepileptic medication
placenta, commonly performed between 11 and 13 weeks
b) Bacterial infection
gestation. If genetic mutations are known, CVS or amniocen-
c) Autoimmune
tesis can be used to obtain fetal genetic material and identify if
d) Genetic
the baby is a carrier. Amniocentesis is sampling of amniotic
e) Viral infection
fluid and performed after 15 weeks gestation. Ichthyosis
Which one of the following managements would be indicated
cannot be diagnosed from antenatal ultrasound scans. Fetal
as part of his treatment?
blood sampling is performed during labour in the presence of a
a) Intubation as airway involvement is life treating
pathological fetal heart rate to assess fetal compromise.
b) Eye lubrication
Part 2: D.
c) Surgical removal of affected skin
Ichthyosis is a disorder of epidermal maturation, with
d) Limited fluid replacement
excess keratin build up due to the desquamation defect
e) Prophylactic systemic antibiotic treatment
resulting in thick, scaly skin. The stratum corneum is a layer
within the epidermis, it is increased in ichthyosis. Breakdown
Answers of Dermaleepidermal junction would result in skin fragility,
not ichthyosis. Hair follicles may become clogged with skin
Answers case 1
scales resulting decreased hair. Ichthyosis results in an skin
Part 1: B.
barrier defect with increased water loss in transepidermal
Bullous pemphigoid and Linear IgA dermatosis are blis-
layer.
tering autoimmune conditions; you would not expect blisters
Part 3: B.
from birth. Babies with epidermolytic ichthyosis can be born
Ichthyosis results in a skin barrier defect with increased
with large areas of denuded skin and blistering, however the
water loss in transepidermal layer. Careful fluid management
skin fragility would improve with age and patients develop
is required as neonates are at risk of hypernatremia dehydra-
hyperkeratosis. Friction blisters would not explain the
tion. There is a high mortality rate associated with harlequin
continue skin fragility. A diagnosis of Epidermolysis bullosa
ichthyosis, however it is not a palliative condition and with
would explained the skin loss at birth secondary to foetal
parental consent, all attempts to stabilise the neonate should
movements in utero, these birth trauma skin loss often
be made. Surgical removal of collodion membrane is not
improve with non-adhesive dressings, however patients have
recommended. Airway involvement is not associated with
lifelong skin fragility.
collodion membranes and intubation not required routinely.
Part 2: C.
High calorie feeds are often required due to the increased
The absence of microsatellite markers consistent with iso-
energy expenditure of the skin defect.
disomy would make uniparental disomy unlikely. As the pa-
tients genetic results showed a homozygous base pair deletion Answers case 3
in LAMB3 on chromosome 1 this would point towards the Part 1: B.
inheritance pattern of autosomal recessive inheritance of Toxic epidermal necrolysis (TEN) syndrome and Stevens
junctional epidermolysis bullosa. As the mother is heterozy- Johnson syndrome (SJS) form a spectrum of disease of severe
gous for this mutation, it implies it is not dominant inheritance skin detachment with mucocutaneous complications. It is an
or a sporadic mutation. The father stated as wild type for this immune reaction to foreign antigens. Classification depends
mutation means he is not a carrier and the most likely on the percentage skin affected, Steven Johnson syndrome is
explanation is he is not the biological father to this child. <10% total body surface area, 10e30% is SJS/TEN overlap,

PAEDIATRICS AND CHILD HEALTH 29:2 96 Ó 2018 Published by Elsevier Ltd.


SELF-ASSESSMENT

>30% is Toxic epidermal necrolysis. Erythema multiforme is felt to increase a person’s predisposition to developing SJS/
associated with target lesions, however would not have the TEN syndrome when given medication. TEN can be caused by
level of skin detachment described in this case. Junctional bacterial or viral infections; however drug triggers are more
epidermolysis bullosa would result in skin fragility since birth, common.
not sudden onset in a 7 year old. Staphylococcal scalded skin Part 3: B.
syndrome commonly occurs in children under 5 years old and Intubation may be required if there is trachea and bronchi
starts with localised staphylococcal infection, Nikolsky’s sign involvement, however this is rare. Eye lubrication and daily
would be present but you would not expect a history of target ophthalmologist review is required as patient ophthalmology
lesions. complications include corneal ulceration and anterior uveitis.
Part 2: A. Surgical removal of affected skin is not recommended. Pa-
The patient has a history of epilepsy and recent outpatient tients need careful fluid management, with increased fluid
appointment points towards the possibility of a newly started requirements required. Prophylactic systemic antibiotic treat-
an antiepileptic medication. Drug reactions cause the large ment is not recommended, and antibiotics should be started in
majority of Toxic epidermal necrolysis, of which anti- suspected infections.
convulsants are commonly implicated. Genetic factors are

PAEDIATRICS AND CHILD HEALTH 29:2 97 Ó 2018 Published by Elsevier Ltd.


SELF-ASSESSMENT

Self-assessment
Questions g) PICU transfer
h) 2ml/kg 10% dextrose
Case 1
i) Arterial line
An 8 year old boy presents to the emergency department with
Question 1.4: Which of the following statements are not
a 2 day history of pyrexia over 39 degrees, diarrhoea, red lips
correct (choose 3):
and a diffuse macular rash. His mother attended because he
a) Adrenaline has a greater affinity for b1 than ⍺1 receptors
has not passed urine in the last 24 hours. He is usually fit and
b) Noradrenaline has a greater affinity for b1 than ⍺1
well, is fully immunised and has not recently travelled or had
receptors
any unwell contacts.
c) The effects of adrenaline are improved at acidic pH
His triage observations are:
d) Adrenaline causes a greater increase in heart rate than
Saturations: 97% in air, respiratory rate: 50/minute, no
noradrenaline
respiratory distress, heart rate: 180/minute, blood pressure:
e) Adrenaline causes bronchoconstriction
82/45 mmHg, distal capillary refill time: 5 seconds, central
capillary refill time: 3 seconds, temperature: 39 C, inappro-
priately drowsy. Case 2
He is about to have a peripheral cannula inserted. A 7 year old patient with known sickle cell disease (HbSS)
Question 1.1: What would your immediate management presents to his local emergency department with a 12 hour
plan be (Choose 3)? history of worsening shortness of breath. He had 2 days of
a) Chest X-ray fever and nausea 2 weeks previously but has been well in the
b) Urine dip interim. He has never been admitted previously and is not on a
c) Blood culture regular transfusion programme. His admission assessment is:
d) 10ml/kg 10% dextrose Respiratory rate: 60/minute, some accessory muscle use.
e) Broad spectrum antibiotics clear chest with good air entry throughout. Saturations 97% in
f) Paracetamol air, heart rate: 150/minute, blood pressure: 100/60 mmHg,
g) 12 lead ECG distal capillary refill time: 2 seconds, central capillary refill
h) 20ml/kg 0.9% sodium chloride time: 2 seconds, Temperature: 37 C. Drowsy but denies pain
Question 1.2: At this stage what is your diagnosis- (choose anywhere.
1)? His venous blood gas shows:
a) Systemic inflammatory response syndrome (SIRS) pH: 7.25, pCO2: 4.0kPa, pO2: 5.1kPa, Lactate: 22mmol/L,
b) Sepsis HCO3: 16mmol/L, base excess: -16mmol/L, Na: 137mmol/L,
c) Severe sepsis K: 4.1mmol/L, Glucose: 3.7mmol/L, Haemoglobin: 28g/L
d) Septic Shock Question 2.1: What would your immediate management
e) Toxic shock plan be (Choose 3)?
30 minutes later he has received a total of 60ml/kg fluid a) Chest X-ray
resuscitation. He has had an additional wide-bore peripheral b) Salbutamol inhaler
cannula inserted and a peripheral dopamine infusion c) Blood culture
commenced. He has new hepatomegaly and bilateral fine d) Group and save
crepitations on examination. His heart rate is 140/minute, e) Broad spectrum antibiotics
blood pressure: 78/40 mmHg, central capillary refill time: 3s, f) High flow oxygen
peripheral capillary refill time: 5s. Bedside blood sugar is g) 12 lead ECG
4.5mmol/L h) 20ml/kg 0.9% sodium chloride
Question 1.3: What would you like to achieve in the next i) 10ml/kg O-ve packed red cells
15 minutes (choose 3)? You receive a phone call from the haematology laboratory
a) Intubation and ventilation stating that his automated full blood counts are as follows:
b) CPAP initiation Haemoglobin: 31g/L, White cell count: 45 x109/L, Platelets:
c) Furosemide infusion 240 x109/L. Manual cell count is awaited.
d) Central access and adrenaline infusion Question 2.2: Which of the following diagnoses should be
e) Central access and noradrenaline infusion part of your differential- (Choose 2)?
f) 0.3mg/kg IV furosemide a) Low output heart failure
b) High output heart failure
c) Sickle cell crisis
d) Parvoviral infection
James Diviney MB BChir MA (hons.) Cantab. MRCPCH is a Paediatric e) Adenoviral infection
Registrar in the NICU, Whittington Hospital, London, UK. f) Coxsackie viral infection

PAEDIATRICS AND CHILD HEALTH 29:3 146 Ó 2018 Elsevier Ltd. All rights reserved.
SELF-ASSESSMENT

g) Pneumococcal pneumonia Question 3.4: What is the appropriate management esca-


After being transfused his haemoglobin is remeasured at lation ladder?
70 g/L a) Vagal maneuvers, Adenosine 1 mcg/kg, adenosine 2 mcg/
Question 2.3: At what Oxygen saturation would the patient kg, adenosine 3 mcg/kg, Cardiology advice Amiodarone
now become cyanosed? b) Vagal maneuvers, Adenosine 10 mcg/kg, adenosine 20 mcg/
a) 95% kg, adenosine 30 mcg/kg, Cardiology advice Amiodarone
b) 88% c) Vagal maneuvers, Adenosine 100 mcg/kg, adenosine 200
c) 82% mcg/kg, adenosine 300 mcg/kg, Cardiology advice
d) 71% Amiodarone
e) He is too anaemic to be cyanosed d) Vagal maneuvers, Adenosine 250 mcg/kg, adenosine 500
mcg/kg, adenosine 750 mcg/kg, Cardiology advice
Case 3
Amiodarone
An 8 year old is brought to the emergency department by her
e) Vagal maneuvers, Sedation if alert, Synchronised DC shock
parents having awoken in the night with a feeling of her heart
2J/kg, Synchronised DC shock 5J/kg
“going too fast”. She has a 3 day history of cough that is much
f) Vagal maneuvers, Sedation if alert, Synchronised DC shock
worse in the last 12 hours. She is usually fit and well and has
1J/kg, Synchronised DC shock 2J/kg
not attended hospital before.
Her triage observations are: Saturations: 99% in air, res-
piratory rate: 28/minute, no respiratory distress, heart rate: Answers
220/minute, blood pressure: 100/60 mmHg, distal capillary
Case 1
refill time: 2 seconds, central capillary refill time: 2 seconds,
- 1.1: C, E, H
temperature: 37.3 C, alert.
 Blood culture: A pre-antibiotic blood culture is key,
Q3.1: What is your initial management (choose 3)?
especially for gram positive organisms where broad-
a) 12 lead ECG
spectrum antibiotics may have a rapid bacteriocidal ef-
b) CXR
fect, preventing growth in future cultures
c) Blood gas
 Broad spectrum antibiotics: The patient is critically un-
d) Tibial I.O insertion
well with a likely infective aetiology. Reversing the un-
e) Pedal cannula
derlying cause of his decompensation as soon as possible
f) Ante-cubital fossa cannula
is vital. There are mortality benefits in early antibiotic
g) Attach to defibrillator
administration.
h) IV antibiotics
 20ml/kg 0.9% Sodium chloride: The patient is shocked
You are asked to perform an ECG.
and peripherally shut down. Aggressive fluid resuscita-
From the following options answer the 2 questions below.
tion is key to improve oxygen delivery to vital organs.
a) T wave inversion in V1eV3
There is no trauma history and no suggestion of volume
b) T wave inversion in V5eV6
overload so 20ml/kg (rather than smaller aliquots) is
c) RSR pattern in V1
appropriate. Crystalloid fluids are readily available for
d) RSR pattern in V6
rapid fluid resuscitation and may later be followed by 5%
e) Absent P waves
human albumin solution
f) Inverted P waves
g) Broad QRS complexes - 1.2: E
h) No ReR variation  The patient is likely in toxic shock as evidenced by
i) QRs duration >50ms meeting 4 of the below criteria
j) 4th intercostal space, left midclavicular line  Fever: temperature greater than or equal to 38.9 C
k) 4th intercostal space, anterior axillary line  Rash: diffuse macular erythroderma
l) 5th intercostal space, left midclavicular line  Hypotension: systolic blood pressure less than fifth
m) 5th intercostal space, anterior axillary line percentile by age for children aged less than 16 years
n) Paper speed of 10mm/s  Multisystem involvement (three or more of the
o) Paper speed of 25mm/s following organ systems):
Question 3.2:  Gastrointestinal: vomiting or diarrhoea at onset of
Which of the above are normal features on a paediatric illness
ECG (choose 4)?  Muscular: severe myalgia or creatine phosphokinase
Question 3.3: level at least twice the upper limit of normal
What is the appropriate placement for ECG lead V4  Mucous membrane: vaginal, oropharyngeal, or
(choose 1)? conjunctival hyperemia
The AþE consultant advises that the ECG shows supra-  Renal: Urea or creatinine at least twice the upper
ventricular tachycardia. limit of normal for laboratory or urinary sediment

PAEDIATRICS AND CHILD HEALTH 29:3 147 Ó 2018 Elsevier Ltd. All rights reserved.
SELF-ASSESSMENT

with pyuria (greater than or equal to 5 leukocytes  Hypotension is not necessary for the clinical
per high-power field) in the absence of urinary tract diagnosis of septic shock; however, its pres-
infection ence in a child with clinical suspicion of
 Hepatic: total bilirubin, alanine aminotransferase infection is confirmatory
enzyme, or aspartate aminotransferase enzyme  It is the rash that differentiates toxic shock from septic
levels at least twice the upper limit of normal for shock if no organism has been isolated.
laboratory  “Sepsis 3” criteria were introduced for adults in 2016 but
 Hematologic: platelets less than 100 x109/L have yet to be widely adopted in paediatric practice
 Central nervous system: disorientation or alterations
- 1.3: A,D,I
in consciousness without focal neurologic signs
 Intubation and ventilation: Following 60ml/kg of fluid he
when fever and hypotension are absent
is already showing signs of fluid overload and will
 Desquamation: 1e2 weeks after onset of rash
require ongoing fluid resuscitation. Ventilation will
 Probable cases need 4 of the above criteria to be met with
reduce demands on his cardiac output (40% of output
either
may be required to support the effort of breathing) and
 The absence of any growth on blood or CSF culture
increased intrathoracic pressure reduces left ventricular
Or
afterload, reducing cardiac work, especially in patients
 Culture of Staph. aureus or Streptococci.
with raised systemic vascular resistance
 Confirmed cases meet the same culture criteria and have
 Central access with adrenaline infusion: This child is in
all five of the clinical criteria described above, including
cold shock as evidenced by his prolonged distal CRT.
desquamation, unless the patient dies before desquama-
Adrenaline is thus the appropriate choice and is ideally
tion occurs.
delivered via central access though it can be given
 The other diagnoses are defined as:
peripherally. Intraosseous access can be sought if no
 SIRS- 2 of more of the following, where fever or white
expertise for central venous access is available.
cell count must be one
 Arterial line: Invasive blood pressure monitoring is
 Abnormal core temperature (<36  C or >38.5 C)
essential in a patient who remains hypotensive despite
 Abnormal heart rate (>2 standard deviations above
initiation of 2 inotropes.
normal for age, or less than 10th centile for age if
 He is not stable enough for transfer to PICU while his
child aged <1 years)
inotrope requirement is acutely increasing
 Raised Respiratory rate (>2 standard deviations
 Furosemide is not indicated until his blood pressure has
above normal for age, or mechanical ventilation for
stabilised, trying to reverse fluid overload at this point
acute lung disease)
will compromise his tissue perfusion
 Abnormal white cell count in circulating blood
(above or below normal range for age, or >10% - 1.4: B,C,E
immature white cells)  Noradrenaline has a higher affinity for ⍺1 receptors than
 Sepsis: SIRS in the presence of infection adrenaline. Adrenaline has a higher affinity for b1 and b2
 Severe sepsis: Sepsis in the presence of cardiovascular receptors. ⍺1 receptors mediate peripheral vasoconstric-
dysfunction, acute respiratory distress syndrome, or tion. b1 receptors primarily are chronotropic (increased
dysfunction of 2 organ systems. HR), inotropic (increased cardiac contractility) and dro-
 Septic shock: motropic (increased cardiac electrical conduction speed)
 Previously defined as severe sepsis with ongoing  b1 receptors are chronotropic- increasing heart rate.
hypotension despite adequate fluid resuscitation Adrenaline thus has a greater impact on heart rate
 The most recent accepted definition moves away  Adrenaline binds less readily at acidic pH. Thus a rising
from the previous approach and defines septic shock pCO2 or lactate may increase the patient’s adrenaline
as: requirement
 Suspected infection manifested by hypothermia or  Adrenaline causes bronchodilation via b2 receptor bind-
hyperthermia ing in bronchioles
 Clinical signs of inadequate tissue perfusion
including any of the following: FURTHER READING
 decreased or altered mental status Centre for disease control: Toxic Shock, Case definition, 2011.
 prolonged capillary refill greater than 2 seconds Davis et al. American College of Critical Care Medicine Clinical
 diminished pulses Practice Parameters for Hemodynamic Support of Pediatric
 Mottled cool extremities or flash capillary refill and Neonatal Septic Shock, Critical Care Medicine: June
 bounding peripheral pulses and wide pulse 2017; 45(6):1061e1093.
pressure Singer M, Deutschman CS, Seymour CW, et al. The Third Inter-
 decreased urine output less than 1mL/kg/hr. national Consensus Definitions for Sepsis and Septic Shock
(Sepsis-3). JAMA. 2016;315(8):801e810. https://doi.org/10.
1001/jama.2016.0287.

PAEDIATRICS AND CHILD HEALTH 29:3 148 Ó 2018 Elsevier Ltd. All rights reserved.
SELF-ASSESSMENT

Case 2 saturated with oxygen. 50/70¼ 71%. The higher the total
- 2.1: A, F, I haemoglobin the higher the saturation level at which
 10ml/kg O-ve packed red cells cyanosis is evident. This makes it easy to miss cyanosis
 He is profoundly anaemic but not shocked, any fluid in anaemic patients.
that is not packed red cells will haemodilute him
Case 3
further and worsen his oxygen delivery. At such a low
- 3.1: A,C,F
Hb even slight dilution may critically reduce cardiac
 A 12 lead ECG is key in determining the rhythm
oxygen supply
abnormality
 High flow O2
 Blood gas: Provides a rapid exclusion of some of the
 With his critically impaired oxygen carrying capacity
electrolyte abnormalities that may precipitate patholog-
any marginal increase in oxygen delivery will be
ical rhythms.
helpful. By increasing the partial pressure of alveolar
 Ante-cubital fossa cannula: The patient may need aden-
O2 you can improve O2 delivery even with saturations
osine. The rapid degradation of the medication means
in air of 97%
administration should be into a large vessel, as close to
 Chest X-ray
the heart as is practical, followed by rapid flush.
 Acute blood product resuscitation is the main priority
 The defibrillator should be nearby but attaching the pads
for this patient. This may be rate-limited if he has
will prevent accurate ECG lead attachment- especially V1.
existing or developing pulmonary oedema. The chest
She is not shocked so there is enough time to obtain a 12
film will help narrow the differential as to the cause of
lead ECG which may be invaluable in identifying the
the decompensation.
cause of the SVT post cardioversion.
- 2.2: B, D  A chest X-ray is useful but will delay the initial man-
 High output cardiac failure: Cardiac failure is an inability agement of this emergency
of the heart to meet the body’s oxygen demand. Where  IV antibiotics may be indicated but will take up infusion
the carrying capacity of the blood is highly impaired this time on the cannula that may be required for other
can occur even with a healthy heart. His good peripheral medications.
perfusion makes a low output state unlikely. The lactate
- 3.2: A,C,I,O
is from anaerobic metabolism of tissue that is well
 T wave inversion in V1eV3: This “juvenile T-wave
perfused, but with too little oxygen content.
pattern” can persist to adolescence
 Parvovirus:Infection with Parvovirus B19 causes acute
 RSR pattern in V1
bone marrow suppression. In haemoglobinopathy pa-
 Paper speed of 25mm/s
tients the turnover of red cells is increased such that their
 QRS duration >50ms: QRS duration varies with age but is
circulating red cell numbers fall off before the bone
up to 70ms in young children and up to 100ms in
marrow recovers. They may present weeks after the
adolescents
Parvoviral infection as their circulating haemoglobin
 Inverted or absent p waves with no ReR variation and a
falls. In the recovery phase nucleated red blood cells may
rate >200 is very likely to be SVT
be miscounted by the analyser as white cells giving an
inappropriately raised WCC. - 3.3: L
 Sickle cell crisis is unlikely without bony pain or chest  5th intercostal space, left midclavicular line: Key to get
signs. this correct as V5 and V6 are placed in the same hori-
 Pneumococcal pneumonia is more likely in HbSS due to zontal plane
the enhanced susceptibility to encapsulated organisms
- 3.4: C
and can cause a haemolytic uraemic picture with
 Vagal maneuvers, Adenosine 100 mcg/kg, adenosine 200
anaemia secondary to haemolysis. It’s unlikely with a
mcg/kg, adenosine 300 mcg/kg, Cardiology advice
clear chest examination
Amiodarone
- 2.3: D  This is the APLS algorithm given his BP is well
 71%. Cyanosis is visible at levels of deoxyhaemoglobin maintained.
from 20 to 50g/L. With an overall Hb of 70g/L he could  F is the correct algorithm if the patient was showing
appear cyanosed when 50g/L of the haemoglobin is signs of shock.

PAEDIATRICS AND CHILD HEALTH 29:3 149 Ó 2018 Elsevier Ltd. All rights reserved.
SELF-ASSESSMENT

Self-assessment
Questions d) Inborn error of metabolism, duct-dependent cardiac lesion
e) Non-accidental injury, duct-dependent cardiac lesion
Case 1
f) Sepsis, non-accidental injury
A three week old female infant presents with a 12 hour history
of lethargy and poor feeding following a period of being Question 3
inconsolable. She was born at term with no known perinatal You gain intravenous (IV) access. What would be your initial
risk factors for infection. Antenatal scans were normal. On management in the emergency department? Please select
examination she was extremely pale with a central capillary FOUR from the list.
refill time of 4e5 seconds. She had a non-bulging fontanelle a) Blood culture
with reduced tone and reduced responsiveness (V on AVPU b) Bloods including serum ammonia level
scale). There were no signs of increased work of breathing, c) Cranial ultrasound scan
heart sounds were normal and her abdomen appeared non- d) Abdominal ultrasound scan
distended. e) Chest X-ray
Observations: f) IV antibiotics as per unit policy
Pulse: 186/minute g) Commence maintenance IV fluids
Respiratory rate: 50/minute h) Obtain urine sample for culture
Saturations 98% i) Lumbar puncture for CSF culture
Temperature: 37.8  C j) IV fluid bolus 20ml/kg

Question 1 Question 4
Which missing information would you like first as part of your 12 hours later a phone call was received from the microbi-
urgent assessment to help with your differential diagnosis? ology department reporting positive blood cultures. What is
Please select THREE from the list. the most likely pathogen in this case? Please select ONE from
a) Peripheral temperature the list.
b) Presence/absence of femoral pulses a) Coagulase negative Staphylococcus
c) Presence/absence of bowel sounds b) Group B Streptococcus
d) Primitive reflexes c) E. coli
e) Blood pressure d) S. aureus
f) Presence/absence of mottled discolouration of the skin e) Neisseria meningitidis
g) Occipital-frontal circumference f) Listeria monocytogenes
h) Skin turgor
Venous gas results: Case 2
pH 7.0
A 13 year old girl with a complex medical background
pCO2 8.1 kPa
including neurodisability and exclusive percutaneous endo-
HCO3: 14.7 mmol/L
scopic gastrostomy (PEG) tube feeding, presents as a stand-by
Lactate: 10 mmol/L
to the emergency department with a blue-episode at home.
Bedside blood glucose level: 5.4 mmol/L
She was saturating 93% on 15L face mask oxygen, had
BE: -11.6
received a 20ml/kg fluid bolus in the emergency department
Bowel sounds were active and femoral pulses easily
and was commenced on co-amoxiclav for a presumed chest
palpable.
infection.
Question 2 You attend to review and note that she was initially hy-
In view of the information above, what are your top 2 differ- potensive on arrival to the emergency department which had
ential diagnoses? Please select ONE from the list. improved following the fluid bolus. On your arrival you agree
a) Duct-dependent cardiac lesion, intussusception that she has focal right-sided chest signs consistent with a
b) Meningitis, intussusception lower respiratory tract infection. She has a central capillary
c) Sepsis, inborn error of metabolism refill time of 3e4 seconds, a pulse of 136/min and a temper-
ature of 38.1  C. She is responding to voice but appears sleepy,
which her mum reports is new today.
Katherine Jarman MBChB Royal Hospital for Sick Children, Question 1
Edinburgh, UK. Conflict of interest: none declared. What are the next urgent steps in her investigation based on
your clinical findings? Please select TWO from the list.
Natalie Bee MBChB MRCPCH Royal Hospital for Sick Children, a) Capillary gas
Edinburgh, UK. Conflict of interest: none declared. b) Re-check blood pressure

PAEDIATRICS AND CHILD HEALTH 29:4 193 Ó 2019 Elsevier Ltd. All rights reserved.
SELF-ASSESSMENT

c) Blood sugar Haemoglobin 87 mg/dL


d) Chest X-ray WCC 15 x 109/L
e) Urine culture Neutrophils 10 x 109/L
Platelets 42 x 109/L
Question 2
The blood film confirms low platelets and shows
Her repeat blood pressure was 80/49 mmHg. What is
schistocytes.
your next step in her management? Please select ONE from the
What are the most urgent steps in his management now?
list.
Select THREE from the list.
a) 20mL/kg 0.9% saline fluid bolus
a) Catheterise
b) 10ml/kg 0.9% saline fluid bolus
b) Move to HDU
c) 3mL/kg 10% dextrose bolus
c) 12 lead ECG
d) 5mL/kg 0.9% saline fluid bolus
d) Discuss with renal unit/PICU
e) Blood transfusion
e) Send Group and save
f) Request senior review for consideration of inotropes
f) Phone public health
g) Start maintenance IV fluids
g) Give furosemide 1mg/kg
Question 3 h) Give Calcium Gluconate 10% 0.5ml/kg (equivalent to
You are concerned this patient has sepsis. What additional 0.11mmol/kg calcium)
Systemic Inflammatory Response Syndrome (SIRS) criteria
Question 3
would confirm this? Please select ONE from the list.
What is the most likely pathogen to have caused the diar-
a) Urine output less than <0.5mL/kg/hr
rhoea? Please select ONE from the list.
b) Elevated CRP
a) E. coli
c) Elevated lactate
b) Norovirus
d) Elevated or depressed white cell count
c) Campylobacter
e) Elevated or depressed platelets
d) Giardia lamblia
f) Elevated or depressed neutrophil count
e) Salmonella
f) Adenovirus
Case 3
A 4 year old boy presents with a four day history of diar-
rhoea and vomiting. His parents tell you he has been passing
Answers
15e20 loose stools per day, which are watery in consistency. Case 1
No other family members are affected. There is no history of 1 B, C and E
recent travel. His abdomen is soft and mildly tender on deep 2 C
palpation, with bowel sounds present. He is clinically dehy- 3 A, B, F and J
drated and is admitted for intravenous fluids. Admission 4 B
bloods are unremarkable. The sick neonate should lead you to the differential diagnosis
After 24 hours in hospital his diarrhoea becomes bloody. of sepsis, inborn error of metabolism and a duct dependent
You note that his urine output has reduced in the last six cardiac lesion. Intra-abdominal pathology such as midgut
hours. volvulus associated with malrotation or intussusception
Question 1 should also be considered as well as non-accidental injury.
Which investigations would you do urgently? Please select The presence or absence of femoral pulses, presence or
TWO from the list. absence of bowel sounds and the blood pressure would help to
a) Stool bacterial culture refine the differential diagnosis. Given the high lactate and
b) Urinalysis clinical presentation, sepsis and inborn error of metabolism
c) Serum urea, electrolytes and creatinine are the top two differential diagnoses in this case. If an inborn
d) Full blood count and film error of metabolism is suspected, it is important to ask for an
e) Coagulation screen urgent serum ammonia level. Patients should be treated for
f) Abdominal ultrasound scan sepsis until proven otherwise with broad-spectrum antibiotics
g) Stool culture for viruses as per your unit policy. Whilst it would be important to obtain
h) CRP urine and CSF samples for culture, the stabilisation of your
patient and early antibiotic administration is the priority in
Question 2 this patient. The most common cause of neonatal sepsis is
His bloods return with the following results: Group B Streptococcus.
Urea 14 mmol/L
Creatinine 279 mmol/L FURTHER READING
Potassium 7 mmol/L E MacNeill, C Walker. Inborn Errors of Metabolism in the Emer-
Sodium 140 mmol/L gency Department (Undiagnosed and Management of the

PAEDIATRICS AND CHILD HEALTH 29:4 194 Ó 2019 Elsevier Ltd. All rights reserved.
SELF-ASSESSMENT

Known) Emergency Medicine Clinics of NorthAmerica 2018, consensus conference: definitions for sepsis and organ
Vol 36, Issue 2, Pages 369-385 dysfunction in paediatrics, Pediatric Critical Care Medicine.
National Institute for Health and Care Excellence, Sepsis: 2005 Jan; 6(1): 2-8
recognition, diagnosis and early management. Last updated
Case 3
September 2017
1. C and D
A Plunket, J Tong. Sepsis in Children, BMJ 2015: 350:h3017
2. C, D and H
Case 2 3. A
1 B and C This is a case of a patient with haemolytic uraemic syndrome
2 A (HUS) which is characterised by the onset of haemolytic
3 D anaemia, thrombocytopenia and acute renal failure. If HUS
This is a case of a complex patient presenting with sepsis, is suspected from the history, patients should have bloods
probably secondary to aspiration pneumonia. You should taken for renal function, full blood count and blood film. The
consider sepsis in any child with suspected or proven infection blood film will identify red cell fragments (schistocytes) and
with at least two of the following (Of which one must be confirm a low platelet count. A coagulation screen and stool
abnormal temperature or leucocyte count): culture (on sorbitol MacKonkey agar) would also be
- High or low core temperature (<36  C or >38.5  C) important first line investigations. The management of HUS
- Inappropriate tachycardia (refer to age appropriate chart) is supportive. Around 50% of patients with HUS will develop
- Elevated respiratory rate renal failure requiring dialysis in the acute phase and this
- Leucocyte count elevated or depressed for age case exemplifies why accurate fluid balance is important.
In patients with neurodisability their observations may not The patient in this case also developed hyperkalaemia which
score highly on early warning score charts, and special care may lead to arrhythmias. Calcium acts to stabilise the
should be taken with these patients. myocardium whilst salbutamol will drive potassium into
Antibiotics should be delivered as soon as possible in line cells. The majority of typical HUS (HUS associated with
with local guidelines and oxygen given to maintain normal diarrhoea) are caused by verotoxin producing strains of E.
saturations. Wherever possible blood cultures should be taken Coli. Sources of infection can be from contaminated food,
prior to antimicrobial therapy and bloods should be taken for contact with infected animals (e.g. petting farms) or contact
markers which identify organ dysfunction such as creatinine, with another infected person. You should ask about poten-
LFTs, lactate, gas, electrolytes, full blood count and coagula- tial sources in the history and public health should be
tion studies as well as CRP. In this case our patient initially informed.
responded to a fluid bolus but as signs of shock persisted, a
further bolus of 20mL/kg was given. Inotropic support may be FURTHER READING
required beyond this and senior clinicians should be involved M Noris, G Remuzzi. Hemolytic Uremic Syndrome, Journal of the
early in a case like this. American Society of Nephrology, April 2005 Vol 6, Issue 4,
Pages 1035-1050
FURTHER READING Karpman, Hemolytic uraemic syndrome [Review], 2017, Journal
A Plunket, J Tong. Sepsis in Children, BMJ 2015: 350:h3017 of Internal Medicine Vol. 281, Issue 2, Pages 123-148
T Kawasaki, Update on Pediatric Sepsis, 2017, Journal of Masilamani, K., Van Der Voort, J., The Management of Acute
Intensive Care, 5:47 Hyperkalaemia in Neonates and Children, Archives of Disease
B Goldstein, B Giroir, A Randolph. International Consensus in Childhood, 2012, Vol 97, p376-380
Conference on Pediatric Sepsis. International pediatric sepsis

PAEDIATRICS AND CHILD HEALTH 29:4 195 Ó 2019 Elsevier Ltd. All rights reserved.
SELF-ASSESSMENT

Self-assessment

Case 1 In view of the marked asymmetry of the proptosis, a CT


head and orbits was performed (Figure 1).
A 3 year-old girl presents to the emergency department with 3
The report confirmed the presence of an irregular soft tis-
day history of left-sided eye swelling. She is otherwise sys-
sue mass within the left intraconal space with evidence of
temically well with no history of headache or vomiting. On
proptosis. Following discussion with the Paediatric Ophthal-
examination, she is afebrile with heart rate of 96/minute and
mology Consultant, an MRI of the head and orbits was per-
her blood pressure measures 102/56 mmHg. The patient has
formed which confirmed the diagnosis to be a localised ocular
obvious proptosis of left eye with mild erythema and sore to
lymphatic malformation.
touch. Her pupils were equal and reactive. There is no evi-
dence of a localised nerve palsy. Question 3
Compliance with visual acuity assessment was limited in What would be the recommended treatment for this lymphatic
the Emergency Department. malformation?
She is otherwise well and not prescribed any regular a) Chemotherapy
medication. b) Complete surgical excision
c) Watch and wait
Question 1 d) Sclerotherapy
What is the most common cause of proptosis in children? e) Systemic steroids
a) Capillary haemangioma
b) Orbital cellulitis Case 2
c) Graves’ disease
d) Rhabdomyosarcoma A 7-year-old boy presents to the paediatric emergency
e) Dermoid cyst department with a painful right eye, headaches, fevers and a
runny nose for the past four days. He is otherwise presently
Question 2 well, having recently recovered from an upper respiratory
What other investigations would be helpful at this stage? tract infection; he has no history of trauma. On observation he
(Please select 3) is mildly tachycardic, temperature of 38.5 degrees and has
a) ACE reduced urinary output.
b) Biopsy On examination he has periorbital oedema with erythema
c) CRP and protrusion of the right eye. He has difficulty opening the
d) CT right eye and pain on eye movement. He is not compliant with
e) CXR visual acuity testing.
f) dsDNA, ANCA, ESR
Question 1
g) FBC and blood film
What is the most likely diagnosis?
h) TFT
a) Venous sinus thrombosis
i) VMA
b) Periorbital cellulitis
c) Orbital cellulitis
d) Sinusitis
Roisin Begley MBBS BSc PG Cert Medical Simulation MRCPCH ST8 e) Retrobulbar haemorrhage
Paediatric Emergency Medicine, Paediatric Emergency Depart-
ment, University Hospital of Wales, Cardiff, UK. Conflicts of Question 2
interest: none declared. What are the possible complications of orbital cellulitis?
a) Optic neuropathy
b) Meningitis
Caitlin Peers Medical student, School of Medicine, Cardiff Uni-
c) Cavernous sinus thrombosis
versity, Cardiff, UK. Conflicts of interest: none declared.
d) Keratopathy
e) All of the above
Rebekah Caseley MBBCh MRCEM ST5 Emergency Medicine,
Paediatric Emergency Department, University Hospital of Wales, Question 3
Cardiff, UK. Conflicts of interest: none declared. CT scan shows intraorbital abscess, inflammatory stranding in
the intraconal fat, oedema of the extraocular muscles and
Jeff Morgan MBBS BSc MRCPCH Paediatric Consultant, Paediatric evidence of ethmoid, maxillary and cavernous sinus with a
Emergency Department, University Hospital of Wales, Cardiff, UK. cavernous sinus thrombosis. What Chandler Class of orbital
Conflicts of interest: none declared. complications does this patient have?

PAEDIATRICS AND CHILD HEALTH 29:5 243 Ó 2019 Elsevier Ltd. All rights reserved.
SELF-ASSESSMENT

c) Persistent diarrhoea
d) Weight loss
e) Palpitations and restlessness
On performing Thyroid function tests the results come back
as follows:
 TSH: 0.1 uIU/ml (0.3e4.45 uIU/ml),
 T4: 25.01 pmol/L (9.0e19.0 pmol/L),
 T3: 8.62 pmol/L (2.6e5.7 pmol/L)
Question 2
Given the severity of the proptosis a CT scan of the orbits is
performed. What is the most likely appearance on CT in
Graves’ disease?
a) Muscle thickening and enhanced surrounding fatty strands
b) Diffuse fusiform enlargement of the extraocular muscle
belly and tendon
c) Pressure erosion of lateral orbital rim from enlarged
muscles
Figure 1 CT of head and orbits d) Chronic ethmoid and maxillary sinusitis
a) I e) Well circumcised lesion with cystic appearance
b) II Question 3
c) III What is the preferred treatment in this case?
d) IV a) Carbamazepine
e) V b) Carbimazole
Question 4 c) Surgery
Which of the following structures are at risk of damage d) Propranolol
following infection in the cavernous sinus? e) Radioactive Iodine
a) Internal carotid artery
b) External carotid artery Case 4
c) Facial nerve A 12-year-old boy attends the Emergency Department after
d) Mandibular branch of trigeminal nerve being hit in the face by a cricket ball. He complains of pain to
e) Sphenopalatine ganglion the left side of his face and double vision. On examination he
Question 5 has bruising to the left infra-orbital margin and over the
What is the preferred treatment for orbital cellulitis in this case? zygoma. He has pain on opening the eye and his visual acuity
a) IV broad spectrum antibiotic, IV anaerobe cover and Clexane is 6/9 on the right (6/6 on the left). His visual fields are intact
b) Oral penicillin based antibiotic and oral anaerobe cover however he has diplopia on upward gaze and impaired right
c) IV broad spectrum antibiotic and Clexane sided superior gaze was noted.
d) IV broad spectrum antibiotic, oral anaerobe cover and The child has no other medical or ophthalmological history
Clexane of note.
e) IV broad spectrum antibiotic and topical anaerobe cover Question 1
Given the history, what is the likely injury?
Case 3 a) Blowout orbital floor fracture
A 15-year-old female presents with progressive bilateral b) Orbital roof fracture
proptosis over the last 3 months, with reported decline in c) Zygoma fracture
weight gain and noticeable changes in behaviour. No pain in d) Trap-door orbital floor fracture
eye reported or observed, no history of trauma to the eye. Question 2
Examination: visual acuity normal, marked proptosis, no What is the gold standard imaging modality of choice for this
lid lag or retraction, pupils regular, lens clear and iris normal patient?
colour and pattern. Intra ocular pressure (IOP) right 16 mm a) MRI facial bones
Hg, left 18 mm Hg. Fundoscopy: optic nerve normal. b) CT facial bones
Question 1 c) Ultrasound orbits
What is the most common presentation of Graves’ disease in d) Facial bones X-ray
the paediatric population? Question 3
a) Diffuse goitre If the child started vomiting, taking in to account the mecha-
b) Proptosis nism of injury, what should you consider?

PAEDIATRICS AND CHILD HEALTH 29:5 244 Ó 2019 Elsevier Ltd. All rights reserved.
SELF-ASSESSMENT

a) Intracranial bleed Case 2


b) Retrobulbar haemorrhage 1) c
c) Stimulation of the oculocardiac reflex 2) e
d) Concussion 3) e
4) a
5) d
Answers
Presenting features of painful red eye and loss of visual acuity
Case 1 with signs of systemic infection are highly suggestive of an
1) b orbital cellulitis. A diagnosis of pre-orbital cellulitis would give
2) c,d,g tenderness, redness and possibly fever however there would
3) d be no proptosis, reduction in visual acuity or reduction in
While Grave’s disease and other thyroid conditions are the ocular eye movements present. Venous sinus thrombosis and
most common cause of proptosis in adult. This is not the case retrobulbar haemorrhage both have different presenting fea-
for children. Orbital cellulitis and malignancy are the most tures and sinusitis may be a factor leading to infection of the
common causes. Malignancies that can result in exophthalmus orbit however would not be the most accurate diagnosis in this
are retinoblastoma, optic glioma, rhabdomyosarcoma, histio- case. Almost 90% of patients with orbital cellulitis had exist-
cytosis, Ewing’s sarcoma and metastases. Other causes of ing sinusitis, many have multiple sinuses involved and this
exophthalmus are inflammation (e.g. orbital pseudo tumour), may be diagnosed on imaging.
vascular malformations (e.g. capillary haemangioma) and Orbital cellulitis may cause a range of complications
developmental anomalies (e.g. dermoid cyst). including optic neuropathy, meningitis, cavernous sinus
The most appropriate initial investigations would be FBC thrombosis and keratopathy.
with blood film, CRP and CT. These are looking for features
suggestive of infection or malignancy. Further laboratory tests
could include TFT, dsDNA, ANCA, ESR and vanillylmandelic
Chandler Stage Clinical description and
acid (VMA). These are looking for signs of thyroid disease,
classification definition
inflammatory disease and more specific tests for malignancy
(VMA is a catecholamine metabolite where elevated levels can
I Inflammatory C Eyelid oedema and erythema
be associated with neuroblastoma).
oedema C Normal extra ocular move-
This was a rare case of an ocular lymphatic malformation,
ment and visual acuity
a benign tumour representing 1e3% of orbital masses. The
II Orbital C Diffuse oedema of orbital
pathogenesis of this vascular-type malformation is described
cellulitis contents without discrete
as abnormal growth of endothelial type channels. Clinically,
abscess formation
ocular lymphatic malformations tend to manifest with eyelid
III Sub C Collection of purulent
swelling or proptosis, occasionally acutely over 2e3 days.
periosteal exudate beneath periosteum
Other recognised signs reported include ptosis, decreased
abscess of lamina papyracea
vision and or motility. Orbital imaging is essential, whilst CT C Displacement of globe
may be requested in the first instance to exclude other orbital
downwards and laterally
pathologies, MRI is the preferred modality to determine the
IV Orbital C Purulent collection within
extent of the malformation. Imaging usually identifies a multi-
abscess the orbit
cystic orbital lesion, with or without evidence of fluid levels C Proptosis
suggestive of intralesional bleeding. C Chemosis
Traditionally, a conservative approach is followed unless C Ophthalmoplegia
need for surgical management deemed necessary. Reports C Decreased vision
describe spontaneous resolution via this method in selected
V Cavernous C Bilateral eye finding
patients. However, due to the variable nature in the clinical
sinus C Prostration or weakness
course of ocular lymphatic malformations, some practitioners
thrombosis C Meningismus (triad of nuchal
opt for non-surgical interventions such as sclerotherapy, cor-
rigidity, photophobia and
ticosteroids or laser therapy. For those patients with more
headache)
severe malformations associated with cosmetic deformity or a
high risk of visual compromise, then surgical excision is the
recommended option. This can often be difficult due to the
location and cystic structure of the lesion with recognised risks Infection of the Cavernous sinus may cause infection of
of haemorrhage or damage to adjacent structures. many adjacent structures. Within the orbit the orbital veins
draining in both antegrade and retrograde directions may
FURTHER READING: allow infections to spread from superficial areas into the
Orbital Lymphatic Malformation (Lymphangioma) C. Blake et al. cavernous sinus. Passing through the sinus, the oculomotor,
Webeye.ophth.uiowa.edu trochlear and trigeminal nerve branches (ophthalmic and

PAEDIATRICS AND CHILD HEALTH 29:5 245 Ó 2019 Elsevier Ltd. All rights reserved.
SELF-ASSESSMENT

maxillary) as well as the internal carotid artery and the sym- FURTHER READING
pathetic nerves of the orbit may be affected. If severe enough Chaudhry IA, Al-Rashed W, Arat YO. The Hot Orbit: Orbital
infection may spread collaterally to the pituitary and possibly Cellulitis. Middle East African Journal of Ophthalmology.
the meninges. 2012;19(1):34e42. https://doi.org/10.4103/0974-9233.92114.
Treatment is required immediately in all cases of orbital https://www.ncbi.nlm.nih.gov/pmc/articles/PMC3277022/
cellulitis to prevent the possible spread of the infection to all https://cks.nice.org.uk/hyperthyroidism#!scenario
structures mentioned above. The optimal treatment covers Case 4
Staphylococcus and Streptococcus infections using and broad 1) d
spectrum IV antibiotic plus anaerobic cover and clexane to 2) b
treat the cavernous sinus thrombosis in this case. 3) c
FURTHER READING
Orbital floor fractures are usually the result of blunt trauma to
Chaudhry IA, Al-Rashed W, Arat YO. The Hot Orbit: Orbital the face and are normally occur due to low-velocity and high-
Cellulitis. Middle East African Journal of Ophthalmology. force crush injuries. Orbital floor fractures in young children
2012;19(1):34e42. https://doi.org/10.4103/0974-9233.92114. (i.e. under 7 years old) are less common due to their thicker
sinus walls, higher proportion of cheek fat, greater bone
Case 3 elasticity and smaller mid-face. Under the age of seven, orbital
1) a roof fractures are more frequently seen. Orbital floor fractures
2) a in children present differently than in adults. They will
3) b frequently demonstrate diplopia and limitations in ocular
Graves’ disease is much less common in the paediatric pop- movements however, unlike adult patients, tend to have
ulation than in the adult, however it is important to diagnose minimal or no peri-orbital swelling. They may also present
due to its impact on growth and development of the child. By with pain, infraorbital anaesthesia, enophthalmos and sub-
far the most common cause of hyperthyroidism in children is conjunctival haemorrhage.
Graves’ disease, with the peak in diagnosis occurring during The most common orbital floor fracture in children is the
adolescence and with a female dominance. The classical “trap door” type, where the elastic paediatric bone breaks in a
insidious onset of the disease may lead to months or years linear manner along the infraorbital nerve canal, with a hinge
before diagnosis is made. Diffuse goitre is common in chil- at the medial edge. The orbital contents herniate in to the
dren, possibly effecting speech or swallowing. However, pre- fracture and are then tapped when the bone recoils. This
sentations involving ophthalmic abnormalities are less causes impaired eye movements with associated diplopia and
common in children than adults and much less common in requires urgent surgery to prevent tissue ischemia.
pre-pubertal patients.
Ophthalmopathy is due to the inflammatory aspects of
Graves’ disease in the extraocular eye muscles and fat and
connective tissue surrounding the orbit. The resulting prop-
tosis may restrict eye movement and cause signs of periorbital
oedema. On CT this presents as orbital muscle thickening
enhanced with fatty strands around it. There is diffuse fusi-
form enlargement of the extraocular muscle belly however the
tendon is spared and there are usually no enhancing lesions
with intact bony boundaries of both orbits.
Carbimazole would be the first line treatment in this case.
Carbimazole and propylthiouracil are both antithyroid drug
treatments and are used to decrease thyroid hormone synthesis,
by acting as a preferred substrate for iodination by thyroid per-
oxidise. Propylthiouracil is usually not used first-line, due to a
small risk of severe liver injury (about 1 in 10,000 adults), except
in certain circumstances such as in the first trimester of preg-
nancy, or for treatment of thyroid storm. Antithyroid drugs are
used with aim to in short term preparation for radioiodine treat-
ment or surgery, medium-term inducing remission of Graves’
disease and long-term where radioiodine treatment or surgery is
contraindicated or declined. Radioiodine treatment is not recom-
mended for people with active Graves’ orbitopathy as it may cause
progression of the condition. Beta-blockers such as propranolol
may provide rapid relief of adrenergic symptoms, and may be the
only treatment needed for hyperthyroidism caused by thyroiditis. Figure 2 Xray of facial bones

PAEDIATRICS AND CHILD HEALTH 29:5 246 Ó 2019 Elsevier Ltd. All rights reserved.
SELF-ASSESSMENT

Once identified, orbital floor fractures should be referred to have a high false negative rate and do not contribute to sur-
the maxillofacial surgical team. Children with soft tissue gical planning (Figure 2).
entrapment should have urgent release of the soft tissues, Children presenting with an orbital floor fracture can oc-
ideally within 24 hours. casionally experience vomiting and an associated bradycardia
If orbital floor fracture is suspected, targeted imaging or syncope due to the oculocardiac reflex caused by increased
should be requested. High resolution head or orbital CT is the ocular pressure. This presentation is strongly associated with
gold standard however it delivers a high dose of radiation to inferior rectus muscle entrapment and can be mistaken for
the developing lens and is relatively poor at imaging soft tis- head injury, leading to delayed diagnosis.
sue. Although useful in adults, CT has been shown to under-
estimate the degree of soft tissue entrapment in children and FURTHER READING
MRI can be considered as an alternative. Wei, LA. et al. Pediatric orbital floor fractures. Journal of Amer-
Facial view X-rays may show herniation of the orbital ican Association for Pediatric Ophthalmology and Strabismus.
contents in to the maxillary antrum (teardrop sign, see 2011. 15(2). 173e180.
Figure 2) or a fluid level within the sinus; however, X-rays BMJ Best Practice. Orbital fractures. 2018.

PAEDIATRICS AND CHILD HEALTH 29:5 247 Ó 2019 Elsevier Ltd. All rights reserved.
SELF-ASSESSMENT

Self-assessment
Questions Question 3
How would you treat this child?
Case 1
a) Ceftriaxone IV 80mg/kg
A 2 year old female presents to the emergency department
b) Penicillin V PO for 10 days
with a 5 day history of fever up to 39.5 C and poor oral intake.
c) IVIG and aspirin 7.5mg/kg QDS
She has visited the GP twice since the onset of illness, firstly
d) Aciclovir IV
diagnosed with a viral upper respiratory tract infection, and on
e) Co-amoxiclav PO for 7 days
the second visit diagnosed with tonsillitis and prescribed
f) Supportive care
Penicillin V. Her mother is concerned that today the fever is
not responding to paracetamol and ibuprofen. There is no
history of foreign travel and she is fully immunised. Case 2
On examination, the child is alert and crying but appears A 13 year old male, previously fit and well, has been
miserable. She has a widespread blanching erythematous referred by his GP with fever up to 40 C for 6 days with
maculopapular rash. Heart sounds are normal with no mur- associated weight loss. He visited the GP on day 3 of the illness
murs present and respiratory examination is also normal. who diagnosed a likely viral illness. He revisited 2 days later
Abdominal examination reveals a soft, non-distended as the fever was not responding to anti-pyretic therapy and the
abdomen with no palpable organomegaly. ENT examination GP performed basic blood tests. The patient’s CRP was noted
reveals injected tympanic membranes and enlarged, red ton- to be 450 mg/dL so his GP urgently referred him to hospital for
sils with no apparent exudate. She has a red tongue and further investigations.
bilateral conjunctival injection. His two main complaints are fever, worse at night, and loss
Observations: Temperature 39.2 C, HR 145/min, RR 32/ of appetite. He has lost 6kg in the last 25 days. There is no
min, BP 80/40 mmHg and saturations 96% in room air. history of coryzal symptoms, cough, or night sweats. On ex-
amination, he appears cachectic and tired. There is a 2 cm by 1
Question 1 cm left-sided axillary lymph node palpable with some small
Which of the following results could support your diagnosis? cervical lymph nodes also present. His chest is clear and he
Please select THREE from the list. has normal heart sounds. He cannot walk due to pain in his
a) Elevated platelet count back and both legsalthough there are no obvious joint swell-
b) Blood culture positive for coagulase negative ings. He has a new widespread maculopapular rash on his face
Staphylococcus and trunk with a pink appearance.
c) Raised urea and creatinine His blood tests are shown below. He is treated with IV Cef-
d) Elevated transaminases triaxone and Azithromycin for two days and then switched to IV
e) Elevated CRP Meropenem due his symptoms not responding to treatment. He
f) Naso-pharyngeal aspirate positive for Rhinovirus continues to spike temperatures up to 39 C mostly overnight.
g) Elevated ESR Blood cultures are negative at 5 days, as are cultures for throat
h) Urine microscopy showing 50e100 WCC swabs and urine. His viral PCR screen (including EBV, CMV and
Toxoplasmosis) is also negative, as is a nasopharyngeal aspirate.
Question 2 He has no neurological symptoms and a CT head with
Which of the following investigations is subsequently most contrast was normal with normal CSF fluid microscopy, cul-
urgent? ture and biochemistry. His MRI spine is also normal. MRI of
a) Chest X-Ray his lower limbs reveals small amounts of fluid in the joint
b) Blood culture space of the left knee. Abdominal ultrasound shows no
c) Urine culture masses, however does reveal an enlarged liver and spleen. He
d) Bacterial throat swab has an MRI abdomen which shows an infiltrative hep-
e) Viral throat swab atosplenomegaly. Biopsy of his axillary lymph node reveals
f) Echocardiogram inflammatory cells with no blasts seen.
g) ECG
Question 1
h) Abdominal Ultrasound
What is the likely diagnosis?
a) Kawasaki Disease
b) Intracranial abscess
Anjalika Mallick MBBS MRes MRCPCH Paediatric ST4 Trainee, c) Mycoplasma infection
Department of Paediatrics, Northwick Park Hospital, London d) Influenza A
North West Hospitals Trust, London, UK. Conflicts of interest: e) Acute lymphoblastic leukaemia
none declared. f) Systemic onset JIA

PAEDIATRICS AND CHILD HEALTH 29:6 281 Ó 2019 Elsevier Ltd. All rights reserved.
SELF-ASSESSMENT

His blood tests are as follows:

Haemoglobin 7.4 g/dl


Haemoglobin 7.9 g/dl
White cell count 25.3 g/dl
White cell count 18.2 g/dl
Neutrophils 20.1 x 109/l
Neutrophils 14.6 x 109/l
Lymphocytes 5.1 x 109/l
Lymphocytes 3.1 x 109/l
Platelets 470 x 109/l
Platelets 400 x 109/l
Sodium 142 mmol/l
Sodium 143 mmol/l
Potassium 4.1 mmol/l
Potassium 2.8 mmol/l
Urea 3.2 mmol/l
Urea 3.0 mmol/l
Creatinine 60 mmol/l
Creatinine 55 mmol/l
ALT 40 U/l
ALT 25 U/l
ALP 200 U/l
ALP 100 U/l
Bilirubin 6 umol/l
Bilirubin 6 umol/l
Albumin 15 g/l
Albumin 22 g/l
CRP 438 mg/l
CRP 14.5 mg/l
ESR 560 mm/hr
Ferritin 420 ng/ml
Blood film Reactive

His stool culture sent for viral and bacterial screening is


negative and abdominal X-ray shows no signs of obstruction
g) Systemic lupus erythematosus with some bowel wall oedema in the ascending colon.
h) Hodgkins Lymphoma Question 1
i) Non Hodgkins Lymphoma What is the suspected diagnosis? (Choose ONE answer)
j) Sarcoma a) Salmonella Typhii
Question 2 b) Crohn’s Disease
Which investigation is the next most appropriate? c) Norovirus
a) Abdominal Ultrasound d) Shigella
b) Naso-pharangeal aspirate e) Ulcerative colitis
c) Bone marrow aspirate f) Campylobacter jejuni
d) MRI head g) Viral gastroenteritis
e) Echocardiogram h) Dysentery
f) PET scan i) Coeliac Disease
g) Autoimmune panel Question 2
Question 3 His symptoms improve and you discharge him with follow up
How would you treat this patient? investigations including: (Choose TWO)
a) Prolonged course of IV antibiotics a) FBC
b) High dose methylprednisolone b) Abdominal X-Ray
c) Oseltamivir phosphate (TamiFlu) c) Anti-TTG (tissue transglutaminase)
d) Erythomycin/Doxycycline d) Stool miscroscopy and culture
e) Chemotherapy e) Abdominal ultrasound
f) Surgery þ Chemo/Radiotherapy f) Stool for ova and parasites
g) Faecal calprotectin
Case 3 h) Upper gastrointestinal endoscopy
A 14 year old boy presents with a 3 day history of fever with i) CRP
associated diarrhoea. He returned from a holiday in Greece 7 days j) Anti-endomysial antibodies
ago where he ate chicken, fish and rice at the hotel. He has no k) Lower gastrointestinal endoscopy
coryzal symptoms and whilst he feels nauseated there is no history Question 3
of vomiting. He is having 10e12 loose motions daily and today he Which of the following complications has the strongest asso-
noticed a small amount of blood mixed in with the stool. ciation with this condition?
On examination he looks well. He is of slim build however a) Liver abscess
warm and well perfused. His heart sounds are normal and he has a b) HIV infection
clear chest. His abdomen shows generalized tenderness with no c) Osteomyelitis
guarding or rebound tenderness. He has normal bowel sounds. d) Primary sclerosing cholangitis
Rectal examination reveals no anal tags, fissures or haemorrhoids. e) Gastrointestinal Stromal Tumour

PAEDIATRICS AND CHILD HEALTH 29:6 282 Ó 2019 Elsevier Ltd. All rights reserved.
SELF-ASSESSMENT

f) Lymphoma Differential diagnosis should include the following:


g) Fistulae  Streptococcal infection
h) Anal fissures  Staphylococcal infection
i) Meningo-encephalitis  Measles, rubella, EBV, Influenza A þ B, Adenovirus, HHV-6
 Mycoplasma pneumonia
 JuvenileRheumatoid arthritis
Answers
 Stevens-Johnson syndrome
Case 1  Leptospirosis
1. A, E and G  Rickettsia
2. F  Drug reaction
3. C
Kawasaki disease is a systemic vasculitis of unknown aeti-
ology resulting in multisystem small and medium vessel
inflammation. It most commonly presents in children aged FURTHER READING
under 5 years with peak onset at 2e3 years of age. Kawasaki Brogan et al. Kawasaki Disease: an evidence based approach to
disease usually presents with high fever for greater than 5 days diagnosis, treatment and proposals for future research. Arch
with no focal source. This may be accompanied by bilateral, DisChild 2002; 86:286e290
non-suppurative conjunctivitis, dry and cracked lips, straw- Kawasaki Disease. Arch Dis Child Educ PractEd 2004; 89:ep3-
berry tongue and cervical lymphadenopathy. Often swelling ep8
and peeling of the hands and feet may be present. Giant cor- Kawasaki Disease: Summary of the American Heart Association
onary artery aneurysms (seen in up to 40% of untreated pa- Guidelines. American FamilyPhysician 2006; 74 (7)
tients), have been associated with fatal cardiac arrest. IVIG is Case 2
thus given to prevent their occurrence. 50% of coronary artery 1. F
aneurysms resolve within 5 years if treatment is initiated 2. C
promptly. Aspirin is given lifelong for confirmed cases of 3. B
coronary artery aneurysm to prevent aneurysm rupture. The Systemic onset juvenile idiopathic arthritis is seen in around
acute phase of the disease is associated with raised white cell 10% of cases of juvenile idiopathic arthritis. The incidence
count, CRP and ESR. The platelet count may initially be low or of sJIA is 10e20 in every 100 000 patients with an equal
normal however rapidly rises in the subacute phase of disease. predominance in male and females with a peak incidence
There may be deranged liver function due to hydrops of the between 18 months and 2 years. These children appear un-
gallbladder. Echocardiogram detects coronary artery aneu- well with significant constitutional symptoms and usually
rysms and is essential in short or long term management of present with systemic symptoms around 6 weeks prior to
this condition. joint complaints. There is often a morbilliform or salmon
Incomplete or partial Kawasaki presents with fever greater coloured rash. Often associated are internal organ involve-
than 5 days with 2 or 3 of associated criteria below. It is ment such as pleuritis, pericarditis and pericardial tampo-
important to consider early treatment in this cohort as some nade. Hepatosplenomegaly occurs in 70% of children.
cases progress to complete Kawasaki disease whilst some may Laboratory findings show signs of inflammation with
remain as incomplete Kawasaki disease. elevated CRP, ESR, white blood cell count and platelet count
with a microcytic or normocytic anaemia. The arthritis can
be extensive and refractory to treatment therefore these
children often have long term disability. There is no specific
diagnostic test for sJIA and as fulminant bacterial and/or
Criteria for diagnosis of Kawasaki disease (Major plus 4 Minor): viral infections can present in a similar fashion, treatment
Major Criteria with antibiotics and vigorous investigation for possible in-
Fever for a minimum of 5 days fections does take priority. It is important to rule out sepsis
Minor Criteria prior to initiating steroid therapy.
Changes in arms or legs: swelling of hands or feet, desquamation In this case, with a normal blood film and no malignant
of fingers/toes cells seen on lymph node biopsy, this is reassuring in terms of
Polymorphic exanthema involving trunk and extremities a malignant diagnosis however bone marrow aspirate for
Painless, non-supparative bilateral conjunctival injection histology and microbiology is definitive in confirming a diag-
Strawberry tongue, redness and cracking of lips, erythema of oral nosis of leukaemia. Sarcoma presents as bone pain or mass
mucosa and intracranial abscess would be evident on CT scan of brain.
Cervical lymphadenopathy Systemic lupus erythematosus could present with symp-
Reactivation of BCG scar toms of fever and weight loss however classically presents
No other explanation for symptoms with a butterfly rash, joint stiffness and pain. It also has a
female predominance.

PAEDIATRICS AND CHILD HEALTH 29:6 283 Ó 2019 Elsevier Ltd. All rights reserved.
SELF-ASSESSMENT

FURTHER READING
Arthritis Research UK: Centre for adolescent rheumatology www. Investigations for Result
centre-for-adolescent-rheumatology.org Ulcerative Colitis
Children’s Chronic Arthritis Association (CCAA) www.ccaa.org.uk FBC Anaemia, elevated
Oxford Paediatric and Adolescent Rheumatology Centre platelet count
(OxPARC) www.ouh.nhs.uk/oxparc ESR May be elevated
CRP May be elevated (usually mildly)
Case 3
Albumin Low
1. E
p-ANCA Positive
2. G and K
Faecal calprotectin Elevated in IBD, normal in functional
3. D
disease
Inflammatory bowel disease (IBD) is a term encompassing
MRI abdomen Bowel thickening,
Crohn’s disease and ulcerative colitis. Crohn’s disease can
inflammation, strictures
affect any part of the gut (mouth to anus) whereas ulcerative
Colonoscopy Inflammatory mucosal change. Histology:
colitis affects only the colon. Incidence of IBD peaks in
crypt abscesses, ulceration and
adolescence with a strong genetic susceptibility (30-fold
neutrophillic inflammation
increased risk in those with a first degree relative with IBD)
and environmental factors (most of which have not been
formally identified). In ulcerative colitis, symptoms include
abdominal pain, diarrhoea (often with blood and mucous),
urgency and tenesmus. Toxic megacolon is a life-threatening (a monoclonal antibody to tumour necrosis factor TNF) is
complication and surgical emergency characterized by used to manage both acute and chronic disease.
abdominal pain and distention, grossly dilated colon and high Maintenance treatment of ulcerative colitis is with amino-
risk of perforation. Extraintestinal manifestations include salicylate drugs (5-ASA) such as mesalazine oral steroid
primary sclerosing cholangitis, uveitis, arthritis and pyo- treatment and immunosuppressive ages such as 6-mercapto-
derma gangrenosum. Ulcerative colitis is associated with a purine or azathioprine. Colectomy with colostomy/ileostomy
higher rate of cholangiocarcinoma and bowel cancer in adult formation is performed as an emergency or elective procedure
life. for symptom management.
Initial management of ulcerative colitis flares includes in-
fectious screening and pain management (avoid NSAIDs). FURTHER READING
Nutritional support is required ideally involving normal European Society for Paediatric Gastroenterology Hepatology
enteral diet with supplements and monitoring of electrolytes. and Nutrition (ESPGHAN) http://www.espghan.org/fileadmin/
In severe cases, bowel rest and total parenteral nutrition are user_upload/guidelines_pdf/Updates/Management_of_
required þ/ surgical intervention. Intravenous steroid ther- Paediatric_Ulcerative_Colitis__Part.2.pdf
apy (for example methylprednisolone at 1mg/kg/day) leads to Turner D, Mack D, Leleiko N, et al. Severe pediatric ulcerative
symptom improvement in up to 70% of patients and is the first colitis: a prospective multicenter study of outcomes and
line medical treatment of ulcerative colitis. Infliximab predictors of response.Gastroenterology 2010; 138:2282e91.

PAEDIATRICS AND CHILD HEALTH 29:6 284 Ó 2019 Elsevier Ltd. All rights reserved.
SELF-ASSESSMENT

Self-assessment

Questions
Case 1
A 15 month girl is referred by her GP to the Emergency
Department due to severe pallor.
This is on a background of refusing to weight bear (following
normal walking at 10 months) and vomiting for the past 1
month. She has been off her food and her mother reports that for
the past month her diet has consisted mainly of milk.
The girl has had no fever, blood loss or stool changes with
the exception of a moderate degree of constipation.
There is no past medical history of note.
On examination, she appears irritable and has significant
conjunctival pallor. She is reluctant to walk and when placed
on her feet she is quite unsteady. She has brisk reflexes but
aside from this, the rest of her neurological examination is
normal though direct fundoscopy is not possible due to lack of
co-operation. She is afebrile and observations are within
normal limits.
Her full blood count reveals a severe microcytic hypo-
chromic anaemia with a haemoglobin of 35 g/litre. Her ferritin
is 2.9 mg/litre. A blood film also shows numerous pencil cells
and marked anisopoikilocytosis. White cell count and CRP are
normal. Figure 1 CT head (Case 1).
Q1. Which of the following would NOT cause a microcytic
anaemia? (Choose ONE)
Q5. Which of the following therapies should be initiated?
a. Iron deficiency
a. Low Molecular Weight Heparin (LMWH) for 3 months
b. Thalassaemia b. Low Molecular Weight Heparin for 6 months
c. Lead poisoning c. Warfarin for 3 months
d. Sideroblastic anaemia d. Warfarin for 6 months
e. Haemolytic anaemia
She is admitted to hospital where she receives a blood Case 2
transfusion. Over the next few days, she develops seizures and A nine-month-old boy was referred by his GP to the on-call
a fever of unknown origin. Following this, she becomes paediatric registrar with a history of being unwell for a week
drowsy with GCS 6e7. A head CT (without contrast) is per- with coryzal symptoms and reduced feeding. Over the last 24
formed (Figure 1). h his work of breathing has been increasing and mother re-
Q2. What is shown on this CT scan? (Choose TWO) ports a reduced number of wet nappies. He was born at 38
a. Cerebral venous thrombosis weeks gestation by caesarean section due to breech presen-
b. Hydrocephalus tation. The pregnancy was otherwise uneventful. He required
c. Thalamic infarction admission to SCBU for phototherapy and was discharged
d. Thalamic haemorrhage home at 5 days of age. Mother says he hasn’t been well since
Q3. Which of the following would be your next investiga- the age of four months with recurrent episodes of coughing
tion of choice? (Choose ONE) and a runny nose. Over the last three weeks he has been
a. CT angiography having very loose stools and occasional vomiting. The stool
b. MRI consistency has not improved after changing his milk to an
c. MR Venogram (MRV) amino acid based formula. He is the only child of unrelated
d. EEG parents. There is no significant family history of note.
Q4. Which of the following is NOT a risk factor for the On examination his height and weight are on the 2nd
development of venous sinus thrombosis? centile. He is alert but quiet. There is oral thrush. He has
a. G20210A mutation significant respiratory distress, tachypnoea and subcostal and
b. Factor V Leiden intercostal recession is observed. No wheeze or crackles are
c. Protein C or S deficiency present on auscultation. Examination of the abdomen reveals
d. Blood group O a liver edge palpable at 2 cms below the costal margin.

PAEDIATRICS AND CHILD HEALTH 29:7 323 Ó 2019 Published by Elsevier Ltd.
SELF-ASSESSMENT

A blood gas shows a significant respiratory acidosis and he Q2. What is the underlying diagnosis? (Select ONE answer
is electively intubated and ventilated. A bronchoalveolar ONLY).
lavage (BAL) is undertaken and silver stain performed on the a) BernardeSoulier syndrome
lavage material (Figure 2). b) Bloom syndrome
c) DiGeorge syndrome
d) Human immunodeficiency virus (HIV)
e) SchwachmaneDiamond syndrome
f) WiskotteAldrich syndrome (WAS)
g) X-linked agammaglobulinaemia (XLA)
h) X-linked severe combined immunodeficiency syndrome
(XSCID)
Case 3
A 15-year-old boy presents with an episode of involuntary
jerking movements of his right arm lasting a few minutes. He
has been unwell for the preceding 3 weeks, with coryzal
symptoms, sore throat and a headache. His previous medical
history is unremarkable and illicit drug use is denied. Exami-
Figure 2 Silver stain of BAL (Case 2). nation is normal, as are his renal profile, full blood count, C-
reactive protein, glucose and calcium. He is observed over-
Q1. What is the cause of this child’s respiratory symptoms? night, with no further episodes, and is discharged the
(Select ONE answer ONLY). following day.
a. Adenovirus pneumonitis He presents again 3 days later with worsening involuntary,
b. Chronic granulomatous disease (CGD) non-rhythmical movements of his head, neck and right arm
c. CMV pneumonitis and leg. The movements become more obvious during anxi-
d. Granulomatosis with polyangiitis (formerly termed Wege- ety, improve with distraction and disappear with sleep. The
ner’s granulomatosis) headache is unchanged. During this presentation, previous
e. Haemophagocytic lymphohistiocytosis (HLH) illicit drug use is disclosed, including cannabis, ecstasy and
f. Influenza A pneumonia amphetamines. He denies any use over the last 7 weeks. Ex-
g. Langerhan’s cell histiocytosis (LCH) amination is otherwise normal. He is admitted for observation
h. Pneumocystis jiroveci pneumonia (PJP) and continues to be symptomatic. Both CT of the head and
i. RSV bronchiolitis EEG are normal.
j. Tuberculosis Q1. Which of the following is the most appropriate diag-
Other investigation results are as follows: nosis? (Choose ONE)
Partial seizures
Full blood count analysis: Parkinson’s disease
Sydenham chorea
Hb 90 g/dL Tourette’s syndrome
WCC 9.84  109/litre Brain tumour
Platelets 170  109/litre He continues to be symptomatic. A urine toxicology screen
Lymphocyte 1.40  109/litre is positive for cannabinoids. All blood tests are normal, except
count for a raised antistreptolysin-O test (ASOT). He is commenced
CD3 0.0%  109/litre on haloperidol, which improves his symptoms. Two days
CD19 96.0%  109/litre later, he complains of a strange sensation in his tongue. There
CD16þCD56þ 1.0% 0.01  109/litre are no abnormalities noted on examination. Two hours after
CD3þCD4þ 0.0% 0.00  109/litre this, he becomes distressed. His eyes are rolled up with
CD3þCD8þ 0.0% 0.00  109/litre involuntary protrusion of his tongue and a backward tilt to his
head. He is able to make controlled movements with effort. All
reflexes are normal.
Q2. Which of the following is now the most appropriate
Immunoglobulins: diagnosis?
Partial seizures
IgG 2.4 g/litre (39) Complex seizures
IgA <0.02 g/litre (0.150.7) Dystonic reaction
IgM 0.2 g/litre (0.41.6) Supratentorial factors
Tardive dyskinesia

PAEDIATRICS AND CHILD HEALTH 29:7 324 Ó 2019 Published by Elsevier Ltd.
SELF-ASSESSMENT

Q3. Which of the following drugs would be the most PJP is common in adult patients with HIV and should be
appropriate to treat his ongoing choreoform movements? considered in other groups of immunocompromized patients
Penicillin such as those with haematological malignancies, history of
Carbamazepine solid organ or bone marrow transplant and chronic use of
Lamotrigine steroids or other immunosuppressive drugs.
Ethosuximde The clinical presentation of PJP is non-specific; patients
Erythromycin commonly present with fever, tachypnoea, cough and hyp-
oxia. Chest auscultation is usually clear. Changes on chest X-
ray are typical of interstitial pneumonitis with diffuse pulmo-
Answers
nary infiltrates; other changes observed may include pneu-
Case 1 matocoeles and pneumothorax. CT scanning is sensitive and
Answers: E, A & C, C, D and A indicated when chest X-ray findings are not helpful. Typical
All answers except haemolytic anaemia cause a microcytic CT appearances are of ground glass attenuation with inter-
anaemia. Haemolytic anaemia presents with a normocytic lobar septal thickening. If clinical suspicion is high and
hypochromic anaemia. In this girl’s case, iron deficiency is the sputum culture is negative, BAL may be performed. Pneumo-
most likely cause of her microcytic anaemia. Iron deficiency is cystis jirovecii cannot be grown in vitro, so it is identified in
a common condition affecting up to 25% of children world- sputum or BAL fluid with stains such as Geimsa and Diff-Quik,
wide. During infancy and adolescence, poor intake is the to detect trophozoite and cyst forms, or methenamine silver
commonest underlying cause. However malabsorption, for and toluidine blue which identify the cyst wall. The image
example due to Coeliac disease, and blood loss should be here shows a cluster of cysts stained in black with silver stain
excluded. with host tissue counter-stained green. Alternatively, direct
The head CT shows deep cerebral venous thrombosis and immunofluorescence with monoclonal antibodies can be used.
resulting thalamic venous infarction. MR Venogram is the best In future, serum b-D-glucan levels may be used routinely to
imaging modality for cerebral sino-venous thrombosis help support or refute a PJP diagnosis.
(CVST). The MRV in this case confirmed the findings from CT Treatment of PJP is with high dose co-trimoxazole. Dura-
showing bilateral thrombosis of the internal cerebral veins and tion of treatment is generally three weeks in patients with
basal veins of Rosenthal with resulting bilateral thalamic underlying HIV and two weeks in other patients. If patients are
venous infarction. Her pyrexia of unknown source is also a not clinically improving within a few days, alternative thera-
result of thalamic involvement. pies such as pentamidine should be considered. Corticoste-
Blood group O is actually associated with an increased roids may also be used as an additional treatment option in
bleeding risk. In this particular child, no coagulopathies were patients with PJP.
identified but rather her severe iron deficiency anaemia (IDA) Adenovirus or RSV pneumonia are differential diagnoses
was the likely cause of CVST. Several cases have been for severe respiratory distress in an infant, although they are
described in the literature where children developed stroke usually associated with crepitations and wheeze on examina-
associated with anaemia. A number of mechanisms may tion. Furthermore, radiological findings are more likely to be
explain this association. Firstly, iron deficiency can lead to reticulonodular than diffuse. The presentation of CMV pneu-
thrombocytosis, possibly secondary to increased erythropoi- monitis is very difficult to distinguish clinically from PJP e
etin levels. Furthermore, the microcytosis seen in IDA can there may also be few changes on auscultation and radiolog-
cause abnormal haemodynamics with reduced deformability ical features on chest X-ray and CT will be similar in both
and increased viscosity, again predisposing to thrombosis. conditions. Here, the BAL histology is diagnostic for PJP.
Besides supportive therapy, the mainstay of management Specific tests for CMV include a rise in CMV IgM, anti-CMV
for CVST is anticoagulation with low molecular weight hepa- early antigen monoclonal antibody testing on neutrophils or
rin. This promotes recanalization of the thrombus as well as direct PCR for CMV in blood, BAL fluid or sputum. Chronic
reducing the risk of further thrombosis though this has to be granulomatous disease (CGD) may present in a young infant
weighed against a risk of further haemorrhagic infarction. with a background of failure to thrive, diarrhoea and hep-
MRV should be repeated at 3 months at which point anti- atosplenomegaly. However, in CGD, cutaneous manifestations
coagulation can be discontinued if complete recanalization has are usually present and lung disease is pneumonia with ab-
occurred. Otherwise, therapy should be continued for a min- scesses and granulomas, rather than interstitial disease.
imum of 6 months. Granulomatosis with polyangiitis is a rare multi-system dis-
order. CT changes may include diffuse ground glass changes,
FURTHER READING but often also diffuse alveolar haemorrhage, and patchy
Khan A, Hussain N, Gosalakkal J, et al. Severe anemia causing consolidation. Histology would show vasculitis, necrosis and
cerebral venous sinus thrombosis in an infant. J Pediatr granulomatous inflammation. The presentation described here
Neurosci 2012; 7: 30e32 would also fit with TB, but examination findings would be
Case 2 more impressive, with positive ZiehleNeelsen staining for
Answers: H and H acid fast bacilli (AFB) on sputum. This presentation is not

PAEDIATRICS AND CHILD HEALTH 29:7 325 Ó 2019 Published by Elsevier Ltd.
SELF-ASSESSMENT

consistent with LCH in an infant, where cutaneous changes Case 3


would be more common; lung disease tends to occur in older Answers: A, B and C
children and adults and is associated with cystic changes and In the case described, the history of drug use and positive
pneumothorax. HLH again may present with an unwell infant urinary cannabinoids are confounding factors. The presenting
and a history of failure to thrive, but the child would be sys- symptoms of chorea are not classically associated with drug
temically unwell with lymphadenopathy and features of bone withdrawal. Parkinson’s disease can also be excluded; not
marrow, skin, liver and CNS disease, rather than pulmonary. only is this extremely rare in the paediatric population, but the
The results here show normal numbers of CD19þ B cells, main clinical features are tremor and rigidity. It is also pro-
but impaired B cell function with low immunoglobulins, gressive in nature. The presenting features closely resemble
impaired cytotoxic CD4þ and CD8þ T cell numbers, as well as those of Tourette’s syndrome, and indeed the symptomatic
low CD16þ CD56þ natural killer (NK) cells, a combination treatment is very similar. The distinguishing features in this
which only occurs with severe combined immunodeficiency, case are that the movements are not voluntarily reproducible
or SCID. This term covers a group of inherited disorders of (as usual in Tourette’s) and the raised ASOT, indicating a
which the most common type is X-linked SCID (XSCID) streptococcal infection.
because the mutated gene, which normally produces a re- Sydenham chorea (‘St Vitus dance’) is the most common
ceptor for activation signals on immune cells, is located on the acquired chorea of childhood and a cardinal feature of rheu-
X chromosome. The classic symptoms of SCID include an matic fever. Alone, it is enough to diagnose rheumatic fever.
increased susceptibility to a variety of infections, including Chorea occurs in approximately 10e20% of patients, with a
acute otitis media, pneumonia, oral thrush and diarrhoea. female to male ratio of 2:1. Most patients present between the
Because children with SCID experience multiple infections, ages of 5 and 15 years, with a peak in adolescence.
they fail to grow and gain weight as expected (i.e., failure to The underlying pathological process is thought to be an
thrive). Children with untreated SCID rarely live past the age immune-modulated response to a group A b-haemolytic
of two years. The most effective treatment for SCID is bone Streptococcus infection. Antineuronal antibodies cross-react
marrow and stem cell transplantation. with basal ganglial neurons, causing an imbalance in the
BernardeSoulier is a group of hereditary platelet disorders dopaminergic, cholinergic and gamma-aminobutyric acid
and would have thrombocytopenia, giant platelets and pro- (GABA) systems.
longed clotting times, but not low immunoglobulin, T cells or The features are an insidious onset of rapid, purposeless,
NK cells. Patients with Bloom syndrome (congenital telangi- non-repetitive, involuntary movements that are worse with
ectatic erythema) may have reduced immunoglobulins, but stress and disappear with sleep. Hypotonia and muscular
not reduced T or NK cells. DiGeorge syndrome can be asso- weakness can also be present. Symptoms can occur up to 6
ciated with low T cells, particularly the CD3þ/CD4þ popula- months after the initial infection. The diagnosis is clinical,
tion, and impaired T cells can affect B cell function, with with most investigations unremarkable. There may be evi-
concomitant low immunoglobulin levels. In this case, both the dence of a previous streptococcal infection.
T and NK cell levels are profoundly reduced, which is not The prognosis is good, with most cases resolving within 3
consistent with a diagnosis of DiGeorge syndrome. Infants e6 months. Severe cardiac involvement is rare in patients
with HIV may have a low CD4þ T cell count, neutropenia and with chorea.
thrombocytopenia, but hypogammaglobulinaemia is associ- Treatment of Sydenham chorea is symptomatic. Occa-
ated with end end-stage disease only, and the NK cell popu- sionally, sedatives such as diazepam are required if the
lation should not be reduced. SchwachmanneDiamond is an movements are self-injurious. Dopaminergic blockers such as
autosomal recessive disorder characterized by exocrine haloperidol are effective, but have a well-known side effect
pancreatic failure, skeletal abnormalities and bone marrow profile including tardive dyskinesia (which can persist
failure e such patients have pancytopenia on FBC estimation. following drug withdrawal) and dystonic reactions. Dystonic
WiskotteAldrich syndrome patients show thrombocytopenia reactions are treated with antimuscarinic drugs such as pro-
with small platelets, usually have isolated low IgM only, low cyclidine, which was used in this patient. Anticonvulsants
CD8þ T cells, but normal NK cells. X linked agammaglobuli- such as carbamazepine and sodium valproate are thought to
naemia (XLA) is caused by mutations in the gene for Bruton’s act on the GABA system and are also effective in controlling
tyrosine kinase, affecting maturation of B cells, so CD19þ B the choreoform movements. Penicillin is used to treat the
cells will be reduced with very low immunoglobulin levels, source of the infection.
but normal T cell counts.
FURTHER READING
FURTHER READING Mohamed I, Moorjani B. Chorea in children. www.emedicine.
Punpanich W, et al. Antibiotic and systemic therapies for pneu- com/neuro/topic644.htm, accessed August 2006.
monia in HIV-infected and HIV-exposed children. J Infect Dev Sydenham’s chorea. In: The Merck manual of diagnosis and
Ctries 2012; 6:109e19. therapy. Section 19, chapter 271. Neurological disorders.
Stern A, et al. Prophylaxis for Pneumocystis pneumonia (PCP) in www.merck.com/mrkshared/mmanual/section19/chapter271/
non-HIV immunocompromised patients. Cochrane Database 271a.jsp, accessed August 2006.
Syst Rev 2014; Issue 10. Art. No.:CD005590.

PAEDIATRICS AND CHILD HEALTH 29:7 326 Ó 2019 Published by Elsevier Ltd.
SELF-ASSESSMENT

Self-assessment
Questions
Motor NCS
Case 1
A 4-year-old boy presents to the paediatric emergency Nerve/ Latency Amp.1 Dur.(ms) Dist. Vel.(m/
department with a 1-day history of right hip pain that left him Sites (ms) e2 (cm) s)
screaming and crying all night. There is no history of injury, (mv)
but his parents think this could be related to him walking long L COMM PERONEAL e EDB
distance during a day trip. They were also concerned about his Ankle 8.05 ([) 0.1 6.00
delayed motor development including walking at 18 months, (Y)
poor balance and inability to jump. His father has recently Fib 24.20 0.5 7.20 20 12.4
been diagnosed with a neurological condition of which several Head (Y) (Y)
paternal relatives are also affected. L TIBIAL (KNEE) - AH
On examination, his observations were all within normal Ankle 11.05 0.4 4.35
range. His gait showed mild foot drop. There was bilateral pes ([) (Y)
cavus with mild toe clawing. No obvious joint pathologies
were present. He was hypermobile in his thumb, fingers, el-
bows and knees (Beighton score 7/9). Muscle bulk and tendon
reflexes in the lower limbs were reduced. Abdominal exami- C. Charcot Marie Tooth disease
nation was unremarkable. D. Familial amyloid polyneuropathy (FAP)
1. What investigations are most likely to yield a diagnosis? E. Refsum’s disease
Select TWO answers. F. Chronic dysimmune neuropathy
A. MRI spine
3. What is the aetiology of this condition? Select TWO
B. DNA testing
answers.
C. Nerve conduction studies
A. X-linked recessive inheritance
D. Nerve biopsy
B. Autosomal dominant inheritance
E. Anti-GM1 antibodies
C. Autosomal recessive inheritance
F. Lumbar puncture
D. Axonal death and Wallerian degeneration of sensory
He subsequently had nerve conduction studies (NCS) and
and motor nerves
an EMG performed, with the following results.
E. Peripheral demyelinating neuropathy

Sensory NCS
Case 2
A 10-day-old male infant was seen in the paediatric Emergency
Nerve/ Rec. Site Latency Amp.2 Department with his mother. She reported the baby had
Sites (ms) e3 multiple episodes of vomiting fresh blood within the last 24 h.
(mV) There was also blood in the stool, and more recently oozing of
L SURAL e Lat Malleolus blood from the umbilical stump. He had been exclusively
Calf Lat Absent bottle fed and mum said he had been difficult to wake for feeds
Malleolus (Y) since birth. He was born at 39 weeks by emergency C-section
due to foetal bradycardia and was given two days of intrave-
nous antibiotics following maternal pyrexia in labour, but was
well on discharge. No family history of bleeding disorder was
All abnormal values are marked in bold. An up arrow ([) noted.
indicates values greater than reference range; a down arrow On examination, he had a sunken anterior fontanelle and
(Y) indicates values smaller than reference range. cold extremities, but perfusion was normal with CRT less than
2. What is the most likely diagnosis? Choose ONE answer 2s. No petechiae or bruises were noticed. He was afebrile;
only. intermittently tachypnoeic but with no other signs of respira-
A. Ehlers-Danlos syndrome type 3 tory distress. Chest examination was unremarkable. Abdomen
B. Muscular dystrophy was soft, non-distended but the lower edge of liver was
palpable.
Vitals
Heart rate: 150/min, Temperature: 36.3  C, Respiratory
rate: 68/min, SpO2: 99 %, BP: 80/35 mmHg

PAEDIATRICS AND CHILD HEALTH 29:9 411 Ó 2019 Published by Elsevier Ltd.
SELF-ASSESSMENT

1. What initial investigations would you like to perform first? finger-to-nose test, particularly on the right side. The rest of
Select FOUR answers. the physical examination was normal.
A. Abdominal X-ray 1. What investigation should be carried out urgently? Choose
B. capillary blood gas ONE answer.
C. Stool culture A. Lumbar puncture
D. Partial septic screen (FBC, CRP, urine sample, blood B. Full blood count
culture) C. Toxicology screening
E. Liver function tests including clotting D. Neuroimaging
F. U&Es E. Metabolic profile
G. Blood film F. All of the above
Capillary blood gas showed marked metabolic acidosis G. None of the above
with a lactate of 9.8. Initial blood tests showed liver
2. What is the most likely cause of his ataxia? Choose ONE
dysfunction, prolonged PT, aPTT and thrombocytopenia.
answer.
He was given Vitamin K, intravenous antibiotics and fluid
A. Miller Fisher Syndrome (MFS)
boluses, and admitted to Paediatric Intensive Care Unit for
B. Acute post-infectious cerebellar ataxia (APCA)
further management. Blood products were given in an attempt
C. Acute disseminated encephalomyelitis (ADEM)
to correct his bleeding diathesis.
D. Cerebellar stroke
2. What is the cause of his deterioration? Choose ONE answer.
E. Meningitis
A. Macrophage activation syndrome
B. Disseminated intravascular coagulation 3. What is the most appropriate management? Choose ONE
C. Thrombotic thrombocytopenic purpura answer.
D. Haemolytic uraemic syndrome (D-) A. Watchful waiting
Additional screening tests for infectious agents were B. 5-day course of oral Aciclovir
requested and he was started on empiric IV acyclovir. His C. Intravenous antibiotics
blood PCR then came back positive for high level of HSV2 D. Oral prednisolone
DNA. E. Intravenous immunoglobulins
3. What is most effective at reducing HSV related neonatal
deaths? Choose ONE answer.
A. Send CSF for HSV PCR for every suspected neonatal
Answers
sepsis case and start empiric Aciclovir treatment. Case 1
B. Consider HSV as a differential diagnosis for sepsis, Q1 B and C
especially in critically ill neonates that are not Q2 C
responding to treatments. Q3 B and E
C. C-section in women with active genital herpes lesion Charcot Marie Tooth disease Type 1a
D. Antiviral prophylaxis during pregnancy The abnormal examination findings in the lower limbs
Case 3 alongside developmental motor delay and a positive family
A 2-year-old boy is referred to the Paediatric Assessment Unit history raise strong suspicion of a hereditary neuropathy. The
with rapid onset ataxia. He started becoming a bit wobbly diagnostic process often involves electrophysiological studies
yesterday morning, which gradually worsened throughout the (EMG and nerve conduction studies) and molecular genetic
day. Today he is unable to stand without support. His mum testing. As molecular testing is now easier to perform and
also noticed that he is unable to hold a spoon without spilling readily available, nerve biopsy is usually of no additional
the content, and his speech is more slurred than usual. He has value. Nonetheless, it can still be useful in some cases to
fallen several times and seems to be increasingly frustrated by narrow down differential diagnoses and guide the direction of
his impaired activity. There is no known history of trauma or genetic studies.
ingestion. He currently has a mild chicken pox infection that The nerve conduction studies (NCS) showed an absent
started 8 days ago but has been otherwise healthy and meeting sural potential together with reduced motor amplitudes and
all developmental milestones. Family history is unremarkable. significant impairment of conduction velocity (less than 35m/
On examination, he was alert, smiling and playful. His s), which is consistent with a demyelinating neuropathy.
observations were as follows: Charcot Marie Tooth (CMT) disease is the most common
BP 85/48 mmHg, Pulse 99, Temperature 36.4  C, Resp 24, inherited peripheral neuropathy. Other neuropathies that can
SpO2 100% present with lower limb weakness need to be differentiated
Multiple crusted chicken pox lesions were seen. When from CMT: FAP and Refsum’s disease are inherited conditions
asked to walk, he had a staggering, wide-based gait. On characterized by neuronal deposits of amyloid and phytanic
neurological examination he had no nystagmus, pupillary acid respectively, therefore the onset requires accumulation
abnormalities or facial asymmetry. Tone, power and reflexes over time and is typically after adolescence. Dysfunction of
were normal in all limbs. There was some past pointing in the other organ systems is also typical. Chronic dysimmune

PAEDIATRICS AND CHILD HEALTH 29:9 412 Ó 2019 Published by Elsevier Ltd.
SELF-ASSESSMENT

neuropathy is an acquired immune-mediated neuropathy with predominantly activated. In addition, specific features in TMA
fluctuating severity but is still progressive overall. are useful for diagnosis: a low level of ADAMTS13 in TTP, and
CMT has over 80 causative genes but there are two main presence of Shiga-toxin in HUS. Macrophage activation syn-
forms: CMT1 (demyelinating) and CMT2 (axonal). The divi- drome is a hyperinflammatory syndrome that is more
sion is based on the nerve conduction velocity (NCV): con- commonly associated with rheumatic disease. Although it
duction slower than 35m/s indicates CMT1 and NCV greater shows features of DIC, the onset typically involves non-
than 45m/s indicates CMT2. Further classification depends on remitting high fever and severe pancytopenia in all three
genetic testing and is useful for the purpose of prognosis and blood cell lines, which does not fit with this case.
genetic counselling. CMT1 is the more common form, of Two observational studies in the US and UK have shown
which 90% are CMT1a. CMT1a is caused by a mutation in the that on average, ED sees one neonatal HSV (nHSV) patient per
peripheral myelination protein-22 (PMP22) gene which results year, and the incidence is rising. Mortality in babies with
in severe demyelination in sensory (A-beta, A-delta and B fi- disseminated HSV infection prior to Aciclovir treatment is
bres) and motor (A-alpha) nerve fibres and impaired con- 85% and can be reduced to 29% following early treatment.
duction velocity. This is what the patient had in this case. However, validated clinical predictors for HSV infection are
CMT2 involves direct axonal death and Wallerian degenera- lacking, likely due to the low incidence and the variable, non-
tion, therefore conduction velocity is preserved or only mildly specific initial presentations of this infection. Starting empiric
reduced. In general, CMT2 patients tend to be less disabled Aciclovir treatment on every suspected neonatal sepsis case
and have less sensory loss than those with CMT1. runs the risk of exposing a large number of infants to the
potential side effects of Aciclovir and prolonging their hospital
FURTHER READING stay. It is very important, though, to always consider nHSV
Pareyson D, C. Marchesi. Diagnosis, natural history, and man- infection as a differential diagnosis for neonatal sepsis espe-
agement of Charcot-Marie-Tooth disease. Lancet Neurol., 8 cially when presented together with liver dysfunction.
(2009), pp. 654-667 NHSV is most commonly acquired peri- and postpartum;
Case 2 intrauterine infection is possible but rare. HSV2 accounts for
Q1 A, B, D and E 80% of all cases. The risk of disseminated nHSV is higher if
Q2 B the infection is newly acquired by mother, as there is no time
Q3 B for the immune system to generate neutralising antibodies that
Neonatal herpes simplex virus infection can be delivered to the baby via the placenta. Elective C-sec-
The presence of blood in vomit and stool could potentially tion reduces, but does not completely eradicate HSV infection.
indicate a serious GI problem such as necrotising enterocolitis, Prophylactic antiviral therapy during late pregnancy is
intestinal malrotation or intussusception, which should be becoming an increasingly common clinical practice, however
ruled out first by an abdominal X-ray. In the acute setting, there is no definitive evidence supporting its safety and effi-
capillary blood gas serves as a quick and non-invasive test to cacy, and subclinical viral shedding still occurs despite
monitor ventilation, electrolytes and acid-base balance. Liver suppression.
function with coagulation profile is essential in any child
FURTHER READING
presenting with bleeding. Although most observations were
Kimberlin David W.. Neonatal herpes simplex infection. Clin
within the normal range, sepsis is an important differential
Microbiol Rev Jan 2004, 17 1-13
diagnosis, especially considering his increased risk of infection
Mahant S., Berry J.G., Kimberlin D.W. Neonatal HSV disease:
due to maternal pyrexia during labour. Thus, completing an
balancing the low incidence with the need to treat promptly.
infection screen involving FBC, CRP, blood culture and uri-
(2018) Pediatrics, 141 , art. no. e20173647Pubmed Partial
nalysis would be appropriate, withholding the lumbar punc-
atlno unk
ture until clotting has returned to normal.
In this case, disseminated intravascular coagulation (DIC) Case 3
secondary to sepsis from disseminated HSV2 infection was the Q1 G
diagnosis. The raised inflammatory cytokines stimulate tissue Q2 B
factor-dependent coagulation, which is strong enough to Q3 B
overcome the effects of antithrombotic regulators such as Acute postinfectious cerebellar ataxia
protein C and antithrombin, allowing thrombosis to take place Initial evaluation of a child presenting with ataxia should
throughout the vasculature. This causes microvascular oc- focus on excluding the serious and treatable causes such as
clusion, thromboembolism and multiorgan failure. At the CNS infections, intoxication, and space occupying lesions. The
same time, DIC rapidly consumes platelets and clotting fac- temporal course of ataxia is diagnostically useful: onset of less
tors, which manifests as bleeding. Thrombotic micro- than 72 h makes acute traumatic, toxic or infectious causes
angiopathies (TMAs) such as haemolytic uraemic syndrome more likely, whereas recurrent or chronic ataxias are sugges-
(HUS) and thrombotic thrombocytopenic purpura (TTP) may tive of an underlying problem such as metabolic syndromes,
resemble DIC superficially. However, in TMA the coagulation congenital malformations or a brain tumour.
profile (aPTT, PT) is normal, as only the platelet pathway is

PAEDIATRICS AND CHILD HEALTH 29:9 413 Ó 2019 Published by Elsevier Ltd.
SELF-ASSESSMENT

Urgent investigations should be targeted on the basis of Miller Fisher Syndrome is a rare variant of GBS that can
clinical suspicion. Obtaining toxicology screening without a present predominantly as ataxia. However, it can be distin-
known history of ingestion is likely to be low-yielding in the guished from APCA by the presence of anti-GQ1b and absent
absence of safeguarding concerns. CSF studies showing mild limb reflexes. ADEM is differentiated from APCA in that it
lymphocytic pleocytosis and elevated protein is a common and typically involves focal neurological deficits and altered
non-specific finding of postinfectious cerebellar disorders, mental status, thus necessitating rapid imaging: MRI would
hence LP has little diagnostic value. There is no indication for show polyfocal demyelinating CNS lesions. Cerebellar stroke
imaging in this child as he did not have worrying signs of is an extremely rare cause of childhood ataxia that is often
raised intracranial pressure or infection, and imaging would accompanied by headache and visual changes. Bacterial
most likely be normal. Ataxia caused by metabolic disorders meningitis is also an unlikely cause in this case as there was
are often episodic but can resemble acute ataxia at the first no fever or meningism.
attack. However, there may be other clues such as pre- Given the most likely diagnosis of APCA, the ataxia is self-
prandial worsening of ataxia and developmental delay. limited and will resolve spontaneously over 10e21 days.
After excluding other possible causes, a clinical diagnosis Treatment with oral Aciclovir has the potential to shorten the
of acute post-infectious cerebellar ataxia (APCA) can be made course of the Varicella infection thus allowing the child to
based on the history of rapid onset ataxia, prodromal Varicella return to his normal activities quicker.
infection and the presence of pure cerebellar symptoms. APCA
most commonly affects preschool-age children following an FURTHER READING
infection. Frequent causes include Varicella, Mycoplasma, Caffarelli M, Kimia AA, Torres AR. Acute ataxia in children: a re-
EBV, HSV, Parvovirus B19 and enteroviruses. The underlying view of the differential diagnosis and evaluation in the emer-
cause of APCA has not been fully established but is believed to gency department. Pediatr Neurol 2016; 65: 14e30.
be autoimmune in nature.

PAEDIATRICS AND CHILD HEALTH 29:9 414 Ó 2019 Published by Elsevier Ltd.
SELF-ASSESSMENT

Self-assessment
Questions d) Fluid bolus followed by iv maintenance fluids 0.9% saline/
5% dextrose þ10mmol potassium per 500ml bag
Case 1
e) Fluid bolus followed by iv maintenance fluids 0.9% saline/
A 3-year-old girl was referred to paediatrics with a few weeks’
5% dextrose
history of fatigue, ‘growing pains’ at night and frequent fevers
f) Hyperhydration 2 litres/m2/day with maintenance potas-
and upper respiratory tract infections.
sium added
On admission her temp is 38.2 C, BP 95/50, heart rate 180
g) Hyperhydration 2 litres/m2/day
beats per minute, respiratory rate 35 breaths per minute and
h) Allopurinol
oxygen saturations 98% in air. She appears a bit pale, has a
i) Blood transfusion
few petechiae on her arms and bruises on her shins and but-
tocks. She has a soft systolic heart murmur, her chest is clear, Case 2
she has no respiratory distress and the tip of the spleen is felt 1 The 3-year-old girl in the previous scenario is diagnosed with
e2cm below the costophrenic angle. She has red, enlarged acute lymphoblastic leukaemia (ALL) at her primary treatment
tonsils with no exudate and a slight white coating to the centre (PTC). She commences treatment as per the
tongue. UKALL2011 trial protocol, receiving induction chemotherapy
Her full blood count is: with standard dexamethasone (6mg/m2/day for 28 days),
Hb 6.5g/dl vincristine, asparaginase and daunorubicin and intrathecal
White blood cells 64 x 109/l methotrexate. She has a portacath device inserted for central
Platelets 25 x 109/l venous access.
She is discharged home on day 10 of the protocol with a
Question 1
plan to return to the PTC for her next chemotherapy, dauno-
What would be the most helpful investigations at this stage?
rubicin and vincristine, which is due on day 16. However she
Please select FIVE
feels unwell at home and shivery but does not have a fever.
a) Skeletal survey & ophthalmology review
The parents call the PTC for advice and are told to come to
b) Blood film
your district general hospital (DGH) for a review.
c) Blood culture
On arrival she is alert and co-operative but appears tired,
d) Routine coagulation studies
pale and a bit listless. Her parents say she has not eaten
e) Urea & electrolytes, liver function studies & CRP
anything today and had a small vomit. Her chest is clear, she
f) Urea & electrolytes, liver function studies, calcium, phos-
has no rashes and the portacath site looks as though it is
phate & urate
healing cleanly. Her tongue has a white coating.
g) Chest x-ray
Observations: temperature 36 C, heart rate 120 beats per
h) Abdominal ultrasound
minute, respiratory rate 16 breaths per minute, oxygen satu-
i) Throat swab
rations 99% in air.
j) Urinalysis
Her potassium is 5.5mmol/litre and she has only passed Question 1
urine once in the last 24 hours. What would be appropriate management? (Choose one or
Question 2 more options)
What is the most likely diagnosis: Please select ONE a) Take blood for full blood count, CRP & cultures but await
a) Non accidental injury and neglect results before starting antibiotics. You do not know
b) Sepsis whether she is neutropaenic and she does not have a fever
c) Acute meningococcal septicaemia b) Take blood for full blood count, CRP & cultures and start iv
d) Acute lymphoblastic leukaemia, ALL piperacillin/tazobactam
e) Acute myeloid leukaemia, AML c) Wait for urine collection before starting any antibiotics
f) Henloch Schonlein Purpura, HSP d) Do an ineout catheter if she is not able to give a clean-
catch urine sample
Question 3 e) Only send the urine for culture if it is leucocytes and/or
What would be your next steps in management? Please choose nitrites positive
THREE f) Take a group & save and transfuse if necessary
a) Referral to social care
b) Iv ceftriaxone Question 2
c) Iv piperacillin/tazobactam What other supportive care measures should be in place?
a) Co-trimoxazole 2 days per week
b) Antiemetics as required
c) Nystatin treatment course

PAEDIATRICS AND CHILD HEALTH 29:10 455 Ó 2019 Elsevier Ltd. All rights reserved.
SELF-ASSESSMENT

d) Nystatin treatment course followed by nystatin or flucon- The infant was discharged on gaviscon but seen 3 days
azole prophylaxis later by her GP with ongoing crying and vomiting and was
e) Regular chlorhexidine mouth washes prescribed ranitidine in addition to gaviscon. Eight days later
f) Varicella Zoster Vaccine (VZV) for household non-immune she saw the GP again as there was no improvement and was
contacts commenced on an extensively hydrolysed formula. Two days
g) Avoid school/nursery whilst neutropaenic later the health visitor sent her back to the GP due to poor
Your patient makes a good recovery from this episode and weight gain and there was now a history of blood and mucus
completes her induction chemotherapy and moves onto the in the stool and more forceful vomiting and she was sent back
consolidation phase of treatment, which includes cyclophos- to paediatrics. She had a small, faint bruise on her face below
phamide, cytarabine, mercaptopurine and intrathecal the right eye. The mother said she hit herself in the face whilst
methotrexate. screaming.
On day 11 of this regimen she wakes up late after a difficult Bloods:
night’s sleep with lots of waking and crying and being difficult to Hb 83
console. She is aggressive and combative in the morning, which Platelets 801
is very unusual for her and then has a brief tonic-clonic seizure at White cells 17
home and is unrousable thereafter. Her parents call an ambu- Coagulation profile normal
lance. In the ambulance she has no further abnormal movements
Question 2
but is unresponsive with poorly reactive pupils and a blood sugar
What investigations/management might be appropriate at this
of 7mmol/litre. She is afebrile. She is intubated and ventilated in
stage? Chose best FOUR
A&E, commenced on iv antibiotics and antivirals and transferred
a) Blood gas
to paediatric intensive care. She self-extubates the next day and
b) Ultrasound abdomen
appears neurologically normal. Her lumbar puncture has 5 white
c) Continue extensively hydrolysed formula
cells and 600 red cells per mm3 and no organisms seen.
d) Switch back to standard formula
Question 3 e) Start amino acid formula
What is the differential diagnosis? Choose THREE most likely f) Omeprazole
a) Viral meningitis/encephalitis g) skeletal survey, CT head & ophthalmology review
b) Bacterial meningitis/encephalitis All the investigations undertaken were normal and she was
c) Aseptic meningitis discharged on an amino acid formula with a plan to review in
d) Central Nervous System relapse one week’s time. Two days later she was still vomiting but it
e) Cyclophosphamide encephalopathy was thought she had not had a long enough trial of the new
f) Methotrexate encephalopathy formula to have any effect.
g) Cytarabine encephalopathy Five days later the mother called an ambulance to the home
as the infant had a high-pitched scream and did not appear
Case 3
‘right’. She was seen in the emergency department where
Note: Each question needs to be considered with only the in-
some left eye deviation and right leg twitching was noted. She
formation given and without benefit of details further in the
underwent a CT scan of her head (see image).
history (no hindsight!). The management described is not
necessarily optimal and should not influence your responses.
A 7-week-old term infant was referred to paediatrics with a
2-day history of vomiting and crying. Her mother thinks she
appears in pain. She examines normally. Her weight is 5Kg
and she takes 180ml of standard formula every 4 hours,
including overnight. Her vomits and distress are usually, but
not always, post feeds.
Question 1
Appropriate advice at this point could include. Choose best
THREE options:
a) Start gaviscon
b) Start ranitidine
c) Start omeprazole
d) Trial of extensively hydrolysed formula
e) Trial of amino acid formula
f) Reduce feed volumes
g) Advice on positioning (head upright after feeds, tilt cot 30
degrees)
h) Encourage baby wearing (use of a sling) and/or holding

PAEDIATRICS AND CHILD HEALTH 29:10 456 Ó 2019 Elsevier Ltd. All rights reserved.
SELF-ASSESSMENT

Question 3 respiratory distress. If the haemoglobin is less than 5g/dl and/


Which of the following are most likely to be true? Choose TWO or signs of cardiac failure discuss with tertiary haematology/
a) The scan appearance could account for all her symptoms oncology service.
b) The bruise to the face noticed previously could have
happened in the way the mother described FURTHER READING
c) A CT scan taken at presentation would probably have been Clinical Review Acute leukaemia in children: diagnosis and
normal management. Mitchell, Hall & Clarke. BMJ 2009;338:b2285
d) Gastro-oesophageal reflux (GORD) is a common cause of doi:10.1136/bmj.b2285.
crying in the newborn period Case 2
e) Crying is the commonest trigger for physical abuse of an 1) B and F
infant Note that this patient is on high-dose steroids, which may
mask a fever. Be cautious when seeing any child during ALL
induction and delayed intensification. This is a time of high
Answers risk for neutropaenic sepsis as the treatment is intense and
Case 1 includes steroids. If in doubt and the child appears unwell,
1) B, C, D, F, G start antibiotics. Do not await the full blood count in a child on
2) D treatment for malignancy who is likely to be neutropaenic.
3) C, G, H Even if the neutrophil count is within normal limits, consider
The full blood count results show a very high white cell count line infection in child with central venous access and do not
with low haemoglobin and platelets in a child with unex- delay treatment. In neutropaenia the urine may not have any
plained bruises and petechiae. It would be unusual to get such leucocytes and young children may empty their bladders
a high white count in sepsis alone and the white coating on the frequently and not produce nitrites in infection. Send all urine
tongue is suggestive of oral candida, hence immunosuppres- samples to the laboratory in such patients. However do not
sion. This makes the most likely diagnosis leukaemia. ALL is allow the collection of a urine sample delay treatment, espe-
much more common than AML in children. cially if there are no symptoms or signs of urine infection. Do
A blood film may show ‘blasts’ (abnormal white cells) and not do ineout catheters or per rectum examinations in neu-
help confirm the diagnosis of leukaemia. There may be coa- tropaenic patients as you may introduce infection, especially
gulopathy associated with both ALL and sepsis, which may in the presence of mucositis, a common side effect of
need treating. Blood cultures should be taken due to the chemotherapy.
presence of fever and the possibility of sepsis complicating The patient may be anaemic and should be transfused if
ALL. The first line antibiotics in a febrile child with likely necessary. Most centres would transfuse if Haemoglobin is
leukaemia would be an antipseudomonal penicillin such as less than 8g/dl.
piperacillin/tazobactam, as per paediatric febrile neutropaenia 2) A, B, C, F
management. The high white count is likely to represent All children on treatment for ALL should receive co-
abnormal, non-functioning white cells and the child is likely to trimoxazole prophylaxis against Pneumocystis jirovecii pneu-
be functionally pancytopaenic. monia, PJP (previously known as Pneumocystis carinii, PCP).
A high white cell count and/or bulky disease in leukaemia Before the routine use of PJP prophylaxis the estimated PJP
are risk factors for tumour lysis syndrome (TLS), where fragile infection rate in children with ALL was 22e43% with a high
blast cells lyse in the circulation releasing uric acid, phosphate mortality rate.
and potassium. Features of TLS include raised urate, phos- Antiemetics are required for children on chemotherapy.
phate and potassium and low calcium and potential renal Some may need them even on days when they are not
failure secondary to hyperuricaemic nephropathy. TLS may be receiving treatment, especially when they are in an intensive
managed and prevented with regular allopurinol, hyper- block. Consider nausea as a differential or significant
hydration of between 2 and 3 litres/m2 per day and regular contributor to the symptoms and signs in this scenario.
monitoring (between 6 and 12 hourly depending on risk) of The child has a white coating to her tongue and is immu-
urea, electrolytes, phosphate, calcium and urate. Severe cases nosuppressed so is likely to have oral candida. A treatment
may require rasburicase (a urate oxidase enzyme that course of nystatin is appropriate but fungal prophylaxis is not
metabolises uric acid to allantoin). A chest x-ray should be usually given to children on standard treatment for ALL.
performed for evidence of bulky disease and/or airway Routine chlorhexidine mouth washes have not been shown to
compromise due to mediastinal mass. Do not add potassium to provide any evidence-based benefit.
fluids due to risk of hyperkalaemia. Children on treatment for ALL and many other malig-
Red cell transfusions should be avoided, especially if the nancies are at high risk from VZV, even if they have previ-
white cell count is greater than 50 x 109/l. Where there is ously had the disease. Most centres would advocate
already leucostasis (increased plasma viscosity due to high immunising all non-immune household contacts.
white counts) a transfusion may increase blood viscosity Children who are well but neutropaenic may attend
further precipitating stroke, haemorrhage and/or acute school/nursery. Different units do give conflicting advice

PAEDIATRICS AND CHILD HEALTH 29:10 457 Ó 2019 Elsevier Ltd. All rights reserved.
SELF-ASSESSMENT

regarding the management of exposure to infection during weeks before switching to an amino acid formula if the sus-
periods of neutropaenia but the risk of infection must be picion of CMPA is still high.
balanced against that of missing educational and social 2. A, B, C, G
opportunities. There is now weight loss, blood and mucus in the stool,
3) A, D, F more forceful vomiting and a bruise on the face. The differ-
This is most likely to be methotrexate encephalopathy. ential could include intussusception, pyloric stenosis, CMPA
Typical presentation is with focal neurological deficit and/or and non-accidental injury. A blood gas may help diagnose
rapid personality change and/or loss of consciousness occur- possible pyloric stenosis and an abdominal ultrasound may be
ring within 1e21 days (average 3 days) after exposure to useful for this and for intussusception. Blood and mucus in the
intrathecal methotrexate. Full recovery is usual within 48 stool might be indicative of CMPA but this infant has only
hours. Intrathecal methotrexate is also associated with aseptic been taking eHF for 2 days so this is not a sufficient trial. The
meningitis but the clinical picture here is more of encepha- mother’s explanation for the bruise is not sufficient. Bruising
lopathy. The differential should include both bacterial and in non-mobile infants is suspicious, especially to the face so
viral encephalitis and the child should be treated with both investigations for non-accidental injury should be undertaken.
antibiotics and antivirals acutely but with this LP result a viral 3. A, E
cause is more likely. Central nervous system relapse is The CT scan shows bilateral subdural collections and
possible on treatment and may present with encephalopathy. multiple skull fractures (only 1 visible on this image) and was
reported has having both new and old injuries. A CT scan
FURTHER READING taken at presentation would probably have shown evidence of
Children’s Cancer and Leukaemia Group: Side Effects and head injury and is the likely cause of the vomiting and crying
Supportive Care information https://www.cclg.org.uk/CSOIR/ from the outset. In paediatrics and general practice we see
Section-7-Side-effects-and-supportive-care. many infants with vomiting and troublesome crying. Most of
UKALL 2011 Trial protocol. these have physiological reflux and/or ‘normal’ crying and an
Case 3 underlying medical problem is unlikely. GORD, lactose intol-
1. F, G, H erance and cow’s milk allergy are often blamed for trouble-
This infant is overfed. A rough guide to feed volumes in bottle some crying but are unlikely causes. Those working in child
fed babies is around 150ml/kg/day and this infant is taking health must be alert to the possibility of non-accidental injury
216ml/kg/day. The first recommendation in the NICE guide- and the wilful withholding of information by caregivers.
line for Gastro-oesophageal Reflux (GOR) in children and Crying is the commonest trigger for physical abuse of an infant
young people is to check for overfeeding and reduce feed and non-accidental injury is most prevalent in the youngest
volumes accordingly. Following that, the guideline advises age groups.
offering smaller more frequent feeds, then a trial of thickened
FURTHER READING
formula. If those measures are not successful, try gaviscon.
Fifteen-minute consultation: troublesome crying in infancy.
Acid suppression is no better than placebo. Positioning may
McKenzie SA, Arch Dis Child Educ Pract Ed. 2013
help reduce reflux and vomiting and carrying can reduce
Dec;98:209-211.
troublesome crying.
iMap Guidelines, Allergy UK https://www.allergyuk.org/health-
There is no particular indication here for cow’s milk pro-
professionals/imap-guidelines.
tein allergy (CMPA). However if there were concerns about
NICE guideline https://www.nice.org.uk/guidance/ng1/chapter/1-
this (strong family history, eczema, other allergies, blood in
ecommendations#initial-management-of-gor-and-gord.
stool) a trial of an extensively hydrolysed formula (eHF)
RACP Paediatrics and Child Health Division Top 5 low-value
would be the first choice. Approximately 80% of infants with
practices and interventions https://evolve.edu.au/published-
CMPA will respond to eHF and this should be given for a few
lists/paediatrics-and-child-health-division.

PAEDIATRICS AND CHILD HEALTH 29:10 458 Ó 2019 Elsevier Ltd. All rights reserved.
SELF-ASSESSMENT

Self-assessment
Questions C It is most common subtype
D It affects T-cells
Case 1
E It is more frequent in children than in adults
A 4-year old girl was seen in the paediatric outpatient
The girl was started on a combined-chemotherapy
department because of being unwell and tired for over a
EsPhALL þ Imatinib and supportive medicine.
month. On examination she had bilaterally swollen lymph
One month later the girl was readmitted, due to side effects
nodes in the cervical region approximately 2.5 cm in size.
of the therapeutic regime In that stage of therapy the protocol
These nodes were fixed to the underlying tissue and dis not
included Cyclophosphamide, 6-Mercaptopurine, Cytosine
hurt on palpation. She had remained afebrile with no concerns
Arabinoside, intratechal Methotrexate and Imatinib.
regarding her growth and weight gain. The mother reported
that she had noticed small spots on her child’s chest and that Question 4
she was bruising more easily. Previous attendances at the GP What are the THREE most likely side effects of chemotherapy?
were for vomiting and recurrent ear infections. A Vomiting and diarrhoea
B Weight loss
Question 1
C Fever
Which of the symptoms are considered as “red flags” in
D Weight gain
lymphadenopathy? Choose FIVE.
E Kidney failure
A Localised swollen lymph nodes in the groin
B Temperatures Case 2
C Weight loss >10% A 15-year-old girl presented with left-sided abdominal pain,
D Fixation to the underlying structure which was described as suddenly having woken her up,
E No pain without any vomiting or diarrhoea. She is normally fit and
F Tiredness healthy, without need for regular medication. She had been on
G Size >2 cm a hiking trip the previous weekend.
H Progressive enlargement She presented with a heart rate of 84, a temperature of 37.3
The results of her full blood count showed: Haemoglobin 
C, a respiratory rate of 22 and oxygen saturations of 100%.
68 mg/dL, WCC 45.8 x 109/L, Platelets 11 x 109/L and Her abdomen was soft with severe pain over the left renal
abnormal white blood cells on film. angle (pain Score 7/10). The pain worsened when she was
moving, especially in hip flexion and internal rotation. The
Question 2
patient was able to sit up in bed. A pregnancy test was
What would be the next most important steps? Select TWO.
negative, the urine dip showed 3þ blood and the blood test
A Referral to oncology
showed no increased inflammation markers.
B Bone marrow aspiration
C Immune phenotyping Question 1
D Blood transfusion Which diagnosis would you like to rule out first? Select TWO:
E Platelet transfusion A Urinary tract infection
F MRI scan B Basal pneumonia
She was referred to the paediatric oncology department C Acute appendicitis
where the immune-phenotyping and microscopy of the pe- D ovarian torsion
ripheral blood confirmed the diagnosis of a Philadelphia E nephrolithiasis
chromosome positive acute lymphoblastic leukaemia (ALL).
Question 2
Question 3 Which investigation would you request next? Choose ONE:
What distinguishes Philadelphia chromosome positive ALL A Ultrasound abdomen
from other subtypes? Choose ONE: B X-ray
A It has a better prognosis C CT abdomen with enhancement
BA target therapy exists
D CT abdomen without enhancement
E Urine sediment
Ultrasound imaging showed the left kidney knee appearing
Nina Annika Herz, Medical Student, University of Cambridge, duplex with a dilated pelvicalyceal system of the lower moiety.
School of Clinical Medicine, Addenbrookes Hospital, Cambridge, No ureteric dilatation was seen and the ovaries appeared
UK. normal. The girl passed urine overnight but the pain persisted.
Peter Heinz MD State Exam Med FRCPCH, Consultant Paediatrician, When further exploring the history it transpired that her
Addenbrookes Hospital, Cambridge, UK. mother had kidney stones in the past.

PAEDIATRICS AND CHILD HEALTH 29:12 547 Ó 2019 Elsevier Ltd. All rights reserved.
SELF-ASSESSMENT

Question 3 Question 2
Was is the next investigational step? Choose ONE: What is the most important next step in her management?
A Another ultrasound abdomen Choose TWO:
B X-ray A Hip X-ray (AP and frog leg/lateral)
C CT abdomen with enhancement B Bloods to check inflammatory markers
D CT abdomen without enhancement C MRI scan of the pelvis/hips
E Urine oxalate D Referral to the on-call orthopaedic team
CT imaging (Figure 1) showed a moderate hydronephrosis E Symptomatic management (rest, analgesia, elevation,
of the left kidney (AP diameter renal pelvis 20 mm) with an crutches)
obstructing 4 mm calculus in the mid-ureter. An X-ray of the hips was requested and the changes are
best seen on the AP view (Figure 2).
Question 4
What is the next most appropriate step in her management? Question 3
Choose ONE: What abnormality does this X-ray show? Choose ONE:
A Pain relief, drinking and moving A Avascular necrosis of the femoral head (Perthe’s disease)
B pain relief, drinking, moving and antibiotics B A right sided joint effusion
C Extracorporeal shock wave lithotripsy (ESWL) C A pathological fracture
D Urine drainage via direct nephrostomy D Osteopenia
E Laparoscopic ureterolithotomy E Slipped upper femoral epiphysis
Blood test results showed elevated free T3 and T4 and
Case 3
reduced TSH confirming a diagnosis of thyrotoxicosis and
A 10-year-old girl presented with acute onset of right hip and
resultant hypocalcaemia and hyperphosphatemia.
wrist pain following a very minor trip and fall onto the ground.
Since the incident she had been unable to weight bear. On Question 4
examination, the right hip and wrist showed no obvious What is the most important next management step in relation
deformity and range of movement was normal once her pain to the endocrine abnormalities?
had been addressed. During the last two years she has not A Lifelong thyreostatic therapy
presented to the GP. Other, more recent health issues the child B Thyroidectomy
mentioned included poor sleep, leading to daytime somno- C Radioiodine therapy
lence, palpitations, anxiety and sweatiness. Plotting her on the D 12e18 month thyreostatic therapy þ symptomatic therapy
growth charts, a fall in her weight growing curve was noted. E Vitamin D and calcium supplementation
Question 1
In children of school age (6e11 years) presenting with an Answers
atraumatic limp, what is the most common diagnosis? Choose
Case 1
ONE:
1) B-D, G, H
A Slipped upper femoral epiphysis
2) A, B
B Perthes’ Disease
3) B
C Juvenile idiopathic arthritis
4) A-C
D Septic arthritis/osteomyelitis
Swelling of lymph nodes is a common finding in children and
E Transient synovitis
is in approximately 98.9% of the cases benign and self-limited
F Benign joint hypermobility

Figure 1 Figure 2

PAEDIATRICS AND CHILD HEALTH 29:12 548 Ó 2019 Elsevier Ltd. All rights reserved.
SELF-ASSESSMENT

but can also be caused by malignancy or infection. Lymph Besides the complex individual chemotherapy protocols,
nodes > 2cm, no matter whether generalised or localised, there are common side effects, including vomiting, diarrhoea
require further investigation. The history can give essential and neutropenic fever. In our case antiemetic, change in diet
information, helping to assess the risk for a more sinister and antibiotic coverage led to improvement in her symptoms
underlying diagnosis and determine further diagnostic steps. and a quick recovery. Other common problems encountered
“Red flags” such as fever, lethargy, anorexia and sweating are abnormal bleeding, fatigue, rashes and bone pain.
may be indicative of malignancy along with other features
listed in Table 1.
In the case of this patient a number of red flags are present FURTHER READING
and in the first instance a full blood count with film is the most https://www.paediatricsandchildhealthjournal.co.uk/article/
appropriate test. The abnormal results and the clinical pre- S1751-7222(15)00225-5/pdf
sentation make leukaemia a likely diagnosis. https://www.lls.org/leukemia/acute-lymphoblastic-leukemia/
All of the suggested investigations are reasonable. How- treatment/ph-positive-all-therapy
ever, the next investigational step, usually performed by the Case 2
paediatric oncology team is the bone marrow testing, either by 1) E
aspiration or biopsy. This is essential for diagnosing malig- 2) A
nancy, however it is an invasive procedure and requires clear 3) D
indication and should not be the first investigational step. 4) A
Following on from this, molecular testing and diagnostic Recent publications suggest, that nephrolithiasis is an
categorization are undertaken to determine the type of leu- increasing problem in children and adolescents. The formation
kemia with a view to tailoring the treatment. Furthermore, of stones is multifactorial, including environmental and ge-
imaging (CT or MRI) can help to detect tumor cells growing netic factors. Our patient presented with flank pain and a
outside of the bone marrow, for instance in lymph nodes and family history of kidney stones, which made nephrolithiasis
spinal cord (see Figure 3). very likely.
Leukaemia and lymphoma are not frequently seen in the A urine-dipstick, testing for haematuria, is a helpful tool for
paediatric population. However, they account for approxi- symptomatic patients. However, a positive result can be due to
mately 40% of cancer in people <20 years. In this age group non-pathologic reasons (e.g. exercise or menstruation) and
ALL has the highest prevalence among all types of cancer. has a high false-positive rate. Moreover, a positive dipstick
Philadelphia chromosome positive ALL is a rare subtype of itself is not sufficient for the diagnosis of haematuria and
ALL, accounting for 25% of ALL in adults and only 3% in should be confirmed by microscopy (see Table 2).
children. The pathogenesis (a translocation of genes between Flank pain in adolescence can be due to several different
chromosome 9 and 22, producing the BCR-ABL fusion gene, pathologies, including amongst many others musculoskeletal,
coding for a permanently activated tyrosine kinase, leading to gynaecological and intestinal abnormalities. Important differ-
uncontrollable cell cycle), allows for a target therapy with ential diagnoses for the acute abdomen in adolescence are
tyrosine-kinase-inhibitors such as Imatinib. This medication is acute appendicitis and ovarian torsion, these are emergencies
more commonly used in CML, where the same pathologic and require immediate therapy (see Table 2). When these
mutation can be found. emergencies have been ruled out or seem unlikely, if there is
an absence of infection markers and normal abdominal so-
nography, other causes have to be taken into account. There is
Red flags e lymphadenopathy a high prevalence of urinary tract infections in teenage girls,
however, with a negative urine-dip stick and without pain and
1. Size >2 cm urinary symptoms it is in this case an unlikely diagnosis. The
2. Fixed to underlying structure location of the pain and the sudden onset during the night are
3. Progressive enlargement typical for renal colic. In children, nephrolithiasis can often
4. Supraclavicular nodes present with uncharacteristic symptoms.
5. Drenching night sweats The finding of the abdominal ultrasound made other dif-
6. Persistent fiver ferential diagnoses, such as ovarian torsion and appendicitis
7. Abnormal X-ray unlikely. Ultrasound could have detected the kidney stone, but
8. Weight loss >10% was not successful in this case. However, a dilated pelvicaly-
ceal system was noted; its aetiology can be due to several
different reasons, including stones but can also be a non-
Table 1
pathologic variation.

Figure 3

PAEDIATRICS AND CHILD HEALTH 29:12 549 Ó 2019 Elsevier Ltd. All rights reserved.
SELF-ASSESSMENT

Red flags e acute abdominal pain in children Red flags e atraumatic hip pain

1. Septic appearance (fever, tachycardia, generally unwell) 1. Fever


2. Respiratory symptoms (tachypnoea, respiratory distress, cough) 2. Malaise
3. Generalized oedema: suspect nephrotic syndrome 3. Morning joint stiffness or pain
4. Significant dehydration 4. Nocturnal pain refractory to simple analgesia & symptomatic during
5. Purpuric rash day
6. Jaundice 5. Joint swelling
7. Polyuria/polydipsia 6. Bony tenderness
8. Peritoneal pain 7. Muscle weakness
9. Feculent vomits 8. Fall in height or weight growing curve
10. History of recent significant abdominal trauma/surgery
11. Irreducible hernia
Table 3
12. Testicular torsion

Table 2
site, onset, character, radiation, association (Weight-bearing,
Nephrolithiasis can be detected by CT, X-ray (if calcium con- mono-vs polyarticular, systemic effect, dermatology), time
taining stones) and ultrasound but repeated ultrasound is course, exacerbating and relieving factors and severity. Addi-
unlikely to lead to the diagnosis. Our patient presented with tionally, a FAST-review (family history, adolescence, safe-
severe pain and in order to have a more detailed picture of guarding and travel history) might be consider. The next steps
potential severe intraabdominal pathologies a CT scan was are an examination including inspection, palpation, assessment
chosen over X-ray. As contrast enhancement has potential side of movement and a systemic examination.
effects, a clear indication is needed as in this case contrast Even though, cooling, rest and pain-relieve are an adequate
enhancement could have masked calcifications. therapy for transient synovitis, this girl presented with some
Decision on the acute management is based on pain, the clinical features that required further investigation, namely
degree of urinary obstruction and the presence of infection. her inability to weight-bear and the disproportionate severity
Due to fewer side effects a conservative approach should be of her pain.
chosen over interventional therapy. In this case, pain-relieve For children >8 years or limping >7 days an X-ray is an
and support of spontaneous stone passage, such as drinking adequate imaging modality as it has a relatively low dose of
and moving, was sufficient. However, sometimes an inter- radiation but is also very likely to show the pathology causing
ventional approach might be required (e.g. big stones, the pain. Additionally, blood tests including full blood count,
obstruction, infection). ESR and CRP are indicated. The indications for CT scans are
relatively limited in this context and may only be used for
further investigations of bony lesions detected on a plain film
FURTHER READING like for example an osteoid osteoma.
https://www.rcem.ac.uk/docs/Paediatric%20EM%20Guidance/ In our case the pathologic fracture was due to endocrine
CEM7187-Abdo-Pain-Pathway—Luton.pdf abnormalities. However, there are several other reasons, such
https://pedsinreview.aappublications.org/content/25/4/131 as chronical illness and genetic disorders, that can also lead to
https://www.ncbi.nlm.nih.gov/pmc/articles/PMC3126087/ pathologic fractures.
Hyperthyroidism is a rare but severe disease in children,
Case 3
with several possible causes, mostly autoimmune.
1) E
Usually the therapy consists of thyreostatic medication
2) A, B
(e.g. carbimazol) and symptomatic therapy (e.g. propanolol),
3) B
taken for 12e18 month in conjunction with thyroid replace-
4) D
ment therapy. In cases of relapse, an interventional resection
The most common reason for hip pain in children (3e10
or radioactive iodine therapy might be required. Additionally,
years) is a transient synovitis. Even though this being a benign
this patient would benefit from ophthalmologic, cardiologic
cause without requirement for treatment, other more severe
and dietarian follow-up.
differential diagnosis (e.g. septic arthritis/osteomyelitis,
Perthes Disease, slipped capital femoral epiphysis, neoplasm
and fracture) have to be considered (see Table 3). FURTHER READING
A ‘red herring’ injury is commonly reported by patients and https://patient.info/doctor/painful-hips-in-children
their parents but in this case the minor fall was in no pro- https://www.paediatricsandchildhealthjournal.co.uk/article/
portion to the severity of the symptoms. S1751-7222(17)30288-3/fulltext#sec3
For atraumatic joint pain in children the “SOCRATES” mne- https://www.ncbi.nlm.nih.gov/pmc/articles/PMC3608005/
monic is suggested for taking the anamnestic history. Asking for:

PAEDIATRICS AND CHILD HEALTH 29:12 550 Ó 2019 Elsevier Ltd. All rights reserved.
SELF-ASSESSMENT

Self-assessment

Questions tolerate this and continued to vomit daily. Her headaches


worsened and she developed blurred vision. She complained
Case 1
of feeling thirsty and was found to be getting up often in the
A 14-year-old girl was admitted with a 3 month history of
night to pass urine. Her weight after 1 month of admission on
reduced appetite, vomiting and weight loss. She complained of
the paediatric ward was 39.7kg (0-25th centile).
being generally tired and had occasional headaches, but was
still able to go to school and participate in sports. There was Q1
no history of double vision. She had a past medical history of Which of the following investigations would be helpful in
anxiety for which she was receiving counselling at school but establishing the diagnosis? Please select THREE from the list:
had no other comorbidities. There was no relevant family a) Capillary blood gas
history and she was not taking any medications. She denied b) Paired urine and serum osmolarity
recreational drug use. On examination, her observations are c) Thyroid function tests
within normal limits. Her weight is 42kg (0-25th centile) and d) Liver function tests
her height is 172cm (50th-75th centile). Cardiovascular, respi- e) ESR
ratory and abdominal examinations are normal. On neuro- f) Prolactin
logical assessment, she is generally weak and unsteady on her g) Anti dsDNA
feet, but has no focal neurological findings. Preliminary in- h) LDH
vestigations are as follows: i) Throat swab
Haemoglobin 146 g/L
Q2
White blood cell count 7.4 x 109/L
Which radiological investigation would confirm the diagnosis?
Lymphocytes 2.3 x 109/L
Please select ONE:
Neutrophils 4.6 x 109/L
a) Abdominal ultrasound scan including kidneys, ureters and
Platelets 367 x 109/L
bladder
Sodium 145 mmol /L
b) Upper gastrointestinal endoscopy
Potassium 3.6 mmol /L
c) Chest X-ray
Urea 2.9 mmol /L
d) MRI head
Creatinine 37 mmol /L
e) None of the above
CRP <2mg/L
Urine dipstick: negative for leucocytes, nitrites, blood, Q3
protein and glucose. Specific gravity <1.005 What is the most likely diagnosis? Please select ONE:
Abdominal ultrasound scan: Normal. a) Bulimia nervosa
On the paediatric ward, a naso-gastric tube was passed and b) Pituitary tumour
she was started on continuous feeds, but was unable to c) Severe gastro-oesophageal reflux disease
d) Medulloblastoma
e) Gastroparesis
Case 2
Sophie Constantinou BSc MBBS MRCPCH ST2 Paediatrics, Uni- A 4 year old boy presents with a 3 week history of general
versity Hospital of Wales, Noah’s Arc Children’s Hospital, Cardiff, lethargy, coryza and cough. He has no significant past medical
UK. Conflict of interest: none declared. history and no known unwell contacts. On examination, he is
alert and playful, but pale. His heart rate is 130 beats per minute
Madeleine Adams MBBCh MRCPCH MD Consultant Paediatric and his temperature is 38.4 C. His respiratory rate, blood
Oncologist, University Hospital of Wales, Noah’s Arc Children’s pressure and oxygen saturations are within normal limits. On
Hospital, Cardiff, UK. Conflict of interest: none declared. cardiovascular examination he has a soft systolic murmur with
a good volume pulse. His chest is clear on auscultation. On
Indu Thakur MBBS MD Paediatrics DNB Paediatrics MRCPCH FRCPath abdominal palpation he has a 2cm liver edge and a palpable
Consultant Paediatric Haematologist, University Hospital of Wales, spleen. He has small cervical lymph nodes. On ENT examina-
Noah’s Arc Children’s Hospital, Cardiff, UK. Conflict of interest: tion, he has large erythematous tonsils with no exudate and his
none declared. ears are waxy bilaterally. He has a petechial rash on his lower
limbs. Preliminary investigations are as follows:
Philip Connor BSc MRCPCH FRCPath Consultant in Paediatric Haemoglobin 44 g/L
Haematology, University Hospital of Wales, Noah’s Arc Children’s White blood cell count 20.3 x 109/L
Hospital, Cardiff, UK. Conflict of interest: none declared. Lymphocytes 15.3 x 109/L

PAEDIATRICS AND CHILD HEALTH 30:1 43 Ó 2019 Elsevier Ltd. All rights reserved.
SELF-ASSESSMENT

Neutrophils 0.4 x 109/L micrograms INH BD, salbutamol 200 micrograms INH PRN.
Monocytes 2.6 x109/L She has no known drug allergies. She lives at home with her
Platelets 17 x 109/L mother, father and sister, none of whom have sickle cell
Sodium 141 mmol /L anaemia. Her sister has autism spectrum disorder. The patient
Potassium 4.9 mmol /L was born in the UK and has received all her vaccinations,
Urea 2.9 mmol /L apart from the MMR vaccine. You note from a clinic letter
Creatinine 37 mmol /L dated 1 month prior that her haemoglobin was 82 g/L. On
CRP 58 mg/L arrival in the children’s assessment unit, her initial observa-
tions are as follows: Temperature 38.2 C, HR 95, BP 110/67,
Q1
RR 24, Sats 94% in air. On examination, she is obviously in
What is the most likely underlying diagnosis? Please select
pain and gives a score of 8/10 for her abdominal pain. She is
ONE:
slightly sweaty, and her palms and conjunctiva appear pale.
a) Meningococcal septicaemia
She has a slightly increased effort of breathing and you think
b) Lymphoma
you can hear scattered wheeze throughout her chest. She has a
c) Idiopathic Thrombocytopenic Purpura
good volume pulse and warm peripheries. Her central capil-
d) Acute Lymphoblastic Leukaemia
lary refill time is 2 seconds. Her heart sounds are normal but
e) Viral tonsillitis
she has a soft systolic murmur heard throughout the precor-
Q2 dium. Her abdomen is not distended, and although she does
Which of the following investigations would help confirm the not have any guarding, complains of diffuse abdominal pain
underlying diagnosis? Please select THREE from the list: on palpation. You cannot feel an enlarged spleen. Neurological
a) Urine dipstick and ENT examinations are unremarkable.
b) Lumbar Puncture
Q1
c) Blood film
What is the next most important step in management? Please
d) Abdominal ultrasound scan
select ONE:
e) Bone marrow aspirate and trephine
a) Insert a nasopharyngeal airway and give supplementary
f) Blood glucose
oxygen
g) CT head
b) Sit up, give supplementary oxygen, give salbutamol and
h) Viral throat swab
ipratropium nebulisers, administer analgesia.
Q3 c) Insert an IV cannula, take blood for FBC and group and
What are the next most important steps in the management of save. Ask the laboratory for a cross match for 2 units of
this patient? Please select ONE: packed red blood cells, administer analgesia.
a) Insert an IV cannula, take bloods for microscopy, culture d) Insert an IV cannula, give a 20ml / kg fluid bolus, take
and sensitivities, take a viral throat swab, give broad blood for a blood culture, administer broad spectrum an-
spectrum antibiotics and give an urgent blood transfusion tibiotics IV.
over 30 min. e) Insert an IV cannula, give 2 units of O Rh d negative blood
b) Insert an IV cannula, take bloods for microscopy, culture STAT.
and sensitivities, take a viral throat swab and give a dose of
Q2
paracetamol.
Which of the following investigations are most important in
c) Insert an IV cannula, take bloods for microscopy, culture
helping to confirm the diagnosis? Please select THREE from
and sensitivities, take a viral throat swab, give broad
the list:
spectrum antibiotics, give a blood transfusion over 3 h and
a) FBC & blood film
discuss with the paediatric tertiary centre.
b) Blood glucose
d) Insert an IV cannula, take bloods for microscopy, culture
c) Urine dipstick
and sensitivities, take a viral throat swab and give broad
d) Abdominal ultrasound scan
spectrum antibiotics
e) Capillary blood gas
e) Insert an IV cannula, take bloods for microscopy, culture
f) Reticulocyte count
and sensitivities, take a viral throat swab, give broad
g) CT head
spectrum antibiotics, give a blood transfusion over 3 h and
h) Abdominal X-ray
discuss with microbiology.
i) Chest X-ray
Case 3
Q3
A 14 year old girl presents with a headache, general malaise
The patient’s initial blood results are as follows:
and abdominal pain. She has no known unwell contacts. She
Haemoglobin 62 g/L
has a background of sickle cell anaemia and asthma. Her drug
White blood cell count 6.7 x 109/L
history includes folic acid 5mg OD, phenoxymethylpenicillin
Lymphocytes 0.6 x 109/L
250mg BD, hydroxycarbamide 400mg OD, paracetamol 500mg
Neutrophils 5.5 x 109/L
QDS PRN, diclofenac 25mg TDS PRN, beclomethasone 200

PAEDIATRICS AND CHILD HEALTH 30:1 44 Ó 2019 Elsevier Ltd. All rights reserved.
SELF-ASSESSMENT

Monocytes 0.6 x109/L 3) C


Platelets 495 x109/L This child is presenting with a probable tonsillar infection or a
Reticulocyte count 24 x109/L chest infection, but his underlying diagnosis is of Acute
LDH 240 U/L Lymphoblastic Leukaemia (ALL). ALL is a malignant clonal
CRP 13 mg/L expansion of early lymphoid precursor cells (lymphoblasts) in
What is the most likely underlying causative organism? the bone marrow, which subsequently replaces normal
Please select ONE from the list: marrow elements therefore resulting in a decrease of cells in
a) Streptococcus pneumoniae other blood lines leading to anaemia, thrombocytopenia and
b) Kingella kingae neutropaenia. Pointers in the history may be non-specific,
c) Parvovirus B19 such as lethargy or myalgia, or there may be a history of
d) Salmonella Typhi repeated infections. The pallor, systolic murmur and petechial
e) Mumps virus rash should point you towards ordering an FBC to check for
the presence of anaemia or thrombocytopenia, and with a
haemoglobin count as low as 44g/L a blood film should be
Answers
additionally requested. The raised total white cell count in
Case 1 conjunction with the low neutrophil count should point you
1) B, C and F towards the diagnosis of ALL. Blast cells may not be provided
2) D as part of the initial FBC, as automated machines may pick out
3) B blasts as a mixture of monocytes and lymphocytes, but soft-
This patient was found to have a rare germ cell tumour ware in the instruments usually detects blasts and flags up
compressing the pituitary gland. Tumours within the pituitary their presence to laboratory staff. A blood film will later
fossa are rare in children and adolescents, however can cause confirm the presence of blast cells in the peripheral blood.
significant morbidity due to compression of the normal pitui- Bone marrow aspirate and trephine, in addition to a lumbar
tary gland. This can lead to hormonal deficiencies, as well as puncture to check for central nervous system (CNS) involve-
cause symptoms through mass effect on surrounding tissues ment, confirms the diagnosis of ALL. Bone marrow samples
and the brain. This patient presented initially with non-specific are additionally tested for specific frequently occurring genetic
signs and symptoms of raised intracranial pressure, such as abnormalities, such as hyperdiploidy (good risk) or for fusion
vomiting and intermittent headaches, only developing double genes like the Philadelphia chromosome (bad risk), which
vision at a late stage. In addition, she complained of symptoms helps to provide information for risk stratification and treat-
of central diabetes insipidus, such as polydipsia and polyuria, ment planning. Philadelphia chromosome positivity is rare in
as well as other non-specific symptoms of pituitary dysfunc- children (3e5%) with historical data giving a poor outcome,
tion such as feeling generally weak. In cases like these, however the use of tyrosine kinase inhibitors like imatinib
sending a hormonal profile may reveal a raised prolactin and have dramatically improved treatment outcome in recent
hypothyroidism if the pituirary gland is involved. Sending a years.
paired urine and serum sodium and osmolarity, in addition to This patient would have required a platelet transfusion
a history of polyuria and polydipsia, will help to establish the prior to a lumbar puncture being performed. An intrathecal
diagnosis of central diabetes insipidus. Treatment with dose of methotrexate would usually be administered during
thyroxine, DDAVP and steroids will generally need to be the procedure to prevent any accidental CNS inoculation. Bone
instigated to replace the functions of the pituitary gland under marrow aspirates or intraosseous needle do not require
specialist paediatric endocrinology guidance. platelet transfusion and can be performed with a platelet count
This patient’s case was complicated by a past history of of 0 as long as the patient is not on any anti-coagulants. In
counselling for anxiety at school, which may have led you to addition to undertaking a partial septic screen and starting
consider the differential diagnosis of an eating disorder such broad spectrum antibiotics, he would also require a blood
as bulimia nervosa. However, the inability to tolerate contin- transfusion. Blood transfusions should not be administered
uous nasogastric feeds in addition to a prolonged history of too quickly as this may precipitate circulatory overload. After
significant weight loss are red flags which do not fit with a stabilisation, these patients need transfer to a tertiary centre
psychiatric cause, and therefore require urgent further inves- for specialist paediatric oncology care.
tigation with a brain scan. The best imaging modality for the
Case 3
diagnosis of any brain tumour is an MRI scan of the brain, as
1) B
small tumours may not be visible on a CT. The ‘HeadSmart
2) A, F and I
Guidelines’ (https://www.headsmart.org.uk/) include a ‘De-
3) C
cision Support Tool’ which can help guide decision making
This patient with sickle cell disease (SCD) presented with
surrounding imaging and onward referral.
signs and symptoms of acute bronchospasm, likely underlying
Case 2 infection and possible vaso-occlusive crisis of the abdomen.
1) D There may also be an element of heart failure caused by the
2) B, C and E drop in her haemoglobin.

PAEDIATRICS AND CHILD HEALTH 30:1 45 Ó 2019 Elsevier Ltd. All rights reserved.
SELF-ASSESSMENT

When faced with a wide variety of signs and symptoms, it penicillin should be held on admission and, after taking a
is important to address each problem systematically, priori- blood culture, broad spectrum antibiotics should be urgently
tising airway over breathing over circulation etc. This patient administered intravenously. A teenager with sickle cell
had signs of increased effort of breathing and her oxygen anaemia would usually be functionally asplenic and therefore
saturations were suboptimal in air, and therefore required at risk of overwhelming infection with encapsulated organ-
urgent treatment with oxygen and inhaled bronchodilators. isms. Parents should be encouraged to take up all routine
Reactive airways disease is common in children with sickle childhood vaccinations, particularly those against strepto-
cell disease as free haemoglobin and arginase released from coccus pneumoniae and haemophilus influenza B. Although
sickled red causes lowered levels of nitric oxide. this patient probably does not require a fluid bolus, she should
It would also be important to treat her pain at this point, as be started on IV fluids to improve blood viscosity and as part
she may be developing acute chest crisis (ACC). A chest X-ray of treatment for possible sepsis, usually at 100% of her
should be considered to look for radiographic evidence of ACC maintenance fluid volume.
- this could be any form of infiltrate or shadow in a febrile Infection with parvovirus B19 can cause profound anaemia
child with SCD which may be difficult (and unnecessary) to in patients with sickle cell disease. If the child has a sibling
distinguish from an underlying chest infection. In some cases, with SCD, they may pass on the virus to them. The incubation
the CXR may look normal early on. All patients admitted with period is 4e14 days. The virus infects red blood cell pro-
a sickle cell crisis should be encouraged to perform incentive genitors in the bone marrow, temporarily halting red cell
spirometry (with hourly use of the device) to prevent devel- production. In SCD, red blood cells have a markedly reduced
opment or worsening of an acute chest crisis. half-life and a patient relies on increased bone marrow pro-
Following stabilisation of the patient’s respiratory system, duction to maintain their Hb. Infection with parvovirus B19
and ideally in tandem, the patient’s circulatory system should can produce a rapid drop in Hb in patients with SCD. This
be swiftly assessed. Pallor, tachycardia and a systolic murmur patient experienced a drop in her Hb of 20g/L from her
indicates the requirement for an urgent FBC to check her Hb. A baseline, and therefore will require a blood transfusion.
blood group, save and cross match should be ordered at pre- Confirmatory evidence for an aplastic crisis can be sought by
sentation, as many laboratories will require an extended cross- ordering a reticulocyte count, which will be inappropriately
match (ABO, Rh Cc/Dd/Ee and Kell matching, together with low for the level of anaemia. Her hydroxycarbamide should be
HbS negative units in England) prior to issuing blood for held whenever there is evidence of intercurrent infection or
transfusion for patients with SCD. The patient’s prophylactic bone marrows suppression as it is by nature cytotoxic.

PAEDIATRICS AND CHILD HEALTH 30:1 46 Ó 2019 Elsevier Ltd. All rights reserved.
SELF-ASSESSMENT

Self-assessment
Questions He is started on intravenous antibiotics in accordance with
local guidelines for a urinary tract infection.
Case 1
Urine and blood cultures subsequently grow Klebsiella
A 15-month-old boy is brought to hospital due to a rigor on a
pneumoniae
background of 2 days of fever up to 39 degrees Celsius. During
An ultrasound shows bilaterally small kidneys and a
this time, he has been indicating abdominal pain and
thickened trabeculated bladder.
discomfort on urination. His appetite has been reduced but he
2) What is the most likely diagnosis?
has been drinking well.
a) Urethral stricture
He was born in India and his mother had a single antenatal
b) Post streptococcal glomerulonephritis
scan at 12 weeks which was normal. The family moved to the
c) Focal segmental glomerulonephritis
UK when he was 6 months old. His parents have been con-
d) Posterior urethral valves
cerned about his appetite and growth but his development is
e) Goodpastures syndrome
normal. There is a family history of type 2 diabetes.
His past medical history includes 1 admission for bron- 3) What is the most appropriate next investigation to confirm
chiolitis and 2 independent sets of antibiotic treatments by the the diagnosis?
GP for dipstick positive lower urinary tract infections. Apart a) Dimercaptosuccinic Acid (DMSA) Scan
from this he is well and takes no regular medication. b) Renal biopsy
On examination, he has cool peripheries and dry lips. His c) Cystoscopy
right flank is tender and his bladder palpable. d) Renal MRI
Observations: e) Micturating Cystourethrogram (MCUG)
Heart Rate 152/minute
Respiratory Rate 47/minute Case 2
Oxygen Saturations 96% in air A previously healthy five-year-old boy presents with increased
Blood Pressure 115/70 mm Hg tripping and difficulties climbing stairs over the last 6 weeks.
About six months ago his parents noticed that he was walking
Investigations: on his tiptoes. Recently he has been playing indoors more
rather than joining his two older brothers playing football
outside.
Hb 73 g/dL
He has no health problems and takes no regular medica-
WCC 19.7 x 109/L
tions. There is a family history of atopy. He lives with his two
Platelets 503 109/L
brothers and both parents. His parents are non-
Naþ 143 mmol/L
consanguineous. He is in year 1 at school but there are
Kþ 6.6 mmol/L
some concerns about learning difficulties.
Urea 17.1mmol/L
He sat independently at 8 months, pulled to stand at 15
Creatinine 342 mmol/L
months and walked at 21 months. He said his first words at 18
CRP 245 mg/L
months and put words together at 3 years. His fine motor and
Urine Dip Leucocytes
social development have been normal.
þþþ
On general examination he appears well. There are no
Nitrites þþ
dysmorphic features and he is well grown. Cardiovascular,
Blood þþ
respiratory and abdominal examinations are normal.
He has a broad-based gait and large calves. His tone is
1) What is the most likely organism to cause a urinary tract normal in all 4 limbs. Power and reflexes are normal in his
infection in this child? upper limb. Patella tendon reflexes are present. His power on
a) Enterococcus faecalis hip flexion and knee flexion is 4/5 and hip and knee extension
b) Escherichia coli is 3/5 bilaterally.
c) Pseudomonas aeruginosa 1) What is the most likely diagnosis?
d) Proteus mirabilis a) Becker muscular dystrophy
e) Staphylococcus epidermidis b) Friedrich’s ataxia,
c) Duchenne muscular dystrophy
d) Guillain Barre Syndrome
e) Charcot-Marie-Tooth Disease
Claire Strauss BM BCh (Oxon) MA (Oxon) PGDip MRCPCH, North Mid-
dlesex University Hospital NHS Trust, UK. Conflict of interest: 2) Which two investigations are most likely to be diagnostic?
none declared. a) Creatinine Kinase

PAEDIATRICS AND CHILD HEALTH 30:3 114 Ó 2019 Elsevier Ltd. All rights reserved.
SELF-ASSESSMENT

b) Electromyography k) Urinary ketones


c) Lumbar puncture for oligoclonal bands
2) What is the most likely diagnosis?
d) MRI calves
a) Tuberous sclerosis
e) Muscle biopsy
b) Late onset Group B Streptococcal sepsis
f) Microarray
c) Meningitis
g) Tensilon test
d) Maple Syrup Urine Disease
h) Karyotype
e) Phenylketonuria
i) MRI spine
j) Nerve biopsy
Answers
3) Which would be the most appropriate initial treatment?
a) Corticosteroids Case 1
b) Rest and monitoring of respiratory function 1) B: Escherichia coli
c) IVIG 2) D: Posterior urethral valves
d) Acetylcholinesterase inhibitor 3) E: Micturating Cystourethrogram
e) Plasma Exchange The typical organism causing urinary tract infections (UTIs) is
E. coli. Other (atypical) organisms which cause UTIs include:
Case 3
Klebsiella spp, Pseudomonas aeruginosa Enterococcus spp.,
A first time mother brings her 7 day old infant to hospital with
Proteus spp., Enterobacter spp., coagulase-negative Staphylo-
concerns about poor feeding and frequent crying for 3 days.
cocci and Acinobacter spp.
The antenatal period was unremarkable with normal ultra-
This child has had multiple UTIs, including this acute and
sounds. The mother was healthy, not taking any medications
atypical one, at a young age (n.b. in accordance with NICE
or illicit drugs. The baby was born at 37 þ 2 by spontaneous
guidance urine samples which are positive for leucocytes or
vaginal delivery and discharged home the same day.
nitrites should be sent for culture). There should therefore be a
The family is of Eastern European origin and the parents
high suspicion of a renal tract abnormality. His mother did not
are first cousins. The mother had a younger sister who died in
receive full antenatal care which meant that any renal pelvis
early infancy.
dilatation was not detected.
The baby is mixed fed but requires waking in order to feed.
Posterior Urethral Valves (PUV) are the most common
He spends 10 minutes intermittently at the breast and then
cause of bladder outflow tract obstruction in boys. They
takes approximately 10e20 ml of term formula. He usually has
should ideally be suspected antenatally through renal dilata-
small vomits afterwards. He passes urine approximately 2e3
tion (and oligohaydramnios if severe) on ultrasounds. The
times a day and soft yellow stool once a day.
earlier they are detected the better the outcome. Delayed
Observations
identification can result in severe Vesiculo-ureteric-reflux,
T: 36.9 degrees Celsius
recurrent UTIs and renal scarring. It can therefore lead to
HR 148/minute
chronic renal failure and even a need for renal transplant.
RR: 46/minute
This is why it is essential to investigate UTIs in very young
Oxygen Saturations: 99% in air
children, particularly if atypical or recurrent (see NICE guid-
ance link below).
Blood glucose 4.2mmol/L
A thickened and trabeculated bladder wall is a classic
On examination, the baby is asleep but rousable. He is
finding on ultrasound in a case of PUV. Whilst initially there is
mildly jaundiced. His fontanelle is normotensive. Cardiovas-
renal dilatation if the valves are not treated the kidneys will
cular, respiratory and abdominal examinations are normal.
scar and shrink. The gold standard of imaging for diagnosis is
Tone in his legs is increased and he does not display a moro
the MCUG.
reflex. The baby’s legs appear to be cycling. The nappy is wet
An MCUG involves catheterizing the child and filling the
and smells sweet.
bladder with contrast. The child then empties their bladder. X-
1) Which TWO initial investigations are most likely to provide
rays are taken throughout this process demonstrating any
a diagnosis?
reflux of contrast into the ureters.
a) Serum ammonia
Following diagnosis the main treatment is valve ablation
b) Plasma Amino acids
although urinary diversion is sometimes used.
c) Urine organic acids
d) Lumbar puncture
e) Serum lactate
Further reading
f) Blood culture
g) Blood glucose https://www.nice.org.uk/Guidance/CG54
h) Creatinine Kinase Gilcrease-Garcia, B. et al. Posterior urethral valves [Radiopaedia].
i) Cranial ultrasound Available at: https://radiopaedia.org/articles/posterior-
j) EEG urethral-valves [Accessed 28th July 2019].

PAEDIATRICS AND CHILD HEALTH 30:3 115 Ó 2019 Elsevier Ltd. All rights reserved.
SELF-ASSESSMENT

Useful patient information: https://www.gosh.nhs.uk/conditions- 2) D: Maple Syrup Urine Disease


and-treatments/conditions-we-treat/posterior-urethral- Although the correct answer here is Maple Syrup Urine Dis-
valves-puv ease (MSUD) it is more important to recognise a likely meta-
Case 2 bolic condition. Important initial investigations for a potential
1) C: Duchenne Muscular Dystrophy metabolic order include; serum ammonia, serum lactate,
2) E and F: Muscle Biopsy and Microarray serum glucose and ketones.
3) A: Corticosteroids There are multiple clues in this vignette to an underlying
Muscular Dystrophy should always be suspected in a boy with metabolic condition:
gross motor delay. At the age of 4 Duchenne Muscular Dys-  The parents’ consanguinity
trophy (DMD) would be the most likely form. Becker muscular  Symptoms began after the baby would have fully estab-
dystrophy tends to present later and more mildly. lished feeding
DMD is an X-linked genetic condition where there is a lack of  Abnormal neurology
dystrophin in skeletal muscles. The main problem is progres-  A family history of early infant death
sive skeletal muscle weakness but cardiomyopathy can occur The sweet-smelling urine is a specific clue to MSUD.
and up to 30% of children with DMD have intellectual delay. Although also a metabolic disease phenylketonuria
DMD is a progressive, life limiting condition with a typical wouldn’t be expected to present until several months of age,
life expectancy of no more than 30 years, even with respira- or so acutely, and would not cause sweet smelling urine.
tory support. Most of the other investigations listed would be useful and
Although there is significant ongoing research there is important to do in a neonate with suspected seizures but not
currently no cure for DMD. Steroids can improve muscle diagnostic of a metabolic condition. Hypoglycaemia and
function but other treatment is supportive such as gentle ex- hyperammonaemia are rare in MSUD.
ercise and management of cardiomyopathies. Potential thera- Maple syrup urine disease (MSUD) is an inherited disorder
peutic approaches for the future include dystrophin in which the body is unable to process branched chain amino
restoration or replacement through genetic therapies, stem cell acids properly due to an absence or defect of the branched-
treatment and mechanisms of muscle growth and protection. chain alpha-ketoacid dehydrogenase complex enzymes. It is
The history given; of new tiptoe walking and difficulty named for the classically sweet smelling urine of affected in-
climbing stairs is classic in Duchenne Muscular Dystrophy due fants. There are several varieties, the neonatal type is called
to proximal leg weakness. Gower’s sign (walking hands up classic maple syrup urine disease.
legs to stand up) is a further classic feature. Calf psudohy- The condition is autosomal recessive and the estimated
pertrophy is due to a buildup of fatty and connective tissue. frequency is 1 in 185,000 infants. It is one of the metabolic
There may also be muscle aches and scoliosis may develop. conditions tested for on the newborn blood spot card.
Muscle biopsy will show replacement of dystrophin with The initial presentation may be non-specific with signs
fatty connective tissue and is diagnostic for DMD. Genetic present by day 4e5 of life:
investigations will also be diagnostic and determining the  Poor feeding
exact mutation may help the family in testing younger children  Vomiting
or for prenatal testing.  Lethargy
Guillain Barre syndrome would usually come on more In MSUD however these signify encephalopathy which may
acutely and is not associated with developmental delay or also cause abnormal movements (fencing and cycling) as in
large calves. There is often a sensory component. this case. Untreated, it progresses to seizures, coma and death.
Other differential diagnoses to be considered are spinal In the classical form this may occur by 7e10 days of age.
muscular atrophy type 3 and juvenile myotonic dystrophy. MSUD is characterized by disturbed amino acid concen-
trations is plasma, particularly increased branched chain
amino acids due to a defect in the branched chain alpha-keto
Further reading acid dehydrogenase complex. This results in large quantities
https://www.genome.gov/Genetic-Disorders/Duchenne- of branched chain amino acids and branched-chain ketoacids
Muscular-Dystrophy in the serum and urine.
https://rarediseases.info.nih.gov/diseases/6291/duchenne- A diagnosis can also be reached through genetic testing for
muscular-dystrophy the most common causative mutations as well as enzymatic
https://www.ncbi.nlm.nih.gov/pmc/articles/PMC3482409/ analysis of lymphocytes and skin cells.
Source of support for families: https://www.mda.org/disease/ Later, acute crises in MSUD can result from psychological
duchenne-muscular-dystrophy or physiological stresses (e.g. infection, surgery). In younger
https://www.musculardystrophyuk.org/ children these cause nausea, anorexia, altered level of con-
sciousness, acute dystonia, and ataxia. In older children, they
may cause cognitive impairment and hyperactivity. These
Case 3 crises are associated with a risk of cerebral oedema and coma.
1) B and C: Plasma Amino Acids and Urine Organic Acids Individually these episodes are largely reversible but if

PAEDIATRICS AND CHILD HEALTH 30:3 116 Ó 2019 Elsevier Ltd. All rights reserved.
SELF-ASSESSMENT

prolonged, particularly in the developing brain, they can cause National Organization for Rare Diseases (2017), Maple syrup
long term neurological damage. Ketonuria can be used as a urine disease. [Online] Available at: https://rarediseases.org/
surrogate marker for the metabolic instability which may rare-diseases/maple-syrup-urine-disease [Accessed 28 July
indicate a pending crisis. 2019].
Management of maple syrup urine disease is largely diet Strauss K. et al. (2006) Maple syrup urine disease [online].
modification to avoid leucine, isoleucine and valine. Available at: https://www.ncbi.nlm.nih.gov/books/NBK1319
Families should also be offered genetic testing and coun- [Accessed 28 July 2019].
selling as well as advice on the need to prevent and seek early
medical attention for metabolic crises.

Further reading
Burton B (1998) Inborn errors of metabolism in infancy: a guide to
diagnosis. Pediatrics [online]. Volume 102, Issue 6. Available
at: https://pediatrics.aappublications.org/content/102/6/e69
[Accessed 28 July 2019].

PAEDIATRICS AND CHILD HEALTH 30:3 117 Ó 2019 Elsevier Ltd. All rights reserved.
SELF-ASSESSMENT

Self assessment
Questions
Case 1 5) How long would you observe a child who has ingested
A 2-year-old girl was brought to the Emergency Department Methadone?
(ED) by ambulance. The child had been found unconscious a) At least 6 hours
and covered in vomitus by the ambulance crew. She was re- b) At least 8 hours
ported to be well that morning. She is known to social ser- c) At least 10 hours
vices, and there is a history of maternal drug abuse and d) At least 12 hours
depression. On arrival to ED her heart rate was 80 beats per
minute, respiratory rate 12 per minute, saturation was 96%, Case 2
BP: 96/66 mmHg. Her GCS was 5/15 and pupils were equal A 1-year-old boy presents to the paediatric Emergency Depart-
and constricted. ment with a midline lower sternal swelling, measuring approxi-
1. What is the most likely drug overdose responsible for her mately 1 x 1 cm that has appeared all of a sudden. The mass is fixed
clinical condition? to the underlying bone, is not pulsatile, has mild overlying redness
a) Fluoxetine of the skin and is tender only on palpation. The boy is otherwise
b) Opioid intoxication well in himself with normal cardiovascular, respiratory and
c) Tricyclic antidepressant intoxication abdominal examinations. There is neither hepatosplenomegaly
d) Paracetamol nor any lymphadenopathy. All observations are within normal
2. Given her clinical condition which TWO are the most parameters, and he is eating and drinking as normal.
important initial investigations? 1. What are the next most important investigations to be
a) Chest X-ray performed in the ED? (Select TWO answers ONLY).
b) Capillary blood gas a) Chest X-ray
c) Urine toxicology b) Ultrasound scan
d) ECG c) MRI
e) Blood culture d) Blood tests e including CRP and WBC
f) Blood glucose e) CT scan
Due to her reduced level of consciousness she was f) Biopsy
promptly intubated and ventilated. High ventilator settings An ultrasound scan shows a mixed echogenic solid mass in
were required. A chest X-ray was performed which showed the lower chest wall adjacent to the costal cartilage. Blood
evidence of aspiration pneumonia. Her blood gas showed tests show a white cell count of 17.4 and a CRP of 9.
respiratory acidosis. In addition an ECG was also.requested 2. What THREE diagnoses are most likely to explain the
(Figure 1). child’s symptoms and radiological findings?
3. What does the ECG show? a) Dermoid cyst
a) Sinus arrhythmia b) Osteomyelitis
b) Heart block c) Neurofibromatosis
c) Prolonged QTc d) Ewing’s sarcoma
d) Torsade de pointes e) Chondrosarcoma
Her mother later mentioned that her Methadone medica- f) Self-limiting sternal tumours of childhood (SELSTOC)
tion was missing. g) Osteochondroma
4. What is the antidote? h) Aneurysmal bone cyst secondary to fibrous dysplasia
a) N-acetyl cysteine i) Osteoid osteoma
b) Pralidoxime chloride 3. What is the appropriate management strategy for this
c) Naloxone hydrochloride condition?
d) Desferroxamine a) NSAIDs
e) Flumazenil b) A wait-and-see approach
c) Surgical excision of the mass
d) Antibiotics
e) Radiotherapy

Peter Heinz State Exam Med Dr med MRCP FRCPCH, Consultant Case 3
Paediatrician, Addenbrooke’s Hospital, Cambridge, UK. An 8-year-old girl is brought to the Emergency Department by
Note: this self assessment has been compiled using cases from her mother. She presents with a 4-day history of abdominal
previous publications. pain, loss of appetite, and three episodes of non-bilious

PAEDIATRICS AND CHILD HEALTH 30:5 191 Ó 2020 Elsevier Ltd. All rights reserved.
SELF-ASSESSMENT

Figure 1 12 lead ECG.

vomiting. She describes her pain as 20-minute-long episodes b) Abdominal X-ray


of cramps that are associated with eating, which began in the c) Abdominal CT scan
right upper quadrant, and have now moved centrally. She has d) Abdominal Ultrasound scan
not opened her bowels in 2 days, and last vomited after e) Blood tests: FBC, LFTs and CRP
breakfast this morning. She was diagnosed with a mild iron f) Blood tests: FBC, coeliac screen
deficiency anaemia one month ago, for which she has been
3. Which TWO other presenting complaints are most
prescribed ferrous sulphate tablets. Kate’s height is on the 25th
commonly seen in this diagnosis?
percentile, and her weight is on the 10th percentile for her age.
a) Abdominal distension
She has a family history of coeliac disease on her mother’s
b) Fever
side.
c) GI bleeding
On examination, there is a non-tender solid mass, 6 x 10
d) Halitosis
cm in size, in the left upper quadrant. The remainder of her
e) Diarrhoea
abdominal exam is normal, she is afebrile, and her heart rate
f) Jaundice
and respiratory rate are within normal limits. She was born at
g) Faltering growth
term by normal vaginal delivery, has no other significant past
h) Haematemesis
medical history and her immunisations are up to date.
1. What is the most likely diagnosis?
a) Cholecystitis Answers
b) Late-onset hypertrophic pyloric stenosis
Case 1
c) Duodenal cancer
1. b
d) Rapunzel syndrome
2. b, f
e) Intussusception
3. c
f) Iron toxicity
4. c
g) Coeliac disease
5. a
h) Appendicitis
Prescription opioid ingestion is one of the most common forms
i) Peptic ulcer disease
of poisoning in children less than 6 years of age. Recent years
2. Which TWO investigations should be ordered first? have seen a large increase in the prescription opioids either for
a) Oesophagogastroduodenoscopy opioid detoxification or pain relief. Methadone is a synthetic

PAEDIATRICS AND CHILD HEALTH 30:5 192 Ó 2020 Elsevier Ltd. All rights reserved.
SELF-ASSESSMENT

opioid. Children are particularly susceptible to the effects of considered the most likely diagnosis over a benign dermoid
methadone, with doses of 10 mg being fatal. The classical cyst and an osteoid osteoma as a dermoid cyst often develops
findings of opioid toxicity include: miosis, CNS depression, very slowly and an osteoid osteoma has different radiological
and respiratory depression. Hypotension, bradycardia, hallu- findings (a round lucency that has a dense sclerotic central
cinations, non-cardiogenic pulmonary oedema and rhabdo- nidus). The initial investigation in this situation is often an
myolysis have also been reported. Most deaths are related to ultrasound scan as it is non-invasive and will help to distin-
respiratory depression. Airway management to ensure guish between a solid and/or cystic swelling and its size and
adequate ventilation and oxygenation is the priority. In chil- relation to neighbouring tissues. X-rays will provide little
dren with respiratory depression, naloxone can be adminis- diagnostic information and CT and MRI scans are likely to
tered. The patient should be observed at least 8 hours require sedation or even a general anaesthetic in this age
following opioid exposure or at least 6 hours after the last dose group. Blood tests such as CRP and white cell count can be
of naloxone. This is because a “paradoxical” increase in opioid used to rule out infection as a cause of the swelling. Ultra-
effect may occur after naloxone administration as it reverses sound of the swelling typically reveals a pre-sternal and
gastro paresis caused by opioid and increases its absorption. sometimes retro-sternal swelling of the soft tissue. Most le-
Blood glucose should be measured as an ancillary test. sions appear to be dumbbell-shaped with a hypo-echoic and
Monitor BP, pulse, RR, saturations and conscious level. Consider inhomogeneous aspect with sharp borders. The lesion consists
blood gas measurement in patients with reduced level of con- of very poorly vascularised tissue, with no connection to the
sciousness and reduced respiratory rate. Methadone may pro- skin or infiltration into underlying bone or muscles.
duce cardiac effects at high doses, including QT prolongation and Histologically, SELSTOC most often show aseptic chronic
torsade de pointes, therefore a 12 lead ECG should be performed. or acute inflammation. Because of this non-specific pathology
Agitation following naloxone treatment should be managed combined with the benign nature of these tumours, the
without sedation, by nursing in dark and quiet environment. treatment is conservative, with a wait-and-see approach with
close follow up in order to prevent invasive diagnostic in-
terventions and unnecessary treatment. SELSTOC may take up
FURTHER READING to six months to resolve.
Toxbase. Methadone Hydrochloride. [ONLINE] Available
at:https://www.toxbase.org/. (accessed 10 Jan 2020).
FURTHER READING
Case 2
Te Winkel ML et al. Self-limiting sternal tumors of childhood
1. b, d
(SELSTOC). Pediatr Blood Cancer 2010; 55:81e84.
2. a, f, i
3. b Case 3
Self-limiting sternal tumours of childhood (SELSTOC) refers to 1. d
a rare, rapidly growing sternal mass due to aseptic inflam- 2. d, f
mation. It is a benign sternal tumour that has neither a 3. d, g
neoplastic nor infectious origin. SELSTOC has so far only been The presenting complaints of non-bilious vomiting and con-
reported in young children. stipation, together with the examination finding of a mass in
An exaggerated response to a certain stimulus, for example the left upper quadrant suggest that some form of small bowel
mild trauma, may play a role in the pathophysiology of the obstruction is the primary cause of her illness. Late-onset
condition. hypertrophic pyloric stenosis is extremely rare, but has been
The main presenting complaint of patients with SELSTOC is observed in children between the ages of 2 and 8 years. In this
a rapidly growing sternal mass. Parents may also feel their Case however, the size of the mass suggests an obstruction of
child has localised pain and a mild fever. A history of mild a different nature.
trauma prior to the development of the mass is often present. Rapunzel syndrome is a fairly rare condition that results
Typically patients presenting with SELSTOC are well. The from the ingestion of large quantities of hair. The body of a
tumour itself normally appears to lie pre-sternally or para- trichobezoar collects in the stomach, with the tail extending
sternally, with the skin overlying the tumour occasionally into the small intestine and, in extreme cases, the right colon.
having a red/blue appearance. Chest wall tumours can origi- In this Case, the tail of this child’s trichobezoar extended
nate from various local tissues: the subcutaneous soft tissue, only as far as her duodenum, however, the size of the body
cartilage, the bony chest wall or the extra pleural region. They within the stomach was sufficient to cause gastric distension
are rare in childhood, and can be grouped into malignant and and obstruction of the pyloric outlet, leading to vomiting,
benign tumours. Malignant tumours include Ewing’s sarcoma, constipation and weight loss. The size of the hairball can
and chondrosarcoma amongst other primary tumours. Sec- result in severe cramps during stomach peristalsis. Cases
ondary tumours may metastasise from an adjacent lung can- such as this typically occur in young girls and are sometimes
cer. Benign tumours of the chest wall include, but are not associated with the psychiatric conditions: trichophagia,
limited to, SELSTOC, osteochondroma, benign dermoid cysts, trichotillomania and pica. Trichobezoars typically require
chondroma, and fibrous dysplasia. SELSTOC can be surgical removal.

PAEDIATRICS AND CHILD HEALTH 30:5 193 Ó 2020 Elsevier Ltd. All rights reserved.
SELF-ASSESSMENT

Figure 2 Trichobezoar removed from stomach and duodenum.

The trichobezoar removed from this girl’s stomach can be seen due to the delayed passage of food through the stomach.
seen in Figure 2. Although abdominal distension is a common sign of small
Although the diagnosis of Rapunzel syndrome is reason- bowel obstruction, the position of the mass in this Case would
ably straightforward, this patient’s family history of Coeliac be too high up to cause a buildup of gas in the bowel.
disease together with the presentation of abdominal pain,
anaemia and relatively low height and weight should raise
concerns about the possibility of coeliac disease, for which FURTHER READING
appropriate investigations should be ordered. Although a tri- Gonuguntla V, Joshi DeD. Rapunzel syndrome: a comprehensive
chobezoar is non-toxic and does no direct damage to the review of an unusual Case of trichobezoar. Clin Med Res
gastrointestinal tract, the sheer size of the mass, and the 2009; 7 (3):99e102. https://doi.org/10.3121/cmr.2009.822.
inability of the human digestive system to break down hair, Kumar BN A, Kumar L N, Thippeswamy J, Rangaswamaiah LN.
means that they can be fatal if left untreated. This is usually Trichobezoar (Rapunzel syndrome) in an adolescent patient
due to the consequences of small/large bowel obstruction. with trichotillomania and generalized anxiety disorder: a Case
Chronic obstruction can also result in weight loss and growth report. Asian J Psychiatr. 2016; 23:44e45. https://doi.org/10.
deceleration in children. Severe halitosis is sometimes also 1016/j.ajp.2016.07.008.

PAEDIATRICS AND CHILD HEALTH 30:5 194 Ó 2020 Elsevier Ltd. All rights reserved.
SELF-ASSESSMENT

Self assessment
Questions d) This is an atypical UTI and renal tract ultrasound should
be performed within 6 weeks
Case 1
An ultrasound scan is performed and this shows that Luke
You see Luke, a 10-week old boy in the emergency department.
has bilateral hydronephrosis. A micturating cysto-urethrogram
He has been unsettled for the past 24 h in that has been vomiting
is then ordered, and an image of this is shown in Figure 1.
and is not able to tolerate feeds. Today his temperature was 38 C
2. What does the image show? Choose THREE correct
at home and is 38.6 C in the department. You obtain intravenous
answers.
access and send blood tests, including blood cultures. A clean
a) Urethral stricture
catch urine sample is also collected and cerebrospinal fluid (CSF)
b) Ectopic urethral orifice
sent to the laboratory for analysis. The baby recovers well after
c) Ureteric dilatation
48 h IV antibiotics and completes a further 5 days treatment.
d) Posterior urethral valves
Blood and CSF cultures show no bacterial growth. The
e) Atopic/pelvic kidney
urine white blood cell count on microscopy is >105/ml and
f) Trabeculated bladder
culture shows Klebsiella pneumoniae >105/ml.
g) Duplex kidney
1. Which of the following statements is true regarding initial
investigation required in this case (choose ONE)?
a) This is an uncomplicated UTI, so no further imaging is Case 2
required Haroon is a 5-year-old boy brought into the Emergency
b) This is an atypical UTI and renal tract ultrasound is Department by ambulance from school. He suffered a fall in
required during the acute illness the school playground in the morning, hitting the back of his
c) This is an uncomplicated UTI and renal tract ultrasound head on a rubber obstacle, after which he appeared to have a
should be performed within 6 weeks seizure, lasting approximately 30 min, during which he was
unresponsive, incontinent of urine and faeces, and was seen to
be foaming at the mouth. He initially complained of a head-
ache at the back of his head, however this resolved within an
hour, and he has been gradually returning to normal levels of
consciousness.
Haroon suffers mild non-specific headaches once or twice
in a month, but has no history of seizures and is well nor-
mally. His weight is on the 7th percentile and his height is on
the 85th percentile for his age. His mother had a haemorrhagic
stroke 11 years before giving birth, and takes levetiracetam to
prevent seizures following this. His father had a case of
tuberculosis 5 years ago, but has been well since. There is no
other family history of note, and he is an only child. He is of
Indian descent, and last visited India 18 months ago. He is
afebrile, and a physical exam reveals no abnormalities. A CT
scan with contrast of Haroon’s head is performed and this is
shown in Figure 2.
1. What is the most likely reason for the ventricular asym-
metry seen? (Choose ONE only)
a) Space occupying lesion
b) Cerebral oedema
c) Positioning in scanner
d) Congenital anomaly
e) Intraventricular haemorrhage
Two lesions in the left frontal and right parietal lobes were
noted, each measuring approximately 2 cm in diameter. After
Figure 1 Micturating cysto-urethrogram.
injection of contrast, these showed mild enhancement.
2. Which of the following investigations should be carried out
next to rule out any underlying conditions? (Choose TWO
Peter Heinz State Exam Med Dr med MRCP FRCPCH, Consultant answers)
Paediatrician Addenbrooke’s Hospital, Cambridge, UK. Conflicts a) MRI head and spine
of interest: none declared. b) Chest X ray

PAEDIATRICS AND CHILD HEALTH 30:6 239 Ó 2020 Published by Elsevier Ltd.
SELF-ASSESSMENT

d) Endovascular surgical occlusion of malformations


e) Use of ibuprofen as needed for management of
headaches
f) Treat conservatively, and arrange regular MRI scans to
monitor the size of malformations
Case 3
Andrew, a 14-year-old boy, presents to the paediatric Emer-
gency Department with very swollen and red eyes, oral and
genital ulceration and a throat that is covered in slough.
Additionally, he complains of a burning sensation when he
passes urine. He has a one week history of ongoing headaches,
lethargy and myalgia. He has been taking amoxicillin for 2
days (which he has taken in the past without any adverse
reactions), however the symptoms started prior to taking an-
tibiotics. Andrew had his meningococcal vaccine seven days
earlier and had felt unwell following this. On examination, he
has swollen lips and mucous membranes inside his mouth and
red conjunctiva, in addition to ocular, oral and genital ulcer-
ations. He exhibits no respiratory or cardiovascular symp-
toms, and has no lymphadenopathy. He is febrile with a
temperature of 39C, his heart rate is 108 beats per min. A
Figure 2 CT scan of the head. presumptive diagnosis of Stevens-Johnson Syndrome (SJS) is
made.
c) Interferon gamma release assay 1. Which TWO initial investigations are most helpful at the
d) Full body CT time of presentation?
e) Cerebral angiogram a) FBC
f) Lumbar puncture b) Serum creatinine
g) Biopsy of intracerebral lesions c) CXR
3. What are the two most likely diagnoses based on the his- d) Skin biopsy
tory and CT scan? (Choose TWO answers) e) Throat culture
a) Tuberous sclerosis f) INR
b) Transverse myelitis 2. Which of the following do NOT feature in the SCORTEN
c) Hydrocephalus score? (Select ONE answer ONLY).
d) Tuberculous granulomas a) Age
e) Cerebrovascular accident b) Urea
f) Cerebral cavernous malformations c) Compromised body surface area
g) Arachnoid cysts d) Serum glucose
h) Metastatic disease e) Serum bicarbonate
After further investigations, the radiologist’s report f) Respiratory rate
concluded that the 2 lesions represented cerebral cavernous
malformations. Seizures and non-specific headaches are the 3. What are the THREE management priorities?
most common clinical presentation of this condition. a) Nutritional support and IV fluids
4. What other clinical signs might you expect to see in pa- b) Antivirals
tients with this condition? (Choose ONE only) c) NSAIDs
a) Papilloedema d) Withdrawal of causative agent
b) Focal neurological deficits e) Paracetamol
c) Limb paraesthesia f) Dressings, topical antibiotics and emollients
d) Hemiplegia g) A wait-and-see approach
e) Decline in cognitive abilities h) IV antibiotics
i) UV light therapy
5. What management is most appropriate going forwards?
(Choose TWO answers) 4. Which THREE of the following are common complications
a) Surgical removal of lesions (craniotomy) during the course of SJS?
b) Start levetiracetam and arrange follow up with neurology a) Dehydration
c) Arrange head MRIs and genetic testing of family b) Oesophageal stricture
members c) Anaemia

PAEDIATRICS AND CHILD HEALTH 30:6 240 Ó 2020 Published by Elsevier Ltd.
SELF-ASSESSMENT

d) Osteopenia patients suffering from them at some point. Other commonly


e) Re-feeding syndrome observed symptoms include focal neurological deficits, non-
f) Urinary retention specific headaches, and cerebral haemorrhages.
CCMs have been observed across a wide range of patient
demographics, and in some cases, it has been found that there
Answers
appears to be a genetic component to the condition, with
Case 1 similar malformations observed in first degree relatives.
b Although genetic evidence concerning familial cerebral
c, d and f cavernous malformations is limited, studies have revealed a
Luke’s urine has grown an atypical organism (other than e- correlation between this condition, and pathogenic variants in
coli), so he should be treated as having an atypical UTI. Ac- the genes KRIT1, CCM2 and PDCD10.
cording to the NICE guidance on UTI in children, he should Owing to the pattern of inheritance seen in some cases, it is
have an ultrasound during the acute illness and should then pertinent to screen the relatives of patients with CCMs, so that
have outpatient MCUG and DMSA scans. they may take the correct precautions, or undergo any
The MCUG shows a trabeculated bladder with dilatation of necessary treatments, were they to be found to have similar
the posterior urethra as well as a degree of vesico-ureteric presentations. This most commonly involves a genetic test,
reflux with ureteric dilatation. and in some circumstances an MRI.
The ultrasound and MCUG findings are consistent with a Management of CCMs depends heavily on the symptoms of
diagnosis of posterior urethral valves. Posterior urethral the individual. In patients with only mild symptoms or those
valves are membranous folds obstructing the posterior urethra completely asymptomatic, treatment most commonly involves
and are the most common cause of urinary tract obstruction in the use of anti-seizure medication, such as levetiracetam, and
male babies. analgesia for the management of headaches. Ibuprofen and
other NSAIDs should be avoided, due to the increased likeli-
hood of haemorrhage. In patients with intractable seizures and
Further reading focal deficits due to mass effect, surgical removal of the lesions
Urinary tract infection in under 16s: diagnosis and management. may be considered, depending on their location and
Clinical guideline [CG54] Published date: August 2007 Last morphology.
updated: October 2018, accessed Jan 2020. https://www.
nice.org.uk/guidance/cg54
Further reading
Case 2
1. c Gonuguntla V, Joshi DeD. Rapunzel syndrome: a
2. a, c comprehensive review of an unusual case of trichobezoar.
3. d, h Clin Med Res 2009; 7(3):99e102. doi:10.3121/cmr.2009.822.
4. b
Case 3
5. b, c
1. a, d
The CT scan shows two lesions in the left frontal and right
2. f
parietal lobes. Without any further investigation, the most
3. a, d and f
likely identities of these lesions are either tuberculous granu-
4. a, e and f
lomas, or metastases, most commonly from a primary tumour
Stevens-Johnson Syndrome is a potentially fatal condition,
elsewhere in the CNS. For this reason, appropriate in-
where apoptosis of the epidermis causes it to separate from the
vestigations to diagnose tuberculosis and an MRI of the head
dermis. The condition often begins with flu-like symptoms,
and spine should be arranged, firstly to provide a more
including high temperature, joint pain and a headache. These
detailed picture of the lesions and secondly to look for any
symptoms often precede cutaneous and mucosal manifesta-
potential primary tumours. Following an MRI study of
tions by a few days. Cutaneous manifestations include
Haroon’s head and spine, it was eventually concluded that the
erythematous macules that tend to quickly coalesce, whereas
lesions seen on the CT scan most likely represent cerebral
mucosal involvement includes erythema and erosions of the
cavernous malformations. Cerebral cavernous malformations
buccal, genital or ocular mucosa. A combination of both these
(CCMs) are vascular deformities which can be found either in
cutaneous and mucosal manifestations is highly suggestive of
the brain or spinal cord. They most commonly consist of
SJS. Prognosis of the condition is determined by calculating
clusters of enlarged capillary channels, and can have signifi-
the SCORTEN score, which takes into account factors such as
cant neurological impacts in patients, either due to mass ef-
age, percentage surface area of epidermal detachment, serum
fects on adjacent areas of the brain, or due to their inherent
urea and glucose. A score of 3 or above should be managed in
increased likelihood of haemorrhage. Seizures are the most
an intensive care unit.
common symptom in patients with CCM, with up to 70% of

PAEDIATRICS AND CHILD HEALTH 30:6 241 Ó 2020 Published by Elsevier Ltd.
SELF-ASSESSMENT

SJS is related to the condition Toxic Epidermal Necrolysis ulceration. Emollients should be used to prevent water loss
(TEN). SJS and TEN are considered to be two ends of a and protect the skin. Nutritional support and IV fluids should
spectrum of severe cutaneous reactions which affect mucous be administered to prevent malnutrition and dehydration.
membranes and the skin. The distinction between SJS, SJS/ Although analgesia is invariably required, one should be
TEN overlap and TEN is made by the percentage body surface careful when giving NSAIDs or paracetamol because they have
area with epidermal detachment. SJS is categorized by under been recorded as causative agents of SJS. Plasmaphaeresis,
10% cutaneous involvement and TEN is categorized by over immunosuppressive therapy, and intravenous immunoglob-
30%. In the stage of epidermal detachment, patients will have ulin have been used but to date there is no evidence to support
a positive Nikolsky sign, where rubbing of the skin causes routine use. The use of systemic steroids remains controver-
exfoliation of the epidermis. However this sign is not specific sial and may lead to an increase in complications, especially if
to SJS or TEN, it may be positive in other conditions such as used long term. Dehydration is a common complication of
autoimmune bullous skin diseases. Drug exposure is the most SJS/TEN because of painful swallowing. As a result, serum
common cause of SJS/TEN. Common triggers include allopu- urea and electrolytes need to be carefully monitored to detect
rinol, sulfonamide antibiotics and aminopenicillins such as hypovolaemia, and a nasogastric tube usually needs to be put
amoxicillin. In this case, since the symptoms appeared before in place to ensure fluid and food intake. Another common
amoxicillin was administered, a drug induced hypersensitivity complication is re-feeding syndrome. After a period of
reaction is unlikely. Other triggers include infection, particu- decreased nutrient intake followed by re-feeding, patients can
larly in children. The pathophysiology of SJS is unknown, develop electrolyte disorders, particularly hypo-
however cytotoxic CD8flT lymphocytes play a role in the phosphataemia. During fasting, the body switches its main
initiation phase, where they initiate apoptosis through the fuel source from carbohydrates to fatty acids or amino acids,
release of proteins like perforin and granzyme B. The defini- and insulin secretion is suppressed. During re-feeding, insulin
tive test for diagnosis of SJS is skin biopsy. A positive result is secretion increases in response to a raised blood sugar, leading
keratinocyte apoptosis with detachment of the epidermis from to synthesis of glycogen, proteins and fats. Insulin stimulates
the dermis. The area of separation may contain many CD8fl T potassium absorption into cells via the sodium-potassium
lymphocytes, the effector cell of this condition. A FBC is also ATPase symporter. Magnesium and phosphate are also taken
useful in determining the stage of the disease. TEN patients into the cells (the latter is important in phosphorylation
may present with sepsis and an elevated white cell count. transduction pathways). As a result, serum phosphate, po-
Serum creatinine is a useful investigation as it may indicate tassium and magnesium all decrease, which is problematic
renal failure, however it is not a primary investigation, nor is a since they are already severely depleted in the fasting state.
CXR. Although not mentioned, a blood culture is also impor-
tant, as it rules out toxic shock syndrome and scalded skin
syndrome, which would test positive for Staphylococcus or Further reading
Streptococcus species. Management of the condition is mainly Harr and French: Toxic epidermal necrolysis and Stevens-
supportive, often requiring intensive care. Since SJS is often Johnson syndrome. Orphanet J Rare Dis 2010; 5:39.
drug or infection induced, the first step of management is to Mehanna HM, Moledina J, Travis J. Refeeding syndrome: what it
remove the causative agent. Next, dressings and topical anti- is, and how to prevent and treat it. BMJ: Br Med J 2008;
septics should be used to cover cleansed wounds to prevent 336:1495e1498.
secondary infection and to treat mucous membrane

PAEDIATRICS AND CHILD HEALTH 30:6 242 Ó 2020 Published by Elsevier Ltd.
SELF-ASSESSMENT

Self-assessment
Questions e. Can tell long stories
f. Uses complex explanations and sequences of event
Case 1
James is a 3 year 6 months old boy referred to community 2) Which of the following is the most likely diagnosis?
paediatrics with parental concerns about his speech. (Choose 1)
James was born at 36 weeks by elective caesarean section a. Attention Deficit Hyperactivity Disorder
for breech. The pregnancy was unremarkable with normal b. Specific speech and language delay
antenatal scans. During the neonatal period, James required c. Autism spectrum disorder
nasogastric feeding for a short duration before he was estab- d. Landau- Kleffner syndrome
lished onto oral feeds. His neonatal hearing screen was e. Deafness
negative. James has had 2 ear infections, requiring oral anti- f. Global developmental delay
biotics and no serious illnesses. At present he is not on any
3) Which investigation would be most helpful at this stage?
medication. James has a 9 year old brother who has Attention
Please select ONE from the list.
Deficit Hyperactivity Disorder and his dad required extra
a. Electronencephalogram (EEG)
support at school.
b. Creatinine kinase
James sat unsupported at 7 months, crawled at 11 months
c. Brain magnetic resonance imaging (MRI)
and walked at 13 months. James had his first words at 12
d. Hearing assessment
months. He was joining 2 words at 2 years and having a to and
e. Genetic studies for Fragile X
fro conversation by 3 years. There were no concerns about
f. Urine organic acids
James’s hearing.
In clinic, parent’s report James can jump, climbs stairs 1
foot per step and ride a tricycle but is sometimes clumsy.
James can eat with a spoon and fork, undress himself with Case 2
assistance, build a tower of 9 cubes and copy a circle. There You are asked to review Freddie, a 7 year old boy, in the
are no concerns with his vision. neurodevelopmental clinic because the family have significant
James started at preschool 6 months ago and initially had concerns with Freddie’s concentration and interaction with his
some difficulty settling in. His teachers commented James peers in class. His teachers report his behaviour to be
could play in parallel and had reasonable eye contact, how- challenging.
ever when he got excited he would sometimes run in circles. Freddie was born at 31 weeks gestation via vaginal delivery
James would approach other children and his teacher’s report after an uneventful pregnancy. He was intubated briefly fol-
he can move between activities without difficulty. lowed by CPAP. He completed a course of antibiotics for
Parents have noticed since starting preschool, James is not presumed sepsis and was noted to have neonatal seizures in
speaking as much as he used to. Initially parents thought this first week of life. His medical history includes bilateral cere-
was because of his new environment, since this was the first bral palsy (GMFCS level IV), periventricular leukomalacia and
time he had been away from parents. However, parents were sleep difficulties. Current drug history includes Melatonin 2mg
most concerned as James could previously use sentences such once nightly.
as “Mummy I want a biscuit” and now he would only say Freddie has a sister who was also born premature, at 35
“Mummy” and point to a biscuit. weeks gestation and is “slightly clumsy”, but there are
1) What is the expected speech development in a 3 year old otherwise no concerns. He attends a mainstream school and
child? has a 1 to 1 teaching assistant.
a. Knows action words e.g. Can you point to the cup? Freddie can walk for several metres when motivated with
b. Understands prepositions e.g. under, behind, on his walker and needs supervision and help to transfer in and
c. Uses questions e.g. what and why? out of his walker. He can use a self-propelling wheelchair but
d. Follows two step command prefers to commando crawl indoors. He is known to physio-
therapy and has previously had Botox administration to lower
limbs for spasticity. Freddie can use a spoon but needs help
using a fork and knife. Freddie had a few words by 18 months
Arti Khistriya MBBS BMedSci MRCPCH ST4, Paediatric Trainee Royal and currently speaks in 4e5 word sentences, but can be very
Berkshire Hospital, Reading, UK. Conflicts of interest: none repetitive.
declared. He often wakes from midnight for 2e3 hours. Freddie
struggles with imaginative play and needs information and
Mallika Babu Kadaba Narasimha MBBS (Bangalore, India) MRCPCH activities broken into small chunks with rewards in order to
(UK), Consultant Neurodevelopmental Peadiatrician Royal Berk- maintain his attention. Freddie has restricted interests -
shire Hospital, Reading, UK. Conflicts of interest: none declared. currently cars and trains. He has some verbal rituals and needs

PAEDIATRICS AND CHILD HEALTH 30:7 270 Ó 2020 Elsevier Ltd. All rights reserved.
SELF-ASSESSMENT

a specific pre-bed routine. He is sensitive to noise and can and the episodes were increasing in frequency, occurring up to
have extremes of emotions with meltdowns. 60 times a day.
1) Which of the following would support a diagnosis of General examination showed features in keeping with
Autism Spectrum Disorder? (Choose 4) Down syndrome. Heart sound was normal and chest was clear
a. Failure to sustain attention in tasks or play activities with some transmitted upper airway sounds. Abdomen was
b. Frequent repetition of set phrases soft. Joseph had generalised low tone but reflexes were pre-
c. Often interrupts sent. He followed a shiny toy through 180 degrees but this was
d. Lack of response to greetings and smiles not always consistent.
e. Over or under-reactive to sensory stimuli 1) Which of the following is the most likely diagnosis? (Chose
f. Preference for routines and structure ONE)
g. Family history of reading, spelling and literacy difficulty a. Gastro-oesophageal reflux
b. Infantile Spasms
2) What is the prevalence of sleep disorder among children
c. Colic
with autistic spectrum disorder? (Choose 1)
d. Normal startle response
a. 1%
e. Myoclonic seizure
b. 5%
c. 25% 2) Which investigation would be most helpful at this stage?
d. 40% Please select ONE from the list.
e. 99% a. MRI brain
b. Electroencephalogram (EEG)
3) What further management of Autism Spectrum Disorder
c. Urine organic acids
would you recommend for this child?
d. Barium Swallow
a) Parenting programme to manage behaviour
e. Sleep study
b) Referral to Child Adolescent Mental Health Services
c) Occupational Therapy sensory workshop 3) What is the NEXT most appropriate management of this
d) Sleep course child?
e) SENCO support at school a. Urgent discussion with the Neurologist regarding
further management
Case 3
b. Ketogenic diet
Joseph was referred to a Community Paediatrician at 9 months
c. Prednisolone
of age because of global developmental delay and cortical vi-
d. Adrenocorticotrophic hormone (ACTH)
sual impairment.
e. Vigabatrin
Joseph is a twin boy with an antenatal diagnosis of Trisomy
21. He was born at term by elective caesarean section due to
maternal age. Joseph was born in good condition and did not Answers
require any resuscitation. He was admitted to the Special Care
Case 1
Baby Unit for nasogastric feeding support as he was noted to
1) A, B, C & D
have significant stridor and feeding difficulties thought to be
2) D
laryngomalacia and reflux.
3) A
On review, Joseph had good neck control and was
At around 2 years of age most children will have built up a
attempting to roll. He was sitting with support but was unable
core vocabulary of about 100e200 words and are able to join 2
to sit alone. When given a toy, Joseph would bring it to the
words together. By 3 years most children use action words and
centre but was unable to transfer toys between his hands.
use increasingly complex sentences including questions. Be-
When given a feeding bottle, he would hold onto the feeding
tween 4 and 5 years children can tell long stories which are
bottle and attempt to put a spoon into his mouth. He often had
mostly grammatically correct and give complex explanations
small possets after each feed and would look uncomfortable
and sequence of events.
when opening his bowels.
In this scenario, careful thought must be given to different
Parents had noticed continuous tuneful babbling and Jo-
causes of speech delay and the key feature of speech loss or
seph had recently started to blow raspberries but was not
regression which is typically seen in Landau-Kleffner Syn-
using any words. He liked to be cuddled and found it difficult
drome, making this the most likely diagnosis.
to be separated from his parents. According to parents, Joseph
Despite a positive family history of Attention Deficit Hyper-
could appreciate their presence if they are close to him but he
activity Disorder, the central features of inattention, over activity
could not fix and follow his vision at a distance.
and impulsivity are not present which would be necessary to
Parents had noticed he would startle even for day to day
make a diagnosis which typically occurs at 6 years age.
sounds of normal intensity. They described sudden bending
Autism Spectrum Disorder is unlikely because there are no
forward and stiffening of the body, arms and legs. There was
features of social interaction difficulties or stereotypic and
no obvious pattern or precipitating factors to the movements

PAEDIATRICS AND CHILD HEALTH 30:7 271 Ó 2020 Elsevier Ltd. All rights reserved.
SELF-ASSESSMENT

restricted behaviours. Likewise, global developmental delay is Deficit Hyperactivity Disorder (ADHD) and behaviour and
incorrect as only speech and language is affected with normal learning difficulties, therefore it is important to identify early
gross motor, fine motor and social development. and manage these conditions in conjunction. Below is a table
Landau-Kleffner syndrome should be considered in a child of common presenting features of children with ASD at
who has language regression. It occurs in children usually different ages:

Pre-school Primary school Secondary school


Communication difficulties Failure to respond to name. Limited or excessive talking at Flat or odd innotation. Unusual
Failure to point or wave. others. Frequent repetition of ways of making themselves
Language delay or regression. set phrases. Odd or understood. Taking things
Echolalia. inappropriate prosody. literally.
Social interaction difficulties Lack of responsive smile. Poor Temper tantrums. Lack of Long standing difficulties in
eye contact. Preference for response to greeting and behaviour and social
solitary play. Lack of turn smiles. communication. Lack of
taking. Lack of pretend play. awareness of personal space.
Difficulty with social situations
and rules.
Stereotypic and restricted Unusual attachment to object, Over-or underactive to sensory Unusual profile of skills and
behaviour toys etc. Unusual sensitivity to stimuli. Preference for routines deficits. Preference for narrow
textures, sounds and other and structures. and specific interests.
stimuli. Hand flapping, tip toe
walking.

between the ages of three and nine years and is characterised The reported prevalence of sleep disorders among children
by loss of language skills and silent electrical seizures. with autistic spectrum disorders (ASD) is between 40% and
Landau-Kleffner is a clinical diagnosis, however, there are 86%. The sleep disorders include taking longer to fall asleep
typical EEG abnormalities over the temporal region of the (longer sleep onset latency), frequent waking during the night
brain and these often increase in frequency during sleep. and shorter sleep.
There may be sudden (even overnight), or gradual loss of Melatonin is a naturally occurring hormone produced by
language understanding and speech and the syndrome has the pineal gland in the brain. It is involved in coordinating the
variable prognosis. Treatment includes steroids and anticon- body’s sleepewake cycle and helping to regulate sleep. It has
vulsants such as Ethosuxamide. However, experience shows been suggested that abnormal melatonin secretion might be
that when steroids are stopped, children often lose their lan- involved in the development and that treatment with mela-
guage again and seem more resistant to treatment after this. tonin might be beneficial.
NICE guidance suggests if a child or young person with
autism develops a sleep problem the following assessment
Further reading should be made:
1) Oxford Specialist Handbook of Community Paediatrics, Srinivas  what the sleep problem is (for example, delay in falling
Gada asleep, frequent waking, unusual behaviours, breathing
2) https://www.epilepsy.org.uk/info/syndromes/landau-kleffner- problems or sleepiness during the day)
syndrome  day and night sleep patterns, and any change to those patterns
3) https://www.gosh.nhs.uk/conditions-and-treatments/  whether bedtime is regular
conditions-we-treat/landau-kleffner-syndrome  what the sleep environment is like, for example:
Case 2 o the level of background noise
1) B, D, E & F o use of a blackout blind
2) D o a television or computer in the bedroom
3) A, B, C, D & E o whether the child shares the room with someone
Autism Spectrum Disorder (ASD) comprises of a triad of fea-  presence of comorbidities especially those that feature hy-
tures including communication and social interaction diffi- peractivity or other behavioural problems
culties in addition to stereotypic and restricted behaviour with
limited imaginary play. As autism is a spectrum of disorders, it  levels of activity and exercise during the day
can vary in severity and intellectual ability.  possible physical illness or discomfort (for example, reflux,
There are several associated co-morbidities with ASD ear or toothache, constipation or eczema)
including, mental health and emotional disorders, Attention

PAEDIATRICS AND CHILD HEALTH 30:7 272 Ó 2020 Elsevier Ltd. All rights reserved.
SELF-ASSESSMENT

 effects of any medication Infantile spasms are usually accompanied by a character-


istic pattern on the electroencephalogram (EEG). This pattern
 any other individual factors thought to enhance or disturb
is called ‘hypsarrhythmia’. The combination of the infantile
sleep, such as emotional relationships or problems at school
spasms, age of onset and EEG pattern defines the epilepsy
 the impact of sleep and behavioural problems on parents or syndrome called, ‘West syndrome’. The typical movement in a
carers and other family members. spasm is of a sudden flexion (bending forward) in a tonic
A sleep plan should be developed in conjunction with the (stiffening) fashion of the body, arms and legs. Sometimes, the
parent or carer to help establish a regular night time sleep episodes are different, with the arms and legs being flung
pattern and a sleep diary recorded over a 2 week period. outwards (these are called ‘extensor’ spasms).
Pharmacological intervention should only be indicated Infantile spasms are the most common seizure type
when sleep problems persist despite following the sleep plan occurring in children with Down syndrome with an overall
or sleep problems are having a negative impact on the child or prevalence of 2.5%e3.1%. Infantile spasms are associated
young person and their family or carers. Consultation with a with poor long-term developmental outcomes and further
specialist paediatrician or psychiatrist with expertise in the seizure types if not diagnosed and treated early.
management of autism should be sought before starting In the UK, prednisolone is recommended as first-line
treatment and medication should be reviewed regularly to therapy. Synthetic ACTH, also known as tetracosactide, is
ensure the benefits outweigh the side effects. proven to be an effective first line hormonal treatment in
The management of Autism Spectrum Disorder should be a some cases. Patients who do not respond to hormonal
multidisciplinary approach with continuous parental support. treatment or who subsequently relapse should receive
Autism is a lifelong disability which affects how people Vigabatrin. Vigabatrin is most effective in children with
communicate and interact with the world; therefore every op- tuberous sclerosis and is thus the preferred first-line
portunity should be taken to ensure patient centred care is treatment modality in this group.
delivered. Pyridoxal phosphate is the active form of pyridoxine, and
may be considered in the treatment of infantile spasms not
responding to first-line treatments. Options following hor-
Further reading monal therapy, vigabatrin, and pyridoxine include a wide
1) Melatonin in autistic spectrum disorders: meta-analysis Archives range of anticonvulsants (in particular sodium valporate,
of Disease in Childhood 2011;96:1172. benzodiazepines such as clonazepam or nitrazepam, top-
2) https://www.nice.org.uk/guidance/cg170/chapter/1- iramate, levetiracetam) or a ketogenic diet. Urgent discussion
Recommendations#specific-interventions-for-the-core- with a Neurologist is paramount in the first instance to initiate
features-of-autism treatment and oversee care.
3) https://www.autism.org.uk National Autistic Society
Case 3
Further reading
1) B
2) B 1) https://www.epilepsy.org.uk/info/syndromes/west-syndrome-
3) A infantile-spasms
Infantile spasms occur in the first year of life, typically be- 2) https://www.ncbi.nlm.nih.gov/pmc/articles/PMC6328947/
tween 3 and 8 months of age. To begin with, the spasms are 3) https://bestpractice.bmj.com/topics/en-gb/752/treatment-
usually infrequent and do not happen in clusters. Therefore it algorithm#patientGroup-0-0
is quite common for the diagnosis to be made quite late.

PAEDIATRICS AND CHILD HEALTH 30:7 273 Ó 2020 Elsevier Ltd. All rights reserved.
SELF-ASSESSMENT

Self assessment
Case 1 C. Commence High Frequency Oscillatory Ventilation (HFOV)
D. Discuss with microbiology
A previously fit and well 8 year old boy presented to his local
E. Repeat cultures
hospital following 9 days of fever, cough and coryza with
F. Repeat CXR
increased work of breathing. He was initiated on intravenous
Repeat CXR showed a right sided white out, with endo-
(IV) co-amoxiclav and clarithromycin, however his work of
tracheal tube in correct position. He was commenced on
breathing continued to worsen requiring escalation of respi-
HFOV. Antibiotics were changed to piperacillin/tazobactam.
ratory support to high flow nasal cannula oxygen and subse-
Aspiration of pleural fluid and initial bronchoalveolar lavage
quently, Continuous Positive Airway Pressure (CPAP). He
(BAL) were performed, which were negative for bacterial or-
failed to improve, therefore was intubated and ventilated and
ganisms. He was found to have positive serology from BAL for
transferred to the Paediatric Intensive Care Unit (PICU).
Mycoplasma pneumoniae, and was IgM positive for Epstein
Initial chest X-ray (CXR) (Figure 1) showed dense consol-
Barr Virus, but IgG negative.
idation of the right lung. His partial pressures of carbon di-
He was noted to have a swollen leg the following day.
oxide subsequently rose on the ventilator and he continued to
Ultrasound Doppler study showed a thrombus extending from
deteriorate clinically.
the right external iliac to the popliteal vein.
Question 2
Choose the next 3 investigations of choice:
A. D-dimer
B. Clotting
C. CTPA
D. Echocardiogram
E. Extended coagulation screen
F. MRA head
D-Dimer was markedly elevated at 6725 and he was
commenced on low molecular weight heparin (LMWH).
Computed Tomography Pulmonary Angiogram (CTPA) scan
(Figure 2) showed bilateral pulmonary emboli (PE), consoli-
dation and infarction with hepatomegaly, and an echocardio-
gram showed evidence of moderate pulmonary hypertension,
but otherwise normal cardiac anatomy.
Extended coagulation screen revealed positive titres for
Lupus Anticoagulant, with thrombotic screen showing low
Protein C levels, normal Protein S levels and low Anti-
thrombin, although we would note that these were performed

Figure 1 The initial chest X-ray.

Question 1
Choose 3 Options for next steps in management:
A. Check for endotracheal tube displacement and replace if
necessary
B. Increase pressures on the ventilator

Gemma Wilson MA(Cantab) MBBS MRCPCH, ST3 Paediatric Trainee,


Addenbrooke’s Hospital, Cambridge University Hospitals, UK.
Conflicts of interest: none declared.

Louise Selby BSc MBBS MRCPCH PGCert (MedEd), ST8 Paediatric


Trainee, Addenbrooke’s Hospital, Cambridge University Hospitals,
UK. Conflicts of interest: none declared. Figure 2 Computed tomography pulmonary angiogram.

PAEDIATRICS AND CHILD HEALTH 30:12 444 Ó 2020 Elsevier Ltd. All rights reserved.
SELF-ASSESSMENT

after the commencement of LMWH and are therefore less community acquired pneumonia range between 4 and 39%.
reliable. Relevant family history included a paternal uncle who Therefore despite the fact DVT is a rare complication in chil-
had an unprovoked deep vein thrombosis (DVT) but no other dren it is important to maintain vigilance for their occurrence
family history of pro-thrombotic conditions. and associated risk factors, including recent surgery or the
He was extubated after a week and continued his recovery presence of cardiolipin antibodies.
on the paediatric ward. Antibiotics were stopped and he
continued LMWH. References
1. Andrew M, David M, Adams M et al. Venous thromboem-
Question 3
bolic complications (VTE) in children: first analyses of the
Out of these risk factors he had for VTE, which is likely to have
Canadian registry of VTE. Blood, 1994: 83, 1251e1257.
been the major contributor?
2. Sauteur P, Unger W, Nadal D et al. Infection with and
A. Intensive care stay
carriage of Mycoplasma pneumoniae in Children. Front
B. Immobility
Microbiol 2016; 7: 329.
C. Mycoplasma pneumoniae
D. Inherited coagulation defect
Case 2
Answers
1. A, C, F A 3 year old boy was brought into hospital by ambulance as a
2. A, C, D trauma call after falling from a second storey window at home.
3. C The window had been opened for 10 minutes as the apartment
was hot, during which time the child had climbed up and out
Discussion of the window. He was unresponsive initially, but had a
Venous thrombosis is rare in children with an incidence of 0.7 fluctuating Glasgow Coma Scale (GCS) with paramedics with a
e1.0 per 100,000 and paediatricians often fail to consider the maximum score of 8/15 (E2 V2 M4). Parents immediately
possibility of thrombotic disease in their patients. There are a called an ambulance and arranged for a neighbour to look
few groups that are recognized as being at greater risk, after his sibling.
including children with malignancy, cardiac disorders Primary survey:
(following cardiac surgery or with moderate or giant coronary Concern for cervical spine injury.
aneurysms following Kawasaki disease) and those with a No visible haemorrhage.
central line. However, there are also several case reports of Airway: patent, Breathing: Equal air entry bilaterally.
infection-related problems. Infection with Mycoplasma pneu- Circulation: Radial pulse present, warm peripheries,
moniae has been linked to stroke, cardiac thrombus, splenic abdomen soft and non-tender, moving left leg well but
infarcts and DVTs with PE. In many of the case studies concern for right femoral fracture.
examined, the patients were found to have acquired activated Disability: GCS 7/15.
protein C resistance, or anti-phospholipid antibodies, such as Everything else: No wounds or lacerations, closed head.
anti-cardiolipin and lupus anticoagulant.
These antibodies cause a pro-thrombotic state in patients, Question 1
as seen in anti-phospholipid syndrome, so the question still What are your 3 immediate management strategies?
remains as to whether Mycoplasma pneumoniae is an addi- A) Start IV antibiotics
tional risk factor in a patient already predisposed to VTE, or B) Intubate and ventilate
whether the infection causes the rise in antibody titre. Snow- C) Immobilise C-spine
den et al. (1990) discovered incidental high titres of anti- D) 20ml/kg fluid bolus of 0.9% NaCl
phospholipid antibodies in the serum of a patient with Myco- E) IV paracetamol
plasma pneumoniae infection when studying the titres be- F) Computed Tomography (CT) head, neck, chest, abdomen
tween normal controls and patients with SLE. They then found and pelvis
that significantly more patients with Mycoplasma pneumoniae He underwent rapid sequence induction, was intubated
had increased anti-cardiolipin IgM and IgG compared with and ventilated and transferred for a CT head, C-spine, chest
other infections, with higher titres in those with more severe and abdomen, and X-ray of right femur and tibia/fibula. CT
infections and cold agglutinins. It would therefore seem that head and neck showed a normal alignment of the C-spine, no
Mycoplasma infection does act as a trigger for the develop- acute intracranial, skull or C-spine injury. There was left lung
ment of pro-thrombotic states. collapse due to the endotracheal tube (ETT) being in right
Rates of detection of Mycoplasma pneumoniae Polymerase main bronchus, and he had a minimally displaced right tibial
Chain Reaction (PCR) in children admitted to hospital with a fracture with fractures of 4th and 5th metatarsals.

PAEDIATRICS AND CHILD HEALTH 30:12 445 Ó 2020 Elsevier Ltd. All rights reserved.
SELF-ASSESSMENT

He was transferred to the Paediatric Intensive Care Unit academic, behavioural, and interpersonal functioning. Out-
(PICU) for insertion of an intracranial pressure (ICP) monitor comes following a TBI vary widely depending on age at injury,
and neuroprotective care management. injury severity, and time since injury, with variation in
methods of measuring outcomes complicating comparisons
Question 2
further. Babikian and Asarnow (2009) have attempted to
Which 3 of the following are features of neuroprotective care
provide a systematic summary of the literature, aiming to take
management?
these differences into consideration. Of note, non-accidental
A) Nurse at 30 degrees head elevation in midline position
head trauma patients were not included.
B) Nurse at 45 degrees head elevation in midline position
Severity of head injury is most commonly defined accord-
C) Maintain pCO2 4.5e5.0
ing to GCS on arrival: mild (GCS 13e15), moderate (GCS 9
D) Maintain pCO2 5.5e6.0
e12), and severe (GCS 3e8). Using this scale, the patient in
E) Maintain normothermia
question was classed as having a severe head injury as had a
F) Aim for hyperglycaemia
GCS of 7 on arrival.
He was placed on 70% maintenance IV fluids, was sedated
Age at injury was split into 0e5 years, and 6e16 years.
and ventilated overnight and his fracture was immobilized.
Methods of measuring neurocognitive outcome are similar
His ICP remained stable so he was extubated the following
within these groups, and by 6 years old, most children have
morning and later had his ICP monitor removed. He was self-
started formal education, affecting their development.
ventilating in air, his C-spine was cleared clinically and he had
When assessing neurocognitive impairments and recovery
no immediate neurological deficits. However, he remained
following a TBI, one can look at recovery in 3 time periods:
sleepy post-extubation. He began to eat and drink normally so
1. 0e5 months post injury; the post-acute phase, where most
IV fluids were stopped. He was stepped down to a general
functional impairments are seen.
paediatric ward, and neuro-observations were continued.
2. 6e23 months post injury; the period of time where most
He remained non-verbal for 2 days and had reduced
recovery is seen.
movement of his right arm with preferential use of his left arm
3. 24þ months post injury; the chronic phase, where most
(non-dominant) so was reviewed by the Paediatric Neurology
recovery has already taken place and functional deficits are
team. On examination, his tone was slightly lower in right
likely to be long lasting.
arm, with spontaneous antigravity movements and normal
‘Neurocognitive outcomes’ is a broad term which encom-
reflexes. It was difficult to assess power as he was uncooper-
passes the following aspects as defined by this meta-analysis:
ative, but he appeared to have bilateral reduction in power in
1. General intellectual functioning; including Full Scale Intel-
keeping with post-concussive head injury. X-ray of right arm
ligence Quotient (FSIQ), Verbal IQ and Performance IQ.
did not show any fracture, so Magnetic Resonance Imaging
2. Attention/executive functions; including working memory,
(MRI) of the brain, spine, neck vessels and brachial plexus
processing speed/reaction time, attention, fluency, inhibi-
was performed to determine whether there was a cause for his
tion and problem solving.
symptoms. MRI showed a small area of oedema in anterior
3. Memory; including verbal/visual immediate and delayed
superior part of left cerebellar hemisphere and adjacent to the
memory.
vermis, a small amount of linear sulcar/foliar haemorrhage
4. Visual perceptual/motor skills.
within the oedema. There was hyperintensity in the right
In summary of the results from the meta-analysis, there is
frontal matter in keeping with insertion of the ICP probe but
little or no impairment after a mild TBI which resolves over
no cause for his right sided motor deficit.
time back to baseline. There are deficits observed after a
He regained his usual level of speech six days following the
moderate TBI with recovery over time, but not quite back to
accident, with improvements in functioning of right arm and
the baseline of the controls. After a severe TBI there are sig-
was discharged home with follow up.
nificant impairments, which do improve over time but at a
Question 3 much slower rate so the difference between these children and
Which 2 safeguarding investigations are needed? controls actually increases over time.
A) Police questioning of parents However, what is important to highlight when looking at
B) Section 47 referral the literature for TBI in children, is that few studies have a
C) Inform social services and safeguarding team of incident universal or standardized approach to defining severity, time
D) Skeletal survey points post injury, age at injury, and methods for assessing
E) Ophthalmology review neurocognitive outcomes, with even fewer studies looking at
Answers outcomes in younger children specifically. In addition, with
1. B, C, F regards to mild TBI in particular, some studies show con-
2. A, C, E flicting evidence. This makes predicting outcomes for children
3. A, C with TBI more difficult.
In patients who have had a severe TBI such as the patient
Discussion in this case, we would expect to see significant impairments in
A major cause of disability associated with traumatic brain general intellectual functioning, some aspects of executive
injury (TBI) is acquired neurocognitive impairment affecting

PAEDIATRICS AND CHILD HEALTH 30:12 446 Ó 2020 Elsevier Ltd. All rights reserved.
SELF-ASSESSMENT

functioning and verbal memory throughout the recovery chest pain but had been short of breath. A chest X-ray (CXR)
phase. After 2 years there are likely to be significant deficits in was performed which showed a large pleural effusion. His
almost all neurocognitive domains, but longitudinal studies of observations were within the normal range and he was well in
patients with severe TBI show some recovery of these over himself.
time.
Question 1
There are no specific National Institute for Health and Care
What are the 2 next steps in management?
Excellence (NICE) guidelines for directing follow up in pae-
A) Insert chest drain
diatric TBIs, and most centres are led by their local policies.
B) USS chest
The Scottish Intercollegiate Guidelines Network (SIGN) have
C) Discuss with tertiary paediatric respiratory unit
advised a telephone/postal follow up for children following a
D) Needle aspiration of fluid and send for MC&S, protein,
mild TBI as there is some evidence that individual children
LDH, pH and cytology
may have disabilities. With regards to moderate and severe
E) Start IV antibiotics
TBI, all children should be followed up by a specialist multi-
disciplinary team to assess rehabilitation needs. Question 2
There are higher rates of secondary attention deficit hy- What is the most likely cause of his effusion?
peractivity disorder (SADHD) and other psychiatric disorders A) Haemothorax
following TBI in children however there are also higher rates B) Chylothorax
of pre-existing attention deficit hyperactive disorder (ADHD) C) Empyema
in children presenting with a TBI which may skew neuro- D) Malignant pleural effusion
cognitive outcomes in large studies. Some cases of ADHD may His case was discussed with a tertiary paediatric respiratory
have been undetected prior to TBI and diagnosed retrospec- team for input, he was started on oral antibiotics and trans-
tively. Younger children may have a prolonged hospital stay ferred for ongoing management. Given his recent thoracic
and recovery time following TBI that may contribute to surgery, chylothorax was the main differential diagnosis and
delayed acquisition of developmental milestones. It is there- his case was also discussed with the cardiothoracic team. The
fore essential that the developmental progress is monitored following day he had an ultrasound of his chest that showed a
following a TBI to monitor both direct and indirect outcomes large, simple effusion and a chest drain was inserted. CXR post
of injury. procedure showed significant improvement in effusion with
relief of symptoms.
References He developed a fever post-procedure, therefore blood cul-
1. Babikian T, Asarnow R. Neurocognitive outcomes and re- tures and a full blood count (FBC) were taken, and he
covery after pediatric TBI: meta-analytic review of the commenced IV antibiotics. He was not neutropenic and clini-
literature. Neuropsychology. 2009 May;23.283-296. doi: cally remained stable, therefore was converted to oral antibi-
10.1037/a0015268. Review. PubMed PMID: 19413443; otics. A repeat CXR showed further improvement of the
PubMed Central PMCID: PMC4064005.Pubmed Partial msc effusion, therefore he was discharged home to complete his
doi antibiotic treatment.
2. Scottish Intercollegiate Guidelines Network: Early man- Answers
agement of patients with a head injury https://www.sign. 1. B, C
ac.uk/assets/sign110.pdf 2. B

Case 3 Discussion
Aetiology: A chylothorax is a rare cause of pleural effusion in
A 15 year old boy presented to his local hospital with a children and is formed due to leakage of chyle from the
persistent cough. He had a past medical history of a localized thoracic duct within the chest. Causes include the following:
mediastinal paraganglioma recently diagnosed following a 1. Congenital malformations of the lymphatic systems, or
pulseless electrical activity (PEA) cardiac arrest in hospital. He associated with certain syndromes.
had initially presented with vomiting, chest pains, apnoeas, 2. Iatrogenic trauma such as post-thoracic surgeries or inva-
low saturations and reduced conscious level and had a sive diagnostic and therapeutic procedures.
cardiorespiratory arrest during intubation. He received 3. Non-iatrogenic trauma.
chemotherapy (carboplatin and etoposide), phenox- 4. High central venous pressure.
ybenzamidine (for blood pressure monitoring as a normeta- 5. Malignant infiltration.
drenaline producing tumour), and had a surgical resection two 6. Miscellaneous causes such as granulomatous infections,
months following diagnosis. staphylococcal discitis or Henoch-Schonlein Purpura.
Following his resection, he was an inpatient for 2 weeks for Symptoms: Presenting symptoms are usually those of a
blood pressure monitoring and had been receiving physio- pleural effusion, such as shortness of breath, cough or chest
therapy. He was re-admitted nineteen days post resection to pain. Rapidly accumulating chylothoraces can lead to hae-
his local hospital with a cough. He had not had any fevers or modynamic compromize and hypotension, cyanosis and

PAEDIATRICS AND CHILD HEALTH 30:12 447 Ó 2020 Elsevier Ltd. All rights reserved.
SELF-ASSESSMENT

significant respiratory distress. Longstanding or large chylo- Secondary management is prevention of recurrence by treating
thoraces can result in symptoms related to malnutrition or the underlying cause. Usually a child is given a 2e4 week trial
immunodeficiency due to loss of lymphocytes and hypo- on non-surgical therapies before the consideration of surgery.
gammaglobulinaemia, and fluid and electrolyte imbalances These include a fat-free diet with the addition of medium chain
due to fluid loss. triglycerides, and in more extreme cases, complete gut rest
If the cause of the chylothorax is traumatic, there is usually with the use of parenteral nutrition (taking care to match
a 2e10 day period between the trauma, and onset of the electrolytes, calories and volume to that of what is lost in the
effusion. Non-traumatic chylothoraces present more insidi- chyle). Alternatively, Octreotide (a synthetic somatostatin
ously, and congenital chylothoraces usually present with res- analogue) has been used with varying results, possibly by
piratory distress within the first 24 hours, with 75% presenting causing a reduction of lymphatic fluid production and/or flow.
within the first week. Reducing flow through the thoracic duct gives it time to heal.
Diagnosis: Diagnosis is initially by an opacity with a fluid If medical management fails to control the chylothorax,
level suggesting a pleural effusion on CXR, followed with surgical intervention is an option. The exact procedure de-
white, milky aspirates obtained following thoracocentesis. pends on identifying the location and site of the rupture of the
Differential diagnosis at this point would be a pseudochylo- thoracic duct. Procedures include thoracic duct ligation via
thorax, which is high in cholesterol rather than triglycerides, thoracoscopy, obliteration of the pleural space either chemi-
and empyema which is high in white blood cells. These are cally or surgically, or finally, placement of a pleuro-peritoneal
differentiated after centrifugation and determination of the shunt to drain chyle from the pleural space without losing the
triglyceride level, pleural fluid to serum cholesterol ratio, and fluid. Pleuro-peritoneal shunts are used when children have
demonstration of chylomicrons in the pleural fluid. chylothoraces that are refractory to treatment or in pre-term
Following diagnosis, further imaging of the lymphatic infants or foetuses.
system may be required such as CT scans, lymphangiography,
lymphoscintigraphy, and occasionally Magnetic Resonance References
Imaging (MRI) of the mediastinum. 1. Tutor JD. Chylothorax in infants and children. Pediatrics.
Management: Initial management is to relieve the pleural 2014 Apr;133:722-733. doi: 10.1542/peds.2013-2072. Epub
effusion causing respiratory symptoms by drainage. 2014 Mar 31. Review. PubMed PMID: 24685960.

PAEDIATRICS AND CHILD HEALTH 30:12 448 Ó 2020 Elsevier Ltd. All rights reserved.
SELF-ASSESSMENT

Self-assessment

Case 1 2. Following these results, what would be your next course


of action? Choose ONE of the options:
A 13 month-old boy was admitted with a 4 day history of fever
a) Perform an oral fluid challenge in view of ketosis and
and reduced oral intake. There was no history of abdominal
repeat urine dip later
pain, rashes, cough or coryzal symptoms. He was reported to
b) Site an intravenous cannula and give a 10% dextrose
have reduced wet nappies by his mother. He was previously fit
bolus immediately
and well but mum had concerns about him being a fussy eater
c) Send urine for urgent microscopy and culture
and felt that he was not gaining much weight. He was not on
d) Perform a blood gas, blood ketones and a BM
any regular medications.
e) Administer subcutaneous fast acting insulin e.g.
On examination, he is afebrile (temperature 36.7 C) with a
Novarapid
heart rate of 147/minute. He is tachypnoeic but auscultation of
f) Send urine for amino and organic acids
his chest does not reveal any signs. Cardiovascular and
g) Commence an insulin infusion
abdominal examinations are unremarkable. Examination of
h) Commence a dextrose infusion
his throat reveals bilateral enlarged tonsils with exudate and
Blood gas, blood glucose and ketones were performed. The
he has bilateral cervical lymphadenopathy on neck
result were as follows:
examination.
Glucose 40.3.
On reviewing his weight, he has dropped from the 50th
Blood ketones 7.3.
centile, on serial measurements taken in the first few weeks to
Blood Gas:
months of life, to below the 9th centile on admission.
pH7.14.
1. Given his presentation and examination findings, which
pCO2 2.5.
initial investigations would you like to perform? Choose
PO2 6.70.
ONE from the list below.
Base excess 22.5.
a) None. No investigations are required
Bicarbonate9.
b) Urine dip/Coeliac screen
Glucose>41.6.
c) FBC/Blood Culture/CRP
Lactate2.2.
d) Viral and Bacterial Throat Swab
The results of the remaining investigations were as follows:
e) U&E’s/Urine dip
White blood cell count3.9 x109/L.
f) TFT’s/Coeliac Screen
Haemoglobin123 g/L.
There was evidence of faltering growth and in line with
Lymphocytes2.0 x109/L.
NICE guidelines: ‘Faltering growth, recognition and manage-
Neutrophils1.4 x109/L.
ment’ a urine dip to exclude UTI and coeliac screen were
Platelets286 x109/L.
planned. Alongside these investigations, FBC, U&E’s, LFT’s
Sodium 124 mmol/L.
and Thyroid function were also performed to exclude any
Potassium4.8 mmol/L.
evidence of chronic illness that may be contributing to poor
Urea 5.7 mmol/L.
weight gain.
Creatinine 53 mmol/L.
The results of the urine dip showed;
Alanine Transaminase24 U/L.
pH 5.5.
Alkaline Phosphatase 369 U/L.
Ketones 4þ.
Bilirubin <3 mmol/L.
Glucose 2þ.
Albumin41 g/L.
Blood 1þ.
C Reactive Protein<5 mg/L.
Leucocytes neg.
Thyroid Stimulating Hormone 0.46 mU/L.
Nitrites neg.
T4 7.1 pmol/L.

Amy Alcock MBBCh BSc, Paediatric ST1, Morriston Hospital, Swansea, Wales, UK.

Amy Owen MBBCh BSc, F2, Morriston Hospital, Swansea, Wales, UK.

Sarah Knox MBBCh BSc, F2, Morriston Hospital, Swansea, Wales, UK.

Eliana Panayiotou MBBS BSc MRCPCH, Consultant Paediatrician, Morriston Hospital, Swansea, Wales, UK.

PAEDIATRICS AND CHILD HEALTH 31:1 55 Ó 2020 Elsevier Ltd. All rights reserved.
SELF-ASSESSMENT

Calcium (adjusted)2.48 mmol/L. was a compensatory mechanism to correct his metabolic


Phosphate 1.43 mmol/L. acidosis and was reflected in his blood gas which showed a
IgA0.42 g/L. pCO2 of 2.5.
Anti IgA TTG antibodies 0.7 U/ml. Following initiation on subcutaneous insulin, as per the
3. The blood gas is consistent with DKA. What complica- DKA protocol this child should have regular BMs and regular
tions should you monitor for? Choose THREE options U&Es to monitor response to treatment but also to monitor for
from below. complications such as hypoglycaemia and hypokalaemia. In-
a) None expected sulin promotes the entry of potassium into the liver and
b) Hypernatraemia muscles resulting in hypokalaemia. Neurological status of the
c) Hypokalaemia child should be observed and regular assessment for head-
d) Hypoglycaemia ache, agitation, irritability, drowsiness, bradycardia and hy-
e) Excessive thirst pertension should be undertaken to monitor for cerebral
f) Cerebral oedema oedema. If cerebral oedema is suspected then hypertonic sa-
g) Hyperkalaemia line (2.7% or 3%) or mannitol (20%) should be given before
h) Hypercalcaemia seeking specialist advice, undertaking a CT Head to exclude
other causes and considering anaesthetic input.

Answers
1. b Further reading
2. d
BMJ Best Practice Failure to Thrive - https://bestpractice.bmj.
3. c, d, f
com/topics/en-gb/747.
Assessment of faltering growth is made in relation to serial
BSPED Guideline on management of children and young
weight measurements taken across a time period. Weights
people under the age of 18-yrs with DKA https://www.sort.
should be plotted on age appropriate and gender specific WHO
nhs.uk/Media/Guidelines/BSPED-DKA-guideline-2020-
growth charts. Assessment of faltering growth should include
update.pdf.
a thorough history focussing on evidence of increased losses
NICE Guidance on Faltering Growth https://www.nice.org.
(excessive vomiting/diarrhoea), increased demands (chronic
uk/guidance/ng75/resources/faltering-growth-recognition-
illness such as Cystic Fibrosis), poor nutritional intake and
and-management-of-faltering-growth-in-children-pdf-
rarer endocrine causes such as hypothyroidism, growth hor-
1837635907525.
mone deficiency or genetic syndromes.
Given this patient’s urine dip was strongly positive for
Case 2
glucose and ketones, diabetes mellitus Type 1 should be
considered in the first instance and therefore it is appropriate A 3 year-old boy was referred from the GP with a 5 day history
to perform a blood gas, blood ketones and a BM. Excluding of an evolving, painful, vesicular rash on the left side of his
diabetic ketoacidosis (DKA) in a new diabetic is a priority. torso. The rash did not cross the midline. He had associated
Untreated DKA can be life-threating or lead to significant temperature spikes and reduced oral intake. His mother also
morbidity long-term. The blood gas result and serum glucose felt that he had reduced urine output. He had no preceding
indeed confirmed that the child was diabetic and in-fact in coryzal symptoms. He was up to date with all immunisations
DKA. He was managed with intravenous insulin and intrave- and the only past medical history of note was recurrent
nous fluids as per BSPED DKA guidance which has recently tonsillitis. He was not on any regular medications. He had no
been updated. previous dermatological conditions.
In this case his faltering growth was secondary to diabetes On examination his initial observations are: Temperature
mellitus. Both his coeliac screen and thyroid function were 38.5oC, pulse 122/minute, respiratory rate 28/minute, satu-
normal. In young children it is not uncommon for diabetes to rations of 96% in air. Systems examination is unremarkable.
present more atypically, as is seen here. There was no history He appears to be in significant discomfort, and prefers to lie on
of polydipsia or polyuria and the mention of picky eating and the unaffected right side. He appears withdrawn and quiet, but
poor weight gain was a passing comment from his mother. is alert and responsive. Capillary refill time is 2 seconds. The
This case illustrates the importance of measuring and plotting rash is limited to the left side of the torso and appears as
weights and heights in children. It was the alarming drop in shown in Figure 1. There is no evidence of rash/vesicles
weight from the 50th centile to below the 9th centile that lead to anywhere else on the body.
the urine dip being performed. Admission blood tests were as follows:
In addition, the importance of an ABCDE assessment of White blood cell count 8.0 x109/L.
children presenting acutely is highlighted. The clue in this case Haemoglobin 126 g/L.
was that the child was tachypnoeic with no other chest signs Platelets 286 x 109/L.
i.e no subcostal or intercostal recession/tracheal tug, wheeze Neutrophils 3.6 x 109/L.
or crepitations. Physiologically, his increase in respiratory rate Sodium 137 mmol/L.

PAEDIATRICS AND CHILD HEALTH 31:1 56 Ó 2020 Elsevier Ltd. All rights reserved.
SELF-ASSESSMENT

Potassium 4.4 mmol/L. are associated with an increased risk of secondary severe skin
Urea 4.9 mmol/L. and soft tissue infections in Varicella Zoster cases. NICE
Creatinine 31 mmol/L. Clinical Knowledge Summaries found no clinical evidence on
CRP <5 mg/L. the use of calamine lotion to relieve itch in chicken pox or
1. Given his likely diagnosis, what advice should be given shingles but anecdotally it can provide some symptomatic
to parents? Choose ONE of the following options. relief.
a) This is not an infectious rash and no isolation strategies IV acyclovir was also commenced in the first instance. Oral
are required. acyclovir has good bioavailability and the IV formulation is
b) Contact with pregnant women and those who are reserved for serious infections. Given the severity of the rash,
vulnerable or immunosuppressed should be avoided. the extent, the fact that new lesions were forming and the
c) Complications in children are common and should be patient’s inability to tolerate oral medicines due to vomiting,
expected to last long term. IV acyclovir was prescribed. Aciclovir has greatest benefit if
d) Family members will require antiviral prophylaxis. administered within 72 hrs of lesions appearing and acts to aid
healing and reduce the severity and duration of the condition.
2. The patient is experiencing significant discomfort.
When prescribing IV acyclovir, ensure the patient has
Which combination of treatment is most appropriate?
adequate hydration as it can precipitate renal toxicity. Given
a) Paracetamol/ibuprofen/oral morphine
that the patient had poor oral intake and reduced urine input,
b) Paracetamol/ibuprofen/calamine lotion
IV fluids were started and urine output monitored. In addition,
c) IV paracetamol/IV morphine
in view of the severity of the presentation and surrounding
d) Paracetamol/Calamine lotion/1% hydrocortisone cream
erythema, IV flucloxacillin was commenced to treat any
e) Regular paracetamol/oral morphine as required
superimposed bacterial infection. Both viral and bacterial skin
3 Given his presentation, what is the most appropriate swabs were taken.
treatment combination? Select ONE of the below options.
a) IV acyclovir, IV flucloxacillin, analgesia
b) PO acyclovir, PO flucloxacillin, cyclizine
c) IV aciclovir, IV flucloxacillin, IV fluids, analgesia
d) IV aciclovir, IV flucloxacillin, IV fluids
e) PO acyclovir, PO flucloxacillin, analgesia

Answers
1. b
2. e
3. c
Given the dermatomal distribution of the vesicular rash,
the presentation is most in keeping with a diagnosis of
Shingles.
It is important to advise that any patient with shingles
undertake regular hand hygiene measures and avoid sharing
towels etc. Shingles is considered infectious until all the ves-
icles have crusted over (usually 5e7 days after rash onset) and
therefore the child should not attend school until this time.
The parents should be counselled that a person who has not
had chickenpox or the varicella vaccine can catch chickenpox
from their child. It is particularly important to avoid contact
with pregnant women, anyone who is immunocompromised
and babies younger than 1 month of age.
The most challenging aspect of this patients’ care was the
degree of discomfort he experienced. Pain control is an
important part of the management of shingles and should be
initiated and escalated as per the pain ladder. In this case it
would be reasonable to start with paracetamol and add
morphine as required for breakthrough pain or prior to pro-
cedures such as when de-roofing vesicles to obtain swabs. As
outlined in NICE guidance, NSAIDs should be avoided as they Figure 1 Vesicular rash on the child’s thorax.

PAEDIATRICS AND CHILD HEALTH 31:1 57 Ó 2020 Elsevier Ltd. All rights reserved.
SELF-ASSESSMENT

Further reading Albumin 50 g/L.


Alkaline Phosphatase 511 U/L.
NICE Clinical Knowledge Summary on Shingles: https://cks.
Alkaline Transaminase 205 U/L.
nice.org.uk/topics/shingles/
Amylase 32 U/L.
The following day his pain continued and his jaundice
Case 3 worsened. His repeat bilirubin was 491. He underwent an USS
A 14-year-old boy with a background of hereditary spher- that showed a few small gallstones and a dilated common bile
ocytosis presented to the paediatric assessment unit with a 6- duct (CBD). His spleen had increased in size since his last USS
month history of recurrent abdominal pain which was more now measuring at 17cm (previously 15cm).
severe over the last 24 hours and localised to the epigastric 5. What is the most likely diagnosis? Select ONE of the
region. It was associated with vomiting and was worse after below options
eating. He had also developed jaundice over the previous 2 a) Hyper-haemolytic crisis
days. He reported no fevers. His urine was dark in colour but b) Pancreatitis
he had no change in the colour of his stools. c) Aplastic crisis
On examination, he is visibly jaundiced. His observations d) Obstructed gallstone in the common bile duct
are all within normal limits for his age. Respiratory and car- e) Viral hepatitis
diovascular examinations are unremarkable. His abdomen is
soft and he reports tenderness in the right upper quadrant
Answers
(RUQ) when palpated. His liver is palpable at 2cm and his
spleen is palpable at 6cm. He has no peripheral stigmata of 1. c, i
liver disease. 2. d, g
From the list below: 3. j
1. Which TWO investigations would you choose to assess 4. c
the synthetic function of the liver? 5. d
2. Which TWO investigations would you choose to assess Hereditary spherocytosis is the most common cause of
for possible haemolysis? inherited haemolysis in Northern Europe and clinical features
3. Given his RUQ discomfort, which other investigation include anaemia, jaundice and splenomegaly. The degree of
would be useful in his assessment? severity varies between patients and signs and symptoms may
a) AST and GGT fluctuate; with patients experiencing periods of ‘hyper-hae-
b) Bone profile molysis’ secondary to external insults such as viral infections.
c) Coagulation The most common complications from this condition include
d) Reticulocyte count the development of pigmented gallstones from chronic hae-
e) ESR molysis; and aplastic crisis from acute infection with
f) TFTs parvovirus.
g) FBC Coagulation screen and albumin are the most common
h) Split Bilirubin tests used to assess the synthetic function of the liver. Albu-
i) Albumin min is the most abundant protein produced by the liver and
j) Amylase liver synthesis of albumin tends to decrease in end stage liver
disease. The liver also produces the majority of clotting fac-
4. What imaging would be most appropriate in the first
tors therefore an increase in prothrombin time indicates po-
instance? Select ONE of the below options.
tential liver damage. FBC and reticulocyte count would both
a) Abdominal Xray
be useful in assessing haemolysis. A haemolytic anaemia
b) CT abdomen
would cause a drop in haemoglobin on an FBC and a raised
c) USS abdomen
reticulocyte count signifies increased bone marrow turnover
d) MRI abdomen
and therefore gives you an indication of the degree of
e) MRCP
haemolysis.
Preliminary investigations were as follows:
In any patient presenting with upper abdominal pain it is
White blood cell count 11.6 x 109/L.
important not to be too narrow with your differentials and
Haemoglobin 112 x g/L.
although unlikely an amylase is a quick and easy test to rule
Platelets 179 x 109/L.
out pancreatitis. In this case, his right upper quadrant pain,
MCV 75 fL.
jaundice and deranged LFTs with a disproportionately high
Sodium 138 mmol/L.
ALP and conjugated hyperbilirubinaemia points towards an
Potassium 4.0 mmol/L.
obstructive bilary picture. Therefore, the most appropriate
Urea 2.7 mmol/L.
initial radiological intervention would be an USS to look for
Creatinine 54 mmol/L.
bile duct dilatation and gallstones. An MRCP would be
CRP 15 mg/L.
useful following USS to further evaluate the location, extent
Bilirubin 338 (conjugated 276) mmol/L.

PAEDIATRICS AND CHILD HEALTH 31:1 58 Ó 2020 Elsevier Ltd. All rights reserved.
SELF-ASSESSMENT

and definitive cause of the obstruction. Abdominal X-rays with pallor, lethargy and fever and would cause a much lower
and CT scan are unlikely to be useful and come with radi- haemoglobin.
ation risks.
With the results of the USS indicating bile duct dilatation,
the mostly likely cause of his acute deterioration in liver Further reading
function is an obstructed gallstone. There may be a degree of
BMJ Best Practice, Hereditary spherocytosis https://
hyper-haemolysis due to the increasing size of his spleen and
bestpractice.bmj.com/topics/en-gb/1143/emergingtxs.
the recurrent abdominal pain however, his haemoglobin is
Bolton-Maggs P., Hereditary spherocytosis; new guidelines,
stable and therefore this is unlikely to be the cause of his acute
https://adc.bmj.com/content/89/9/809.
crisis. Similarly, an aplastic crisis would present differently

PAEDIATRICS AND CHILD HEALTH 31:1 59 Ó 2020 Elsevier Ltd. All rights reserved.
SELF-ASSESSMENT

Self assessment
Case 1 Case 2
A 5-year-old boy presents to the Emergency Department with A 16 years old boy presents to outpatients with weight loss,
an unsteady gait, difficulty using his left arm and facial very poor and selective appetite, poor energy levels, excess
asymmetry. His parents give a 3-day history of a mild non- tiredness and dizzy spells on standing for the last 6 months.
specific febrile illness. He has no significant past medical or His weight is 50 kg with a height of 180 cm. His BMI is 15.4
family history. He usually has good exercise tolerance. His kg/m, percentage weight for height ratio being 25% below ex-
development is normal and his immunisations are up to date. pected weight for age and gender. You consider anorexia nerv-
He is not on any medications. On examination, his BP is 110/ osa as a possibility. The patient informs you that he is extremely
65 mmHg and he is afebrile. His oxygen saturations are 99% worried due to weight loss and is also concerned about his dizzy
in room air. Height and weight are both on the 50th centile. He spells. He tells you that he also has some abdominal pains with
is noted to have a left hemiplegia, with a more pronounced frequent diarrhoea and occasional blood. He tells you that he is
weakness of his left upper limb. He also had a left sided facial not exercising and has lost about 5 kg in the last 6 months.
weakness with drooling. The remainder of his examination is On examination the patient is pale, he has no icterus, no
unremarkable. CT head without contrast is reported as oedema, no lymphadenopathy and no cyanosis. The abdomen
normal. is slightly distended, non tender and without any masses. The
heart rate is 60/minute and blood pressure is low. He shows
Question 1 some muscle weakness with loss of muscle bulk. The rest of
systemic examination is normal. Investigations performed at
What is the most likely diagnosis? Select ONE answer only. the GP surgery showed the following results:
A. Acute Disseminated EncephaloMyelitis (ADEM) Hb: 90 g/L, platelet 565 x 109/L; U&E: potassium and so-
B. Acute ischaemic stroke dium on lower end of normal; Liver function: bilirubin 30
C. Bacterial meningitis mmol/L, albumin 33 g/L, total protein 70 g/L, ALT 67 IU/L; A
D. Mitochondrial myopathy, Encephalopathy with 12 lead ECG shows sinus bradycardia and normal QTc.
E. Lactic acidosis and Stroke-like episodes (MELAS)
F. Todd’s paresis Question 1
A diffusion weighted MRI is performed next and is shown
in Figure 1. Which investigations are missing and you would like to be
done based on this presentation. (Choose TWO answers).
Question 2
Which of the following is NOT an associated risk factor?
Select ONE answer only.
1. Marfan syndrome
2. Methylmalonic acidaemia
3. Trisomy 21
4. Subacute necrotising encephalomyelopathy (Leigh’s
disease)
5. Neurofibromatosis Type 1

Question 3
Which ONE is the next most important step in
management?
1. Oxygen
2. Thrombolysis
3. Commence IV cefotaxime
4. Neurosurgical review
5. IV heparin

Peter Heinz State Exam Med Dr med FRCPCH Consultant Paediatrician,


Department of Paediatrics, Addenbrooke’s Hospital, Cambridge,
UK. Figure 1 Diffusion weighted MRI.

PAEDIATRICS AND CHILD HEALTH 31:2 89 Ó 2020 Elsevier Ltd. All rights reserved.
SELF-ASSESSMENT

A. C reactive protein (CRP) Preliminary investigations are as follows:


B. Bone profile Haemoglobin 12.8 g/dL
C. ESR (erythrocyte sedimentation rate) White blood cell count 9.6 x109/L
D. X ray abdomen Lymphocytes 1.28 x109/L
E. USS abdomen Neutrophils 7.48 x109/L
F. Anti TTG antibody Platelets 258 x109/L
G. Thyroid function test Sodium 142 mmol/L
Potassium 4.4 mmol/L
Question 2 Urea 3.4 mmol/L
Creatinine 63 mmol/L
If above patient presented with same history and exami-
ALT 12 U/L
nation, but bloods showed total protein of 54 g/litre with same
ALP 159 U/L
albumin along with normal platelet count, normal CRP and
Bilirubin 7 umol/L
ESR and the rest of bloods being the same. What possibilities
Albumin 36 g/L
would you consider? (Choose ONE answer).
CRP 16 mg/L
A. History may not be reliable
Urine dipstick for leucocytes, nitrites, blood, protein and
B. Constipation
beta HCG is negative.
C. Indeterminate IBD
Ultrasound scan of the abdomen revealed normal ovaries,
D. Coeliac disease
liver and spleen but an isolated area of bowel wall thickening,
approximately 6 mm in the region of the umbilicus, correlating
Question 3
to the sigmoid colon. (See Figure 2).
The patient gains more weight during feeding management At the time of her ultrasound the pain was settling down
than expected. What are the possible causes for this? (Choose and manageable with regular analgesia (paracetamol and
THREE answers). ibuprofen). It is felt the bowel wall swelling was most likely
A. Excessive drinking of water before weighing secondary to an acute viral illness and she is discharged home
B. Carrying weights secretly in clothes with instruction to attend her GP surgery if the pain recurred.
C. Fluid retention due to poor myocardial contractility and re- Three weeks later she re-presents to the Emergency
feeding syndrome Department with a one day history of sudden onset severe
D. Reduced renal perfusion and fluid retention abdominal pain, this time in the left iliac fossa and a mildly
tender left knee. There is no fever, vomiting, change in bowel
Question 4 habit or urinary symptoms.
Examination reveals a heart rate of 75, blood pressure
Which psychological symptoms are true in a patient with
110/72 mmHg, respiratory rate of 22 and oxygen saturations
anorexia nervosa. (Choose all that apply).
99% in air. Respiratory and cardiovascular examination is
A. Denial of the problem
normal. Abdominal examination shows a soft abdomen with
B. Fear of losing control of eating
left iliac fossa tenderness, but no rebound tenderness or
C. Problem with separation and independence
guarding. The pain is beginning to settle with oral paraceta-
D. Suicidal ideations
mol. Her left knee is mildly swollen but non-erythematous
E. Preoccupation with thoughts of food like cooking for
and has a good range of passive movement however she is
others
tender on active flexion.
Case 3
A 14 year old girl has been admitted with a 3 day history of
gradually worsening abdominal pain. It is localised to around
the umbilicus with no radiation. There has been no history of
any fever, urinary symptoms, changed bowel habit or vomiting.
Menstruation was regular between every 28 and 30 days and has
occurred 7 days previously. She is otherwise fit and well and
plays netball regularly. She is not on any regular medications
(including the oral contraceptive). She denies being sexually
active. Appetite is said to be good up until the occurrence of the
pain and there is no history of recent weight loss.
On examination she is normotensive (BP 108/75) with a
heart rate of 75. Respiratory and cardiovascular examination
are unremarkable. Abdominal examination reveals a soft
abdomen with periumbilical tenderness but no rebound,
guarding or organomegaly. Figure 2 Ultrasound scan of the abdomen.

PAEDIATRICS AND CHILD HEALTH 31:2 90 Ó 2020 Elsevier Ltd. All rights reserved.
SELF-ASSESSMENT

Question 1 can be difficult because of the wide variation in underlying


risk factors. In more than 50% of cases, a risk factor will be
Which other investigations would be helpful at this stage?
elicited in the history. Children are slightly more likely to have
Please select THREE from the list.
ischaemic strokes than haemorrhagic. Risk factors for acute
A. Urea and electrolytes
ischaemic stroke in children differ greatly and are more varied
B. Left knee X-ray
than those in adults. The most common cause of stroke in
C. Urinalysis (including beta HCG)
children is sickle cell disease. Other risk factors include
D. Abdominal X-ray
congenital heart disease, mitochondrial encephalopathies,
E. Repeat ultrasound scan of the abdomen
organic acidaemias and hereditary disorders of connective
F. CRP
tissue such as Ehlers Danlos syndrome (type IV). However,
G. Full blood count
even after extensive investigations, in up to 30% of children
H. Throat swab
with an acute ischaemic stroke, no cause is found. Following
I. Stool culture and microscopy.
an acute ischaemic stroke (AIS), 6% of children will die, 75%
Urinalysis reveals þ protein and þþ blood but is otherwise
may develop chronic neurological and psychological deficits
negative. Blood tests show electrolytes within normal range
and nearly 20% will have another stroke.
and a creatinine of 56 mmol/L. Full blood count and clotting
Figure 1 is an axial DWI (Diffusion weighted image)
screen are within normal range.
showing a small area of restricted diffusion in the right pos-
terior limb of the internal capsule. Urgent neuroimaging is
Question 2 indicated as evaluation of the lesion for the possibility of a
What is the most likely diagnosis? Please select ONE. penumbra may lead to a reduction in morbidity, mortality or
A. Non specific abdominal pain of childhood. recurrence of a stroke. Brain MRI and head and neck MRA
B. Inflammatory bowel disease (Angiography) as soon as possible is recommended for the
C. Appendicitis investigation of children presenting with clinical stroke. If a
D. Henoch Schonlein purpura cerebral sinus venous thrombosis is suspected an MRV
E. Juvenile idiopathic arthritis (Venography) is indicated. If a brain MRI not available within
A 5 day course of oral prednisolone is prescribed with in- 48 hours, CT is an acceptable initial alternative, especially to
structions to dipstick urine weekly at the GP surgery. 4 days rule out an intracranial haemorrhage. It should be noted that
after stopping steroids she represents to the emergency the initial CT can be normal and also cannot assess the pos-
department with worsening abdominal pain and four episodes terior circulation. Children should also be investigated for an
of vomiting. Urine dipstick shows þ protein and þþþ blood underlying prothrombotic tendency.
(currently menstruating). Blood pressure is 114/72 mmHg and Initial supportive management of children with acute
heart rate 68. Clinical examination reveals a purpuric rash ischaemic stroke includes maintaining normal oxygen satu-
over both lower limbs particularly on the ankles. rations, temperature, blood pressure and glucose levels.
Adequate hydration is also important. There is no evidence
that oxygen supplementation to those patients who are not
Question 3
hypoxaemic is beneficial. Prophylactic antiepileptic medica-
What would be your next steps in her management? Please tion in stroke is not indicated. In adult patients, stroke man-
select TWO. agement includes consideration of time critical thrombolysis
A. Once weekly blood pressure and urine monitoring at the and aspirin. In children, anticoagulation, in the form of low
GP surgery for the next 6 months molecular weight heparin infusion should be considered
B. 10 day course of amoxicillin. providing there is no haemorrhage on brain imaging. To date
C. Re-start oral prednisolone at a dose of 1 mg/kg (maximum there have been no evidence based studies supporting
dose 60 mg) for 7 days conclusively the use of thrombolysis and the treatment re-
D. Abdominal X ray mains controversial in children. Aspirin (5 mg/kg/day) is
E. Avoid NSAIDs recommended once there is radiological confirmation of an
F. Surgical review acute ischaemic stroke. Aspirin is contraindicated in patients
with evidence of intracranial haemorrhage on imaging and
those with sickle cell disease. In the latter group of patients
Answers
urgent exchange transfusion is recommended. Should the ex-
Case 1 change transfusion likely be delayed for more than four hours,
1 B a top up blood transfusion is advised, especially if the patient
2 C is severely anaemic.
3 E Acute disseminated encephalomyelitis (ADEM) is charac-
Stroke has been increasingly recognised in infants and chil- terised by a brief but widespread attack of inflammation in the
dren in recent years, the incidence having found to be around brain and spinal cord that damages myelin, the protective
2.3 to 13/100,000 and increasing. Diagnosis and management covering of nerve fibres. ADEM often follows viral or bacterial

PAEDIATRICS AND CHILD HEALTH 31:2 91 Ó 2020 Elsevier Ltd. All rights reserved.
SELF-ASSESSMENT

infections, or less often, vaccination for measles, mumps, or will be if the patient is not willing to disclose his or her mo-
rubella. Symptoms present rapidly, beginning with tives, symptoms and behaviours. If the patient does not reveal
encephalitis-like symptoms such as fever, fatigue, headache, his dieting, purging or exercising behaviour then it will be
nausea and vomiting and in the most severe cases, seizures difficult to make a diagnosis of anorexia nervosa (AN). The
and coma. ADEM typically damages white matter, leading to reasons may be psychological fears or social stigma. It could
neurological symptoms such as visual loss (due to inflam- also be due to little faith in the medical profession due to past
mation of the optic nerve) in one or both eyes, weakness even experiences. The individual may fail to reveal weight con-
to the point of paralysis, and difficulty coordinating voluntary trolling behaviours and collude with the doctor in pursuing
muscle movements. ADEM may be treated with high dose physical investigations to explain the weight loss. Sometimes,
steroids, or in resistant cases, plasmapheresis or clinical observation during a hospital assessment can enable
immunoglobulin. characteristic behaviours to be observed. The history may also
MELAS (mitochondrial encephalopathy, lactic acidosis and need to be taken from family members and close friends as
stroke-like episodes) manifests with muscle weakness and behaviours like food refusal, avoidance of caloric food, food
pain, recurrent headaches, loss of appetite, vomiting, and hiding behaviour and altered body image may be evident from
seizures. Most affected individuals experience stroke-like ep- their history.
isodes beginning before the age of 40. These episodes often If a patient gains more weight than expected then either the
involve hemiparesis, altered consciousness, vision abnormal- weight could be wrong or there could be possibility that it is a
ities, seizures, and severe headaches resembling migraines. result of manipulation. It is good practice to re-weigh patients
Repeated stroke-like episodes can progressively damage the on the same scale, preferably in their underwear or a gown,
brain, leading to vision loss, problems with movement, and a before eating or drinking. The subsequent weights should be
loss of intellectual function (dementia). done similarly, so that weight measurements are comparable.
Some patients drink a lot of water in order to falsify their
Further reading
weights. Their input and output should be supervised strictly.
Stroke in childhood: Clinical guideline for diagnosis, man-
Whatever the weight is, one must not show their opinion and
agement and rehabilitation, accessed October 2020, RCPCH:
stay neutral.
https://www.rcpch.ac.uk/resources/stroke-childhood-clin-
One must be aware of other reasons a patient could show
ical-guideline-diagnosis-management-rehabilitation
more weight gain. Re-feeding syndrome, with fluid retention is
Case 2 an important and life threatening reason to be aware of. Some
1 A&C re-feeding oedema is normal so long as other clinical and
2 A biochemical parameters are stable.
3 A, B & C AN develops over a long period of time. Food related
4 All apply behaviour becomes stricter over a period of time and espe-
There is a possibility that the patient has inflammatory bowel cially at a point when life changes bring fear and insecurity.
disease (IBD). The pointers are in the history of frequent Such restrictions often are reported as generating a sense of
diarrhoea with occasional blood and abdominal pain. The euphoria, at being in control of one’s weight, of feeling su-
bloods show some inflammatory response. The platelet count perior to others who might be struggling to manage theirs, and
is raised and also albumin is also slightly low, bilirubin and of increased confidence and enhanced appearance. Patients
ALT are abnormal. Total protein is 70 g/litre with albumin of report that it is not unusual to feel moments of intense clarity,
only 33 g/litre. This shows that there is disproportionately insight and of feeling more alive than ever. It is also not un-
raised globulin fraction and this again points towards some usual to feel considerable physical energy, which may be
inflammatory process. CRP and ESR will help in establishing experienced as though activity could be sustained for many
the presence of a significant inflammatory response and will hours without food or sleep at least until the food restriction
make the diagnosis of anorexia nervosa less likely. Other in- starts to trigger physical and mental side effects. All this may
vestigations that may be needed are stool test, faecal calpro- serve to reinforce a person’s negative behaviours around food.
tectin and further discussion with a paediatric Food increasingly comes to dominate thoughts, feelings and
gastroenterologist regarding referral and if contrast imaging actions. Self-hatred about appearance, feelings of being
studies are needed before intestinal biopsy. bloated and fat and frequent physical pain are common con-
Although coeliac disease could be present, blood in the sequences of these behaviours. Starting to eat can worsen
stool would be unusual. Blood can be present in constipation, these feelings.
but a history of weight loss and such a low percentage WFH
Further reading
ratio points against this. Indeterminate inflammatory bowel
Eating disorders: recognition and treatment. NICE guideline
disease (IBD) is a diagnosis made after no defining features in
[NG69] Published date: 23 May 2017. Accessed October 2020:
the biopsy/histopathology sample despite features of inflam-
https://www.nice.org.uk/guidance/ng69
mation. Always carefully consider the differential diagnosis
based on history and examination. Case 3
It is important to remember that diagnosis is based on a 1 A, C and E
valid and reliable history from the patient. The main difficulty 2 D

PAEDIATRICS AND CHILD HEALTH 31:2 92 Ó 2020 Elsevier Ltd. All rights reserved.
SELF-ASSESSMENT

3 A&C renal manifestations of HSP are usually self-limiting but a


The combination of abdominal pain and swollen joints should short course of steroids (like that used in an acute exacerba-
lead to the differential diagnosis of Henoch-Schonlein Purpura tion of asthma) may be beneficial. The gastrointestinal
(HSP). It is possible in this case the thickening of the bowel symptoms in this case did resolve after the second short
wall seen on initial ultrasound happened as a result of course of prednisolone. This patient did not go on to develop
resolved intussusception which had been the cause of the any signs of renal disease and has since been discharged from
initial abdominal pain. outpatient follow up.
HSP is the most common form of small vessel vasculitis in
Further reading
children. Prognosis of HSP without renal disease is usually
Bowman P, Quinn M. Should steroids be used to treat
good as this is the main long term complication. Gastrointes-
abdominal pain caused by Henoch Schonlein purpura? Arch
tinal complications can occur in up to 80% of patients with a
Dis Child 2012; 97: 999e1000.
spectrum of severity from mild abdominal pain to gastroin-
Haroon M. Should children with Henoch Schonlein pur-
testinal bleeding and intussusception. Some studies have
pura and abdominal pain be treated with steroids? Arch Dis
shown that children treated with steroids have faster resolu-
Child 2005; 90: 1196e98.
tion of their abdominal pain (as measured by clinical exami-
Ronkainen J, Koskimies O et al. Early prednisolone therapy
nation and symptom diaries).
in Henoch Schonlein purpura. A randomised double blind
However it is possible that giving steroids may mask the
placebo controlled trial. J Pediatrics 2006; 149: 241e47.
more serious symptoms of intussusception and gastrointes-
tinal perforation as well as increasing blood pressure. Extra

PAEDIATRICS AND CHILD HEALTH 31:2 93 Ó 2020 Elsevier Ltd. All rights reserved.
SELF-ASSESSMENT

Self-assessment
Case 1 2 Which of the following investigations are most important in
excluding the differential diagnoses? Please select TWO
A 14 year old boy was brought in by ambulance to the
from the list:
Emergency Department with multiple episodes of vomiting
A) Blood alcohol level
over the last hour. He was unable to give a coherent history
B) FBC and CRP
due to altered consciousness and had been found with alcohol,
C) Tryptase level
confectionery and an out of date salbutamol inhaler on his
D) CXR
person. On examination he was breathing spontaneously with
E) Paracetamol and salicylate level
a soft inspiratory stridor and equal air entry on auscultation.
F) U&E
Oxygen saturations were 95% on room air with a respiratory
A set of venous bloods were sent. Preliminary blood results
rate of 16/minute. Cardiovascular examination demonstrated
were as follows:
a tachycardia (heart rate 115/bpm) and a thready pulse, and
his blood pressure was 105/61 mmHg. Abdominal examina-
tion demonstrated mild central tenderness with no guarding. Haemoglobin 132 g/l
Further exposure did not reveal any obvious skin changes or White blood 3.9  109/l
injuries however his skin felt a little clammy. His pupils were cell count
equal and reactive to light and there was no gross motor Lymphocytes 1.31  109/l
deficit. Neutrophils 1.86  109/l
A venous blood gas demonstrated: Platelets 326  109/l
Sodium 138 mmol/l
Potassium 4.6 mmol/l
pH 7.39
Urea 5.1 mmol/l
pCO2 5.8
Creatinine 78 mmol/l
pO2 5.3
CRP <2 mg/l
HCO3 25
Paracetamol <5 mg/l
BE -1
Salicylates <2 mg/l
Glucose 5.3 mmol/l
Blood 98 mg/100
alcohol dL (Severe
intoxication
250e450
mg/dL)
Point of care capillary ketones: 0.2 mmol/l
1 Of these differential diagnoses which is the most likely
diagnosis? Choose THREE:
A) Acute exacerbation of asthma After 45 minutes of observation he became hypotensive
B) Alcohol intoxication (BP 86/52). He becomes audibly wheezy and his oxygen sat-
C) Diabetic ketoacidosis urations dropped to 91% on room air.
D) Anaphylaxis 3 Which is the correct order of the following management
E) Overdose steps, assuming there is no delay. Please select ONE:
F) Infective gastroenteritis A) Sit upright, give oxygen and IV fluid bolus
B) Assess and protect airway, give high flow oxygen, give
IM adrenaline, give IV fluids and IV steroids
Eva Wooding BMBS, ACF ST2 Paediatrics, Royal Devon and C) Assess airway, give high flow oxygen and nebulised
Exeter Hospital, Exeter, UK. Conflicts of interest: none declared. salbutamol 5mg, and IV fluid bolus
D) Give IM adrenaline, assess and protect airway, give high
flow O2, give IV fluids and IV steroids
Emily Chesshyre BSc, BMBS, MRCPCH, DipTropMed, PGDip Paediatric
E) Give nebulised adrenaline, assess and protect airway,
infectious diseases, Paediatric Infectious Diseases Consultant,
give high flow O2, give IV fluids and IV steroids
Royal Devon and Exeter Hospital, Exeter, UK. Conflicts of interest:
none declared.
Case 2
ˇ
Sian Ludman MBBCh, DRCOG, MRCPCH, MSc (Allergy), European Dip A 9 month old baby is referred to the paediatric assessment
Allergy, Consultant Paediatric Allergist, Royal Devon and Exeter unit with extensive eczema. She had an episode of pyrexia
Hospital, Exeter, UK. Conflicts of interest: none declared. yesterday evening (temperature 38.3 C) which prompted the

PAEDIATRICS AND CHILD HEALTH 31:4 167 Ó 2021 Elsevier Ltd. All rights reserved.
SELF-ASSESSMENT

GP consultation. On arrival to the unit her weight is 7.05 kg Case 3


(10th centile). She was born at term weighing 3.235 kg (50th
A 12 year old girl is admitted to the paediatric resuscitation
centile) after an uncomplicated pregnancy and delivery. She is
area in the emergency department with a fever, diarrhoea and
breastfed and started weaning at 6 months old, but breast-
malaise. On admission her temperature is 38.7 C, she is
feeding has been challenging as she exhibits aversive feeding
tachycardic with a heart rate of 150/bpm, a respiratory rate of
behaviour, including refusing food or drink in spite of
20/bpm and oxygen saturations of 97% on room air. On ex-
appearing thirsty/hungry, pulling away and arching back
amination she feels cold peripherally and has a central capil-
during feeds and faltering growth.
lary refill time of 3e4 seconds. She has no underlying medical
On examination she is alert and active, but notably is un-
conditions and is up to date with her vaccinations. Her mother
settled. She has a mild pyrexia (37.7 C) and her other obser-
states that she had chickenpox recently, and you note ery-
vations are within normal limits. On examination of her skin
thema around a scabbed chickenpox lesion on the lateral
she has 85% coverage with florid, weeping eczema, with dried
aspect of her left thigh. The lesion is very sore. She has been
blood present. She is noted to have an excoriated nappy rash
taking paracetamol to manage pyrexia and discomfort. You
and additional punched out lesions on her left lower abdomen.
take bloods for blood gas, blood culture, full blood count,
Steroid creams were previously trialled with some mild benefit
renal profile, liver function tests, CRP and coagulation screen.
but were discontinued due to parental concerns about growth
You treat her with 20 ml/kg IV fluid bolus and start intrave-
stunting effects.
nous (IV) ceftriaxone and clindamycin.
1 When you are taking a history which are the most important
1 In this scenario which life threatening differential diagnoses
questions to ask? Choose TWO from the following:
need to be immediately considered? Choose TWO:
A) Food triggers and dietary manipulation
A) Toxic shock syndrome
B) Contact triggers including soap and detergent use
B) Impetigo
C) Interference with sleep
C) Gianotti-Crosti syndrome
D) Gastrointestinal symptoms and growth
D) Necrotising fasciitis
E) Pets in home
E) Dermatitis herpetiformis
F) Hayfever symptoms
F) Skin abscess
She was admitted to the ward and commenced on oral
G) Disseminated varicella infection
flucloxacillin, aciclovir, clobetasone butyrate ointment (a
H) Ibuprofen allergy
moderate strength topical corticosteroid) and emollients.
In response to the 20ml/kg fluid bolus her perfusion im-
During admission nursing staff noted that she had frequent
proves. Her heart rate is 130/min and capillary refill time is <2
dark green, offensive, loose bowel motions and she screamed
seconds. Her blood pressure is 108/64 mmHg. However, she is
during and after breastfeeds. Her eczema did not improve on
complaining of increasingly severe pain in her left thigh. The
treatment, although the exudate decreased and the child now
lesion on her thigh appears more erythematous and develops a
remains apyrexial. An HSV skin swab result was returned as
blackened centre. She requires oral morphine to manage her
not detected, thus acyclovir was discontinued. A skin swab
pain and her temperature is noted to be 38.5 C.
sent for microscopy, culture and sensitivity returned with a
2 What is your immediate management? (select the most
heavy growth of methicillin sensitive Staphylococcus aureus
appropriate)
(MSSA).
A) Urgent surgical review for surgical debridement
2 What next management steps would you choose? Choose
B) Add in vancomycin to the antibiotic regime
TWO:
C) Give a further 20 ml/kg bolus of normal saline
A) Change oral flucloxacillin to IV flucloxacillin
D) Add in ibuprofen to her analgesia
B) Change oral flucloxacillin to oral co-amoxiclav
She is transferred to theatre for an emergency debridement
C) Restart Acyclovir
and managed post-operatively on the ICU. Histology shows
D) Take a peripheral blood culture
avascular necrotic tissue with Gram-positive cocci in chains.
E) Review dietary intake of mother and child
3 What is the most likely causative organism?
F) Step up strength of topical corticosteroid to a strong
A) Staphylococcus aureus
steroid
B) Neisseria meningitidis
G) Send stool sample for culture and virology
C) Streptococcus pyogenes
H) Take venous sample for HSV PCR
D) Haemophilus influenza
3 What is the most likely cause for her continued poorly E) Streptococcus pneumoniae
controlled eczema? Her antibiotics were rationalised in line with positive blood
A) Cows’ milk protein allergy cultures and she is treated with a course of IV benzylpenicillin
B) Eczema herpeticum then oral amoxicillin.
C) Antibiotic resistant infected eczema
D) Zinc deficiency
E) Coeliac disease

PAEDIATRICS AND CHILD HEALTH 31:4 168 Ó 2021 Elsevier Ltd. All rights reserved.
SELF-ASSESSMENT

Answers conditions mimic the effects of alcohol ingestion and the smell
of alcohol on the breath is not a reliable marker. One example
Case 1
is diabetic ketoacidosis, which can be excluded by the normal
1) B, D and E
blood glucose and ketones. Where alcohol ingestion is estab-
2) A and E
lished in young children, with a blood alcohol level in excess
3) D
of 50mg/dL there is an associated risk of hypoglycaemia
The expiratory wheeze and stridor alongside abdominal pain,
which must be assessed. Should treatment for alcohol induced
repetitive vomiting, tachycardia, clamminess and a thready
hypoglycaemia be required it is worth noting that this is re-
pulse are suggestive of anaphylaxis. The addition of confec-
fractory to Glucagon management.
tionery in the history would point towards this as chocolates
Once the diagnosis of Anaphylaxis is suspected the APLS
and bakery goods are a frequently occurring area of risk for
algorithm must be followed. This is detailed in the new
food allergic patients. It is important to note that 10% of pa-
Resuscitation Council flowchart for anaphylaxis in the vacci-
tients in anaphylaxis will not have skin changes such as ur-
nation setting, which is an early update on the 2021 anaphy-
ticaria and angioedema, repetitive vomiting after known
laxis guideline changes in light of Covid-19 vaccination
ingestion is also a sign of anaphylaxis. Anaphylaxis is a
reactions. The first priority is to give adrenaline, the algorithm
constellation of non-specific symptoms which does not have
states to give IM adrenaline in the first instance via auto-
single test to rule it out or in at the point of history. In children
injector or needle and syringe, even before establishing an
with suspected anaphylaxis giving IM adrenaline will not
airway; however these can be done concurrently. The key
cause harm but may save a life. Infective gastroenteritis may
point is that IM adrenaline should not be delayed for any
have been a cause for the repeated vomiting but the abdom-
reason. Once these 2 steps have been taken, high flow oxygen
inal pain in association with stridor and thready pulse,
should be given alongside IV hydrocortisone and fluids. Tak-
alongside normal inflammatory markers and white cell count
ing a tryptase level is not an immediate priority in this case,
are not consistent with an infective picture.
but should be done when practicable. The Resuscitation
The assumption is that this young person has a known
Council advise serum Tryptase sampling to be taken at initial
allergy, but he is not carrying an adrenaline autoinjector or
resuscitation, 1e2 hours after onset and after 24 hours/at
wearing a medical alert bracelet. His salbutamol inhaler is out
follow up. Whilst this can be done, it is not likely to be raised
of date and he is engaging in high risk behaviours. A history of
in food anaphylaxis, therefore is not one of the key tests given
poorly controlled asthma is a risk factor for severe reaction in
in question 2, as a normal tryptase does not exclude
anaphylaxis, making good control vital. It is recognised that
anaphylaxis.
adolescents face particular challenges in managing their
Anaphylaxis, including but not limited to peer pressure and Case 2
reduced compliance. Adolescent specialist allergy clinics 1) A and D
focussing on education and national campaigns, such as the 2) E and F
Anaphylaxis Campaign’s 3) A
#TakeTheKit education video help highlight the importance The initial diagnosis is one of infected eczema in an irritable
of carrying one’s adrenaline autoinjector and are a wealth of infant with faltering growth parameters, having nearly crossed
relevant resources for patients and families. 2 centiles since birth, and feeding aversion. It is important to
In a large population based study of young people in the UK exclude infective differential diagnoses including Herpes
aged 10e24 years paracetamol was found to be the most Simplex Virus in your initial management, which requires
common drug responsible for poisoning episodes, followed by urgent identification and treatment with Acyclovir. The
ethanol and then NSAIDs including aspirin. Here, an overdose Acyclovir was discontinued once an HSV skin swab returned
is possible in the context of reduced consciousness and poor as not detected. Acyclovir should be continued in the case of
history. The normal electrolytes, salicylate, paracetamol levels strong clinical suspicion, and a serum HSV PCR is highly
and blood gas on balance makes overdose less likely that specific.
alternative diagnoses, but exclusion by history would be NICE guidelines on atopic eczema suggest that the most
preferable. important elements to be included in the history are listed in
Whilst the blood alcohol levels are raised, they are not Box 1.
sufficiently raised to be suggestive of alcohol toxicity but are Whilst all mentioned areas of the history in Question 1 are
likely to be the cause of the patient’s incoherence and reduced important, it is the identification of food triggers and gastro-
consciousness in the first instance. The literature suggests the intestinal symptoms that are most pertinent in this case, where
spectrum of blood alcohol levels are as shown in Table 1. there is a strong clinical suspicion of eczema refractory to
Where there is a concern of toxic levels of alcohol ingestion standard therapies due to concurrent Cows’ Milk Protein Al-
TOXBASE offers guidance on calculating the volume of lergy (CMPA). This is why reviewing the dietary intake of
ethanol in order to establish whether a minimum period of 4 mother and child is important for a breastfed infant with se-
hours observation is indicated. vere eczema, as in Question 2.
It is important to keep differential diagnoses broad when Atopic eczema is diagnosed in children with pruritic skin in
considering alcohol intoxication. A number of other addition to 3 or more of the following features: active and

PAEDIATRICS AND CHILD HEALTH 31:4 169 Ó 2021 Elsevier Ltd. All rights reserved.
SELF-ASSESSMENT

Box 1. NICE guidelines for key elements to include in the treatment response. For measuring severity, the Patient-
history for patients presenting with atopic eczema Oriented Eczema Measure (POEM) is a validated scale rec-
ommended by NICE. The scores are allocated based on fre-
C Time of onset, pattern and severity
quency of symptoms occurring from 0 (no days) to 4 (every
C Response to previous and current treatments
day). A score of 0e2 is clear or almost clear, a score of 17e24
C Possible trigger factors (irritant and allergic)
is severe and a score of 25e28 is very severe.
C The impact of the condition on children and their parents or
The mainstay of eczema management is emollients and
carers
topical corticosteroids in a stepped treatment regimen adapted
C Dietary history including any dietary manipulation
for the severity of the condition. In the first instance atopic
C Growth and development
eczema can be managed with mild potency topical cortico-
C Personal and family history of atopic diseases
steroids and emollients only. The choice of emollient should
be guided by level of dryness, and child or carer preference.
Adequate emollient use works symbiotically with topical ste-
roids and has been shown to minimise the need for steroids.
For moderate eczema, the next step is to escalate to moderate
visible, or historic dermatitis affecting the flexures (or cheeks potency topical corticosteroids and consider the use of topical
and/or extensor surfaces in infants under 18 months of age); calcineurin inhibitors in children over 2 years, and bandages.
dry skin in the last 12 months; current or previous asthma or Treatment can be stepped up for flares and should be
allergic rhinitis, or atopy in a first degree relative for children continued for 48 hours after subsidence of symptoms in a
under 4 years; onset before 2 years of age for children being “burst regime” of around 5e7 days, and emollient therapy
assessed after 4 years of age. Different distributions and pat- should continue after resolution of flare ups. In our case,
terns of lesions may also be seen in patients of Asian, black escalating the strength of topical corticosteroid would be an
Caribbean or black African descent, including lesions affecting appropriate first step, as in Question 2.
the extensor surfaces in older children, discoid or follicular The potency of topical steroid preparations is outlined in
patterns. Skin Deep is an excellent education resource detail- the BNFc, including all proprietary names. An overview of the
ing skin signs on a range of skin tones. generic formulations and most common proprietary formula-
Potential triggers should be excluded where possible in the tion is listed in Table 2, care must be taken with the similarity
first instance. These may include irritants such as soaps or in names of clobetasone and clobetasol.
detergents, contact allergens, food allergens, inhalant aller- Very potent steroids should be prescribed with specialist
gens or infections. However, professionals should consider advice. Further advice on topical steroids including strength of
food allergy in children with moderate or severe atopic eczema preparation, application guidance and volumes of topical ste-
which has not responded well to optimum management, roid required by body part as measured in Finger Tip Units
especially if associated with faltering growth or features of gut (FTU) is provided by the National Eczema Society.
dysmotility, such as colic, vomiting or altered bowel habit. Steroid phobia amongst carers of children with atopic
A holistic approach to assessing the severity and impact of eczema is a well-recognised phenomenon and reduces optimal
atopic eczema should be taken, considering the effect on management. The side effects of topical corticosteroids are
quality of life, everyday activities, sleep and psychosocial widely known, but accurate incidence data is limited. The
wellbeing. Healthcare professionals should consider employ- most common reported local side effects of topical cortico-
ing objective measures of severity, quality of life and steroid use include skin atrophy, striae, burning, itching,

Blood alcohol levels and clinical features

Level of intoxication Clinical features Blood ethanol concentration


Mild Disinhibition, excitation, emotional lability, 20e50 mg/dL
sociable, talkative, euphoria (4.34e10.8 mmol/L)
Mild Reduced reaction time, diminished judgement, 50e100 mg/dL
fine motor incoordination, dysarthria, nausea (10.8e21.7 mmol/L)
Moderate Blurred vision, violence, disorientation, confusion, 100e200 mg/dL
ataxia, vasodilation, stupor, vomiting, sweating (21.7e43.4 mmol/L)
Severe Diplopia, marked incoordination, coma, hypothermia, 250e450 mg/dL
hypoglycaemia and convulsions (43.9e98 mmol/L)
Potentially fatal Respiratory depression, hypotension, loss of protective Over 450 mg/dL
airway reflexes, hypothermia, incontinence, coma, (>98 mmol/L)
hypoglycaemia and subsequent convulsions. Potentially death.
Table 1

PAEDIATRICS AND CHILD HEALTH 31:4 170 Ó 2021 Elsevier Ltd. All rights reserved.
SELF-ASSESSMENT

Topical corticosteroid preparations categorised by potency

Topical steroids Common brand names


Mild Hydrocortisone
Moderate Clobetasone Eumovate
Betametasone Betnovate-RD
Potent Betametasone Betnovate
Mometasone Elocon
Very potent Clobetasol Dermovate

Table 2

folliculitis, acne-like eruptions and telangiectasia. The pres- the context of a clinically stable picture and where oral anti-
ence of side effects appears to be associated more commonly biotics can be adequately taken and absorbed. Nor is it
with higher potency formulations. Children, and in particular necessary to convert to oral co-amoxiclav as her skin swab is
infants, maybe more susceptible to side effects. The BNFc sensitive to flucloxacillin and the infective element is
cautions, however, against undertreatment of eczema due to improving.
these concerns due to the myriad of benefits to improved The vast majority of patients with atopic eczema will be
eczema control. heavily colonised with S. aureus as part of their skin micro-
It is known that topical corticosteroid use results in a biota. A positive skin swab does not always require treatment
certain amount of systemic exposure, the extent of which is and should be taken into account as part of the wider clinical
determined by a number of factors, including dose, duration picture in a well child. Decolonisation protocols for S. aureus
and level of skin inflammation. Systemic effects of topical generally include nasal Mupirocin 2% wash 3 times daily and
corticosteroids are glucocorticoid-related affecting the hypo- Chlorhexidine or Octenidine wash daily for 5 days. However,
thalamic pituitary axis leading to adrenal suppression and decolonisation may be only milder superior to good hygiene
insufficiency, Cushing’s syndrome and impaired growth tra- measures and compliance to decolonisation may be poor.
jectory. The effect of topical steroids on growth is mixed in the Treating all household members is associated with a reduction
literature, but there are some reports of growth stunting so any in skin and soft tissue infections due to S. aureus.
parental concerns should be addressed. Children with atopic Dietary factors can be implicated in atopic eczema. The
eczema who are not growing at the expected rate, as evi- most common of these are milk, egg, wheat, soy and peanut,
denced by how they plot on UK growth charts, should be which collectively are responsible for 75% of food-induced
referred for specialist growth advice in line with NICE guid- atopic eczema. When taking a dietary history, you should
ance as this may be due to under or over treatment of the consider all products eaten in the last 4 weeks. Where the
eczema or an unrelated condition. infant is breastfed, the mother’s diet should also be monitored
The use of “burst therapy” and maintenance on “weekend over this period. The Breastfeeding Network offers a pathway
therapy” is shown to reduce overall steroid use and will gain for diagnosis of CMPA in breastfed infants. Consider food-
better control of eczema rather than intermittent, scattered or induced atopic eczema in children who have previously
inadequate volume frequent use. The use of topical calci- reacted to foodstuffs or in young children or infants whose
neurin inhibitors should be discussed in steroid dependent moderate to severe eczema has not responded to optimum
eczema as a safe alternative. A shared and transparent man- management, especially where this is associated with gut
agement plan is vital. NICE guidelines recommend educating dysmotility, manifesting as colic, vomiting, altered bowel
children and their carers about atopic eczema and its man- habit, or faltering growth.
agement at every consultation, reinforcing treatment plans CMPA can be IgE or non-IgE mediated. Non-IgE mediated
with verbal and written information and practical demon- CMPA does not need referring to paediatric allergy clinic for
strations of treatment application. skin prick testing, but once confirmed after exclusion and
In our case the infant presented with florid, weeping confirmatory reintroduction, the infant should be referred to
eczema and excoriated lesions. She was commenced on flu- the paediatric dietitians for input on a balanced diet and
cloxacillin and responded to this, becoming apyrexial and with guidance on re-introduction when appropriate. Non-IgE CMPA
a noted reduction in exudate. Her skin swab returned with gastrointestinal symptoms include: loose and/or frequent
heavy growth of Methicillin sensitive Staphylococcus aureus stools (often with blood or mucus), constipation, (especially
(MSSA). MSSA is S. aureus which is sensitive to flucloxacillin. soft stools with excessive straining), abdominal discomfort,
There is not an indication to escalate her to IV flucloxacillin in infantile colic or irritability, food aversion, perianal redness,

PAEDIATRICS AND CHILD HEALTH 31:4 171 Ó 2021 Elsevier Ltd. All rights reserved.
SELF-ASSESSMENT

pallor and tiredness or faltering growth. Exclusion of all the alternative diagnoses in light of the recent varicella
mammalian milk in the mother, and the infant’s diet is pro- infection.
posed to improve the severe eczema in the case. For many otherwise healthy children, Chickenpox is a self-
Coeliac disease and zinc deficiency were offered as poten- limiting disease. Disseminated varicella infection is rare in
tial causes for her refractory eczema. Whilst wheat allergy can immunocompetence, but complications include encephalitis
be associated with food-induced atopic eczema, this is sepa- and meningitis. Varicella lesions are a common precursor to
rate to Coeliac disease. Zinc deficiency affects 17% of the invasive group A streptococcal, or Streptococcus pyogenes, soft
world population, but is uncommon in the UK paediatric tissue infections, from which patients can deteriorate rapidly
population as it is mostly associated with general malnutrition and infections can be life-threatening. Group A streptococcus
due to starvation, severe illness or alcohol dependence. Zinc (GAS) is a Gram-positive beta-haemolytic bacterium which
deficiency can manifest cutaneously as Acrodermatitis Enter- causes non-invasive disease such as pharyngitis and cellulitis
opathica which is characterised by a dramatic, sharply and invasive disease where it is isolated from a normally
demarcated erythematous rash, typically in a horseshoe sterile body site. This includes bacteraemia, necrotising fas-
pattern over cheeks and chin, sparing the mouth. Glossitis, ciitis and toxic shock syndrome. Incidence of invasive GAS
mouth ulcers, symmetrical peri-anal excoriations and buttock (iGAS) infections is increasing. Risk factors for iGAS infection
rash are also common, amongst a number of other features. besides varicella, include living in a household with multiple
children, new use of non-steroidal anti-inflammatory drugs
Case 3
(NSAIDs), and concurrent eczema. GAS carriage is common
1) A and D
among school aged-children and invasive GAS, although rare,
2) A
is more common under 10 years of age compared to the gen-
3) C
eral population, and slightly less common than the general
Chickenpox is a common acute infection of children caused by
population in ages 10e19 years, although epidemiological
the varicella-zoster virus. It is transmitted via personal contact
data in this area is limited. The association between NSAIDs
or droplet spread with an incubation period of 1e3 weeks. It is
and iGAS infections is documented in the literature and as a
characterised by papules which develop into intensely itchy
result paracetamol is recommended instead of ibuprofen when
clear vesicles and pustules over 12e14 hours, and gradually
managing the symptoms of chickenpox in children. However,
dry out over approximately 5 days. Associated features include
an association only has been identified and further research is
nausea, myalgia and anorexia. Impetigo is a superficial bac-
needed to explore causality. Early recognition of iGAS is crit-
terial infection affecting the skin, which can be bullous or non-
ical, and it should always be considered in a patient who de-
bullous in character. It can affect all ages, but is most common
velops a new fever on day 4 of varicella infection, or in any
in children up to 4 years old. Non-bullous impetigo is usually
child with recent varicella infection and increasing pain or
caused by S. aureus and/or S. pyogenes (Group A strepto-
swelling on any body part with non-specific symptoms
coccus). It presents as a vesicular or pustular rash, which are
including lethargy and/or irritability.
rarely intact by examination and form a golden/brown crust as
Necrotising fasciitis is a life-threatening infection of the
they dry out. Impetigo is usually mild and self-limiting with
subcutaneous soft tissue and may also extend into the deep
hygiene measures. Children should stay off school until le-
fascia. It may be mono- or polymicrobial in origin. Necrotising
sions are dry or 48 hours after antibiotics commence, if clin-
fasciitis can be separated into 4 categories, according to aeti-
ically indicated.
ology. Type II account for 20e30% of cases and is generally
Dermatitis herpetiformis is a rare immune-mediated skin
due to iGAS or occasionally S. aureus, instead or alongside.
reaction related to gluten resulting in clusters of vesicular le-
Visible clinical signs, including bruising, bullae and necrosis
sions similar to a presentation of herpes simplex virus. It
are a late finding. The cornerstone of treatment of NF is sur-
mostly affects those aged 15e40 years and is not the diagnosis
gical debridement of the affected area and early surgical re-
here. Whilst a skin abscess may present on the thigh, it is not
view and management is essential, often with repeated
the most likely diagnosis in the context of pain around the side
surgical debridement required. Early collaborative manage-
of a previous chickenpox lesion. Gianotti-Crosti syndrome, or
ment between plastic surgeons, intensivists and microbiologist
papulovesicular acrodermatitis of childhood, mostly affect
or infectious diseases physicians improves survival. Varicella
children aged 6 months to 12 years and is a cutaneous reaction
vaccination has been estimated to prevent at least 10% of all
to viral infection, including Epstein Barr Virus, Cytomegalo-
invasive GAS in children and the majority of cases of
virus and Respiratory syncytial virus. It presents over 3e4
Necrotising Fasciitis due to varicella. A 27% reduction in iGAS
days with widespread red lesion 5e10 mm in diameter, which
was observed in the USA following the inclusion of varicella in
may become purple or develop into vesicles. Affected children
the routine vaccination schedule. There is no approved vac-
may have a mild pyrexia and persistent axillary and groin
cine for Group A streptococcus, but trials are ongoing.
lymphadenopathy, but usually remain well. Cutaneous mani-
Toxic Shock Syndrome (TSS) can occur in up to 50% of
festations of ibuprofen allergy are possible, but less likely that
cases of NF. It is characterised by fever, erythematous rash

PAEDIATRICS AND CHILD HEALTH 31:4 172 Ó 2021 Elsevier Ltd. All rights reserved.
SELF-ASSESSMENT

and multiorgan failure and requires aggressive fluid resusci- responding to aggressive treatment of sepsis and source
tation and inotropic support. In GAS TSS not all classic control.
symptoms may be present, and it should be in the early dif-
ferential for any child presenting with fever, rash and diar- FURTHER READING
rhoea/vomiting. Our patient required initial fluid resuscitation
Curtis N. Toxic shock syndrome: under-recognised and
but did not develop full blown toxic shock syndrome. Both
under-treated? Archives of Disease in Childhood 2014;99:1062
TSS and NF are toxin mediated diseases. Clindamycin inhibits
e1064. https://adc.bmj.com/content/99/12/1062.1.
protein synthesis so is a useful addition to ceftriaxone in toxin
Don’t Forget the Bubbles Skin Deep. Accessed online:
mediated disease by inhibiting bacterial superantigen toxin
https://dftbskindeep.com/.
production. Additional effects are enhancing phagocytosis;
RCPCH Allergy care pathway for anaphylaxis. Accessed
having effective killing in the presence of high bacterial loads
online: https://www.rcpch.ac.uk/resources/allergy-care-
and having a longer post-antimicrobial effect. IV immuno-
pathway-anaphylaxis.
globulin can be considered in toxic shock syndrome not

PAEDIATRICS AND CHILD HEALTH 31:4 173 Ó 2021 Elsevier Ltd. All rights reserved.
SELF-ASSESSMENT

Self-assessment
Case 1 BE -11
Lactate 4.1
A 19-year-old mother presents in established labour to a la-
Glucose 3.3.
bour ward in a large teaching hospital following a concealed
pregnancy. It is her first pregnancy. At delivery, the baby gives Question 2
a cry but is noted to have significantly increased work-of- What following steps would be most appropriate in his
breathing and remains deeply cyanosed. The midwifery team management?
provide PEEP via a face mask and place a crash-call for Choose THREE of the following:
medical assistance. At the time of the neonatal crash team’s a) 10e20 ml/kg bolus of normal saline 0.9%
arrival at 4 minutes of life the baby boy is receiving facemask b) Administer surfactant at 200 mg/kg via ETT
PEEP 5 cm H2O in FiO2 0.45. He has deep subcostal and c) IV 10% dextrose 2.5 ml/kg
intercostal recessions with grunting and intermittent apnoea. d) Commence High Frequency Oscillatory Ventilation (HFOV)
Pre-ductal saturations are 49%. At a glance he appears of a e) Secure umbilical venous and arterial access
good birth weight with features typical of a term infant. He is f) Give IM Atracurium urgently
noted to have a scaphoid abdomen. g) Keep nil-by-mouth with a wide-bore NGT on free drainage
with intermittent suctioning
Question 1 h) Urgent surgical review
What are the most important clinical interventions to make i) Commence active therapeutic hypothermia with
next? amplitude-integrated EEG monitoring
Choose ONE of the following: j) Start inhaled nitric oxide (iNO) at 20 ppm
a) Provide facemask intermittent positive pressure ventilation k) Start a prostaglandin infusion
at 30/5 cm H2O pressure The baby’s mother arrives on the unit and asks for an
b) Perform Less Invasive Surfactant Administration (LISA/ update on her baby’s condition.
MIST) with 200 mg/kg surfactant
Question 3
c) Turn off the overhead heater and commence passive
Which of the following statements are true for her baby?
cooling
Choose ONE of the following:
d) Endotracheal intubation and placement of a wide-bore
a) Her baby is likely to require surgery within the next 24
NGT/OGT with gastric suctioning
hours
e) Increase the PEEP to 7 cm H2O and increase FiO2 until
b) Her baby has a serious condition, which a significant mi-
adequate pre-ductal oxygen saturations are reached
nority of infants do not survive
Following initial stabilisation and transfer to the neonatal
c) Her baby’s condition is always associated with multiple
unit, a chest and abdominal X-ray is performed, as shown in
congenital abnormalities
Figure 1.
d) A full recovery can be expected after surgical correction
By this time, the baby is intubated and is receiving con-
e) Her baby requires transfer for Extracorporeal Membrane
ventional ventilation using pressures of 25/4 cm H2O in FiO2
Oxygenation (ECMO)
0.50. Pre-ductal saturations are 90% and post-ductal satura-
tions are 84%. Heart rate is 180/minute, Respiratory rate 60/ Answers
minute and mean BP 30 mm Hg. CRT is 4 seconds. Venous 1 D
gases are taken from a peripheral line: 2 A, E, G
pH 7.16 3 B
pCO2 7.6
Discussion
pO2 7.5
Congenital diaphragmatic hernia (CDH) is characterised by a
HCO3 16.9
defect in the diaphragm, allowing herniation of the abdominal
organs into the thoracic cavity, with this leading to pulmonary
hypoplasia and persistent pulmonary hypertension of the
Matthew Pickup MB BCh MRCPCH Paediatric Trainee, Paediatric newborn (PPHN). Despite advances in medical and surgical
Department, Noah’s Ark Children’s Hospital, University Hospital of care, including the availability of ECMO, mortality rates
Wales, Cardiff, UK. Conflicts of interest: none declared. remain high, with an approximate survival rate of 70%.
The size and nature of the diaphragmatic defects can range
Thomas Isaac MBChB BSc PGC MRCPCH Paediatric Trainee, Paedi- from small postero-lateral defects to complete absence of
atric Department, Noah’s Ark Children’s Hospital, University diaphragm. Bochdalek, or postero-lateral hernias, are the most
Hospital of Wales, Cardiff, UK. Conflicts of interest: none common (70e75%), Morgagni or anterior hernias account for
declared. 23e28% of cases, and central hernias 2e7%.

PAEDIATRICS AND CHILD HEALTH 31:5 223 Ó 2021 Elsevier Ltd. All rights reserved.
SELF-ASSESSMENT

as possible. Gentle conventional (i.e. PIP <25 cm H2O and


PEEP 3e5 cm H2O) ventilation is recommended, with HFOV
used as a “rescue” option. Generally, maintaining pre-ductal
saturations >85% is recommended. Most infants have PPHN
and often require inotropic support. Inhaled Nitric Oxide is
often used, although due to the underlying nature of CDH e
with hypoplastic lungs and altered pulmonary vasculature e it
is not as effective as it is in other causes of PPHN. ECMO is
considered the last lifesaving option for infants without other
significant life-limiting abnormalities who are not responding
to conventional medical management.
The timing of surgical repair is often a difficult decision,
but generally occurs once reasonable cardiovascular and res-
piratory stability have been achieved, usually at least 48e72
hours after delivery. The pulmonary hypoplasia and PPHN
will not be instantly rectified by surgical correction of the
defect, and indeed the PPHN may be significantly worsened by
the stress of the operation. Post-operative recovery is normally
a long, complicated and risky period for the infant.
Further reading
BAPM. National care principles for the management of
Congenital Diaphragmatic Hernia (2018)
Chandrasekharan PK, Rawat M, Madappa R, Rothstein DH,
Lakshminrusimha S. Congenital Diaphragmatic herniaea re-
Figure 1 Chest and abdominal X-ray taken on admission to the view. Matern health neonatal perinatology. 2017 Dec 1;3:6.
neonatal unit. Snoek KG, Reiss IK, Greenough A, et al. Standardized postnatal
management of infants with congenital diaphragmatic hernia
CDH is in most cases an antenatal diagnosis made on ul- in Europe: the CDH EURO consortium consensus-2015 up-
trasound screening. The presence of additional congenital date. Neonatology. 2016;110:66-74.
abnormalities, herniation of the liver and smaller observed-to-
expected lung-to-head ratios (O/E LHR) are associated with Case 2
poorer outcomes. The majority of CDHs are left-sided (85%). A 36 week gestation male infant is born by SVD with a
Right-sided hernias are far more difficult to diagnose on birthweight of 2.82 kg (57th centile). Antenatal scans had
antenatal ultrasound, especially if the liver is the only organ to demonstrated polyhydramnios and absent stomach but were
herniate. Infants not diagnosed antenatally may present with otherwise unremarkable. There was no significant maternal
respiratory distress and a scaphoid abdomen at birth. medical history. At birth he makes an initial cry and is placed
CDH most often occurs in isolation, although approxi- onto his mother while the cord was clamped and cut. How-
mately 10% of affected infants have chromosomal abnormal- ever, he quickly develops respiratory distress and is trans-
ities and approximately 30% have other associated significant ferred to the Resuscitaire. He is managed in PEEP and his pre-
congenital malformations e often cardiac, pulmonary, CNS or ductal saturations are 45% at 2 minutes of life rising to 56%
musculoskeletal. by 5 minutes of life. He is noted to have copious secretions
Foetal endoscopic tracheal occlusion (FETO) is a possible which are suctioned via a flexible suction catheter passed
future prenatal intervention that has been reported to accel- orally. Following this his saturations rise to 90% and his work
erate lung growth, however this is currently only performed in of breathing improves. By 15 minutes of life he is weaned to
a trial setting. air but has required further suctioning of his copious secre-
Management at birth involves immediate placement of a tions. He has normal heart sounds and easily palpable femo-
wide-bore NGT/OGT with suction to decompress the bowel. rals. He has appropriate tone and has started to root for a feed.
Immediate endotracheal intubation is generally required, un- His mother is keen to breastfeed.
less there is known to be very favourable anatomical factors.
Bag-mask ventilation is to be avoided. Immediate paralysis Question 1
with IM Atracurium is generally not recommended anymore. What procedure and investigation is most appropriate for this
Low-pressure ventilation, preferably using a PIP <25 cm H2O, infant?
is recommended to avoid damage to the hypoplastic lung and Choose ONE of the following.
reduce the risk of pneumothorax. a) IV cannulation and blood culture
Once in the neonatal unit the emphasis in management is b) NG tube insertion and litmus test for aspirate pH
to maintain respiratory and cardiovascular stability as much c) Capillary blood sample and blood sugar

PAEDIATRICS AND CHILD HEALTH 31:5 224 Ó 2021 Elsevier Ltd. All rights reserved.
SELF-ASSESSMENT

d) NG insertion and chest X-ray airway and GI tract occurs. OA is often detected antenatally
e) IV cannulation and chest X-ray with findings of polyhydramnios (due to lack of swallowing of
Following investigation, he is diagnosed with an oesopha- the amnion) and a small or absent stomach bubble depending
geal atresia and possible tracheo-oesophageal fistula. He is on the presence of a distal fistula. Following delivery babies
referred to the paediatric surgical team who plan to take him to with OA, with or without TOF, can present with copious se-
theatre for assessment and management tomorrow morning. cretions and associated respiratory distress including coughing
and spluttering, especially around feeds. Rarely an ‘H type’
Question 2
TOF can occur where the oesophagus and trachea are both
What following steps would be most appropriate in his man-
distally patent but connected. These can be missed in the
agement. Choose THREE of the following:
neonatal period and can present later in childhood with
a) Establish IV access
recurrent chest infections.
b) NG feeding
NG aspiration of gastric fluids and confirmation of acidic pH
c) Insert Repogle tube and place on suction
can be used to exclude OA in cases of polyhydramnios. How-
d) Perform partial septic screen
ever, at birth gastric aspirates may not have sufficiently low pH
e) Start IV antibiotics
and this can lead to a delay in either feeding or appropriate
f) Insert urinary catheter
diagnosis of OA. Therefore, where possible a chest X-ray with
g) Abdominal X-ray
NG in situ should be performed. A chest X-ray with a nasogastric
h) Begin High-Flow oxygen
tube in situ reflexing in a blind ended oesophagus, typically at
i) Make NBM and start IV fluids
T2-T4, is diagnostic of OA. An absent stomach bubble on X-ray
j) Intubate and ventilate
is indicative of isolated OA without TOF, but endoscopy, with or
k) Start Lansoprazole
without bronchoscopy, is the gold standard assessment.
Following assessment in theatre he is found to have an
Repair of OA with primary anastomosis can be complicated
isolated oesophageal atresia with a long gap. Primary repair is
by a large distance between the proximal and distal oeso-
not possible at this time, so a decision is made to place a
phageal segments, a so called ‘long gap’. There is not a
gastrostomy and re-assess after a period of feeding and
consensus on standard definition of long gap but distances less
growing. The surgical team request that a VACTERL screen is
than 3 cm are normally amenable to primary repair at initial
completed. The infant had an echo prior to theatre and has
assessment. Practice varies but in cases of OA with long gap
had a newborn examination completed.
delayed repair following a period of growth, with or without
Question 3 mechanical traction of the oesophagus, is typical. Gastrostomy
What other investigations are required? for feeding is required in this instance and the infant will
Choose TWO of the following: require a Repogle tube throughout this time due to risk of
a) Blood gas aspiration. Measurement of gap distance can be performed by
b) Ophthalmology review fluoroscopic or endoscopic assessment.
c) Ultrasound kidneys, ureters and bladder Over 50% of OA with or without TOF are associated with
d) Abdominal X-ray multiple congenital anomalies. Most commonly these are part
e) Plastic surgery review of the VACTERL association (vertebral, anorectal, cardiac,
f) X-ray spine renal, tracheo-oesophageal and limb anomalies) but CHARGE
g) Ultrasound spine syndrome (coloboma, heart, atresia choanae, retardation of
h) Urine for organic acids growth and development, genital and ear anomalies) and tri-
i) CGH array somy 21 and 18 are also common. To assess for VACTERL a
j) Cranial ultrasound complete baby check, echocardiogram, abdominal ultrasound
k) Guthrie card including kidneys, ureters and bladder and ultrasound of the
spine are necessary.
Answers
1 D
Further reading
2 A, C, I
Hunt RW, Perkins EJ, King S. Peri-operative management of
3 C, G
neonates with oesophageal atresia and tracheo-oesophageal
Discussion fistula. Paediatr Respir Rev 2016;19:3-9. doi: 10.1016/
Oesophageal atresia (OA) occurs following incomplete em- j.prrv.2016.01.002 [published Online First: 2016/02/29]
bryonic compartmentalisation of the foregut occurring in 2.4 Shieh HF, Jennings RW. Long-gap esophageal atresia. Semin
per 100,000 births. Previously a fatal anomaly, advancements Pediatr Surg 2017;26:72-77. doi: 10.1053/j.semped-
in surgical techniques and follow up care have led to a positive surg.2017.02.009 [published Online First: 2017/05/30]
prognosis with over 90% survival. OA is commonly associated van Lennep M, Singendonk MMJ, Dall’Oglio L, et al. Oesopha-
with Tracheo-oesophageal fistula (TOF), although not in the geal atresia. Nat Rev Dis Primers 2019;5:26. doi: 10.1038/
case above, and is classified by where a fistula between the s41572-019-0077-0 [published Online First: 2019/04/20]

PAEDIATRICS AND CHILD HEALTH 31:5 225 Ó 2021 Elsevier Ltd. All rights reserved.
SELF-ASSESSMENT

Case 3 B CT abdomen
C Urine dipstick
A 34 week gestational age female infant was born by emergency
D Urea and electrolytes
Caesarean section after spontaneous onset of preterm labour
E DMSA scan
with an antenatal history of being large for gestational age and
F Chest X-ray
having an exomphalos identified on ultrasound scans. At birth
G Ultrasound kidneys, ureters and bladder
she was born in good condition and stabilised with non-invasive
H Urine microscopy for casts
ventilatory support in the delivery room prior to transfer to the
I Coagulation screen
neonatal unit. On examination she was breathing comfortably
in CPAP 6 cm in 25% FiO2, cardiovascularly stable with normal Answers
heart sounds and femoral pulses. She noted to have a large 1 B, D, I
abdominal wall defect with bowel and liver protruding through 2 C
this, contained within a thin membrane. She was also noted to 3 G
have bilateral anterior ear creases and a large tongue. Her Discussion
weight was 3.72 kg (>99.6th centile) and her head circumfer- Exomphalos, also known as omphalocele, is one of the most
ence was 36 cm (>99.6th centile). A capillary blood gas was common congenital abdominal wall defects, occurring in 1e2
taken which showed the following: per 10,000 births. Exomphalos is a midline defect where
pH 7.23 abdominal cavity contents herniate through the abdominal
pCO2 6.9 kPa wall but are encased in a three-layer covering of outer amnion,
p O2 7.1 kPa Wharton’s jelly and peritoneum. Due to this covering surgical
Na 137 mmol/L closure does not always need to be immediate, as the risk of
K 5.8 mmol/L fluid loss and infection is far less than in gastroschisis. It dif-
Calcium (ionised) 1.15 mmol/L fers from gastroschisis (where abdominal contents lie outside
Chloride 113 mmol/L the abdominal cavity without any covering) and umbilical
Lactate 4.3 mmol/L hernia (where abdominal contents herniate through the peri-
HCO3 23.3 mmol/L toneum at the umbilical ring but are covered with skin).
BE -3.6 mmol/L Unlike gastroschisis, exomphalos is commonly associated
Glucose 0.7 mmol/L with other abnormalities. In this case exomphalos is associ-
Question 1 ated with features typical of Beckwith-Wiedemann syndrome
What are the next most urgent clinical steps? (BWS). BWS is the most common overgrowth syndrome and
Choose THREE of the following: is an imprinting disorder associated with changes at chromo-
A Urgent surgical review some 11p15 region. Exomphalos is a common feature along
B IV access with macroglossia and macrosomia. Other features include
C Partial septic screen and IV benzylpenicillin and gentamicin early onset of labour, enlarged placenta detected antenatally,
D Bolus of 2.5 ml/kg of 10% dextrose IV neonatal hypoglycaemia, diastasis recti, port wine stain,
E Intubation and ventilation with volume guarantee cardiac abnormalities, anterior ear creases, hemihyperplasia,
F Intubation and ventilation with pressure guarantee prominent occiput, hearing loss and renal abnormalities.
G Start a prostaglandin infusion BWS is associated with increased risk of neoplasia, most
H Bolus of 100 mcg/kg morphine IV commonly Wilm’s tumour, hepatoblastoma and less
I Start IV infusion of 10% dextrose at 60 ml/kg/d commonly neuroblastoma or rhabdomyosarcomas. These can
occur in the neonatal period or childhood, although 95% of
Question 2
BWS associated tumours will have occurred by age 8. Sur-
What is the most likely underlying diagnosis?
veillance with regular USS KUB is indicated.
Select ONE of the following:
A Sotos syndrome Further reading
B Infant of a diabetic mother Verla MA, Style CC, Olutoye OO. Prenatal diagnosis and man-
C Beckwith-Wiedemann Syndrome agement of omphalocele. Semin Pediatr Surg 2019;28:84-88.
D Wiskott Aldrich Syndrome doi: 10.1053/j.sempedsurg.2019.04.007 [published Online
E VACTERL association First: 2019/05/11]
She was stabilised and following corrective surgery had a Wang KH, Kupa J, Duffy KA, et al. Diagnosis and Management of
stable neonatal course and was discharged home. Beckwith-Wiedemann Syndrome. Front Pediatr 2019;7:562.
doi: 10.3389/fped.2019.00562 [published Online First: 2020/
Question 3
02/11]
Due to her underlying condition what regular screening
Zammit M, Caruana E, Cassar D, et al. Beckwith-Wiedemann
investigation would be most appropriate to continue until at
Syndrome Review: A Guide for the Neonatal Nurse.
least age 7?
Neonatal Netw 2017;36:129-133. doi: 10.1891/0730-
Select ONE of the following:
0832.36.3.129 [published Online First: 2017/05/13]
A Full blood count

PAEDIATRICS AND CHILD HEALTH 31:5 226 Ó 2021 Elsevier Ltd. All rights reserved.
SELF-ASSESSMENT

Self-assessment
Case 1 a. Congenital adrenal hypoplasia due to gene mutation (e.g.
DAX-1, SF1 mutations)
A male infant, who was born at 35 þ 1 weeks of gestation, was
b. Bilateral adrenal haemorrhage of the newborn
admitted to the neonatal unit at few hours of age due to low
c. Adrenal unresponsiveness to ACTH due to gene mutations
blood sugar of 1.4 mmol/l for feeding support and blood sugar
d. Congenital adrenal hyperplasia
monitoring.
e. Triple A syndrome or Allgrove syndrome (alacrimia,
He was born by elective C-section due to breech presen-
achalasia, adrenal insufficiency)
tation. There was history of maternal gestational diabetes
f. Autoimmune adrenalitis (Addison’s disease)
during pregnancy but there were no other issues during the
pregnancy. There were no risk factors for sepsis. The antenatal
scans and serology were normal. His birth weight was 3.53 Kg. Question 3
His initial hypoglycaemic episode resolved with no further Which investigation would be helpful to establish the diag-
low blood sugar levels. However, he remained in the neonatal nosis? Please select FIVE.
unit for feeding support due to persistent vomiting. His for- a. Urea and electrolytes
mula was changed to hydrolysed formula which led to some b. Blood cultures
improvement with vomiting. c. Liver function test
On day 8 of life, he was noted to have lost 14% of his birth d. CRP
weight. His clinical examination was normal. e. Full blood count
A blood gas was performed: f. 17alpha- OH- progesterone
pH 7.53 g. Synacthen test
pCO2 3.65 kPa h. Urine steroid profile
cBase(Ecf) 2 mmol/L i. Abdominal xray
HCO3 22 mmol/L j. Genetics
Na 130 mmol/L
K 7.1 mmol/L Question 4
Cl 100 mmol/L What medications will the patient most likely need? Please
Lac 1.8 mmol/L select THREE.
Glu 3.3 mmol/L a. Sodium supplements
b. Omeprazole
Question 1 c. Gaviscon
What is the most likely diagnosis? Please select ONE. d. Glucocorticoid
a. Sepsis e. Mineralocorticoid
b. Cow’s milk protein allergy f. First line antibiotics
c. Adrenal insufficiency g. Second line antibiotics
d. Pyloric stenosis
e. Necrotising enterocolitis Question 5
Despite the initial treatment of sodium supplements, gluco-
Question 2 corticoid and mineralocorticoid, sodium remained low and
What is the commonest cause of primary adrenal insufficiency potassium high. What would be the next step in management?
in a neonate? Please select ONE. Please select TWO.
a. To increase the sodium supplements
b. To decrease the sodium supplements
Niki Skaltsa MD MRCPCH Paediatric trainee, ST7, West Suffolk c. To increase the glucocorticoid
Hospital NHS Foundation Trust, UK. Competing interests: none d. To decrease the glucocorticoid
declared. e. To increase the mineralocorticoid
f. To decrease the mineralocorticoid
Binu Anand MBBS MD MRCP(UK) FRCPCH Paediatric Consultant,
West Suffolk Hospital NHS Foundation Trust, UK. Competing Answers
interests: none declared. 1. c
2. d
Raman Lakshman MBBS MD DCH FRCPCH Paediatric Consultant, 3. a, f, g, h and j
West Suffolk Hospital NHS Foundation Trust, UK. Competing 4. a, d and e
interests: none declared. 5. a and e

PAEDIATRICS AND CHILD HEALTH 31:6 266 Ó 2021 Elsevier Ltd. All rights reserved.
SELF-ASSESSMENT

Congenital adrenal hyperplasia (CAH) developed respiratory distress for which she required CPAP
CAH is a group of inherited autosomal recessive disorders, each and remained in the neonatal unit for 3 weeks. During in-
of which is characterized by a deficiency of an enzyme involved fancy, there were concerns about poor growth, delayed
in the synthesis of cortisol, aldosterone or both. CAH is the most development and some subtle dysmorphic features.
common cause of primary adrenal insufficiency in children. She was subsequently tested and diagnosed with Cornelia
Deficiency of 21-hydroxylase is the most common of these De Lange Syndrome. She also had gastro-oesophageal reflux
disorders, accounting for more than 90% of CAH cases. for which she was on Omeprazole.
Salt-wasting 21-hydroxylase deficiency is the result of When she was reviewed at 2 years of age, her height was
complete deficiency of P450c21 leading to insufficient pro- 76.5cm (well below 0.4th centile, -3.3 SDS) and her weight was
duction of both glucocorticoids and mineralocorticoids. 8.8 Kgs (just below 0.4th centile, - 3.1 SDS). She did not appear
Females are normally diagnosed at birth due to virilisation to have any skeletal disproportion and clinically did not fit the
of the external genitalia. Males are either diagnosed during a profile of growth hormone deficiency. A growth hormone
salt-losing crisis (failure to thrive, recurrent vomiting, dehy- stimulation test was organized but eventually not performed
dration, hypotension, hyponatremia, hyperkalemia and due to technical difficulties.
shock) on day 5e15 of life or die, not having been diagnosed. She was again reviewed at 4 years of age and her height
Investigations that help to establish the diagnosis are had remained below the 0.4th centile and well below the
electrolytes, blood sugar, plasma 17a- OH-progesterone, family target height range with her height standard deviation
plasma renin activity, urine steroid profile and blood for ge- score being -2.9.
netic analysis. Synacthen test may be useful when initial test Therefore, decision was made for a trial of growth hormone
results are equivocal. therapy to be started.
Once samples have been obtained for diagnosis, treatment
should be started and include hydrocortisone, 9a-flu-
Question 1
drocortisone and salt supplements. Hydrocortisone dose for
Which condition is NOT associated with Cornelia De Lange
congenital adrenal hyperplasia is 18e21 mg/m2/day in 3
Syndrome? Please select ONE.
divided doses. Fludrocortisone should be started at 25
a. Cleft palate
microgram BD and increased to 50 microgram BD. In resistant
b. Nasal polyps
cases where sodium remains low, the dose can be increased to
c. Gastroesophageal reflux disease
100 microgram BD. Larger doses require more close moni-
d. Pyloric stenosis
toring for hypokalaemia. Salt supplements, in addition, are
e. Intestinal malrotation/volvulus
essential. They can be started at 5 mmol/kg/day but can be
f. Bladder exstrophy
increased if required.
g. Lacrimal duct stenosis
The rest of the diagnoses would be less likely;
h. Hip dislocations
Sepsis or necrotising enterocolitis are less likely on day 8 of
life with normal clinical examination, no risk factors for
sepsis, normal antenatal scans and normal lactate. Question 2
Cow’s milk protein allergy would not manifest that early in Which was the indication for growth hormone therapy in the
life. The baby did not have any other symptoms suggestive of above case? Please select ONE.
cow’s milk protein allergy e. g. refusal to feed, diarrhoea or a. Growth hormone deficiency
constipation, blood or mucous in stool and eczema. The gas b. Prematurity
changes cannot be explained by the diagnosis of cow’s milk c. Gastro-oesophageal reflux
protein allergy where the only changes, if any, would be due d. Cornelia de Lange Syndrome
to vomiting. e. Small for gestational age and failure to catch up
Further reading f. Developmental delay
Park J, Didi M, Blair J. The diagnosis and treatment of adrenal
insufficiency during childhood and adolescence. Archives of
Question 3
disease in childhood. 2016 Sep 1; 101 (9):860e865.
What are some of the conditions where Somatropin (recom-
Ogilvy-Stuart A and Midgley P, Practical Neonatal Endocrinology,
binant human growth hormone) is licenced as treatment op-
2006
tion for children with growth failure? Please select FOUR.
Brook C and Dattani M, Handbook of Clinical Pediatric Endocri-
a. Noonan syndrome
nology, 2008
b. PradereWilli syndrome
c. Chronic renal insufficiency
Case 2
d. Turner syndrome
A 4-year-old girl is being followed up in the paediatric e. Russel- Silver Syndrome
endocrinology clinic. She was born at 34þ 1 weeks of f. Short stature homeobox-containing gene (SHOX) deficiency
gestation with a birth weight of 1.49 Kg (2nd centile), g. Achondroplasia

PAEDIATRICS AND CHILD HEALTH 31:6 267 Ó 2021 Elsevier Ltd. All rights reserved.
SELF-ASSESSMENT

Question 4 haploinsufficiency were 3.8 cm shorter (2.1 standard de-


When should treatment with somatropin be discontinued? viations shorter) than their unaffected relatives.
Please select TWO.
Further reading
a. Growth velocity increases less than 30% from baseline in
Kline AD, Moss JF, Selicorni A, Bisgaard AM, Deardorff MA,
the first year of treatment
Gillett PM, Ishman SL, Kerr LM, Levin AV, Mulder PA, Ramos
b. Growth velocity increases less than 50% from baseline in
FJ. Diagnosis and management of Cornelia de Lange syn-
the first year of treatment
drome: first international consensus statement. Nature Re-
c. Growth velocity increases less than 60% from baseline in
views Genetics. 2018 Oct; 19 (10):649e666.
the first year of treatment
NICE guidelines, Human growth hormone (somatropin) for the
d. Final height is approached, and growth velocity is less than
treatment of growth failure in children, 26 May 2010,
2 cm total growth in 1 year
www.nice.org.uk/guidance/ta188
e. Final height is approached, and growth velocity is less than
de Graaf M, Kant SG, Wit JM, Redeker EJ, Santen GW, Verkerk
4 cm total growth in 1 year
AJ, Uitterlinden AG, Losekoot M, Oostdijk W. Successful
f. Final height is approached, and growth velocity is less than
growth hormone therapy in Cornelia de Lange syndrome.
6 cm total growth in 1 year
Journal of clinical research in pediatric endocrinology. 2017
Answers Dec; 9 (4):366. Pubmed Exact
1. f
2. e Case 3
3. b, c, d and f A 2-year-old girl, who had been diagnosed with congenital
4. b and d hypothyroidism, is being transferred to your care following
Cornelia de Lange syndrome and growth hormone family’s move to the area.
therapy She is the second child to her parents and was born at term
Cornelia de Lange syndrome (CdLS) is a genetic disorder following an uneventful pregnancy and delivery. Her thyroid
which is outlined by a range of multiple congenital anomalies function test on newborn screening test was abnormal.
such as distinctive facial features, prenatal and postnatal Her father, as well as her older brother, had also been
growth deficiency, feeding difficulties, psychomotor delay and diagnosed with congenital hypothyroidism on their newborn
behavioural problems. screening test.
Although it is characterized by intra-uterine growth retar- Question 1
dation and short stature, CdLS is not one of the licenced in- What is being tested on newborn screening test to screen for
dications of growth hormone therapy. congenital hypothyroidism? Please select ONE.
The indications for somatropin (recombinant human a. Free T4
growth hormone) a treatment option for children with growth b. Free T3
failure are as follows (as per NICE guidelines). c. Thyroglobulin
1. Growth hormone deficiency d. TSH
2. Turner syndrome e. TSH receptor antibodies
3. PradereWilli syndrome f. Gene analysis
4. Chronic renal insufficiency
Question 2
5. Born small for gestational age with subsequent growth
Following an abnormal newborn screening test, which in-
failure at 4 years of age or later
vestigations need to be performed to establish diagnosis and
6. Short stature homeobox-containing gene (SHOX)
aetiology? Please select SIX.
deficiency.
a. Free T4
Growth hormone therapy should be stopped when;
b. Free T3
1. growth velocity increases less than 50% from baseline in
c. Thyroglobulin level
the first year of treatment.
d. TSH
2. growth velocity is less than 2 cm total growth in 1 year
e. TSH receptor antibodies
there are insurmountable problems with adherence
f. Thyroid scintigraphy I123, Tc99
3. final height is achieved
g. MRI head
If untreated, men with PradereWilli syndrome have a
h. CT head
final adult height of about 154 cm while women have a final
i. USS of thyroid gland
adult height of 145e159 cm. With regards to girls with
Turner syndrome, they have a final adult height of 136e147 Question 3
cm and they are on average 20 cm shorter than other adult In which aetiology of congenital hypothyroidism it is likely to
women. In terms of SHOX, a small study (26 people with be able to stop treatment after the second year of age? Please
SHOX haploinsufficiency compared with 45 of their unaf- select ONE.
fected relatives) showed that children with SHOX a. Thyroid agenesis

PAEDIATRICS AND CHILD HEALTH 31:6 268 Ó 2021 Elsevier Ltd. All rights reserved.
SELF-ASSESSMENT

b. Biosynthetic defect Babies where the mean TSH of the three tests is 20.0 mU/
c. Thyroglobulin synthetic defect L should be considered to have a positive screening result for
d. Transient hypothyroidism CHT and should be referred, regardless of their gestational age
e. Ectopic thyroid gland at birth, to the paediatric endocrine team.
Babies where the mean TSH of the three tests is between
Question 4
8.0 mU/L and 20.0 mU/L should be considered to have a
What is the aetiology of congenital hypothyroidism if the
borderline result for CHT. A repeat dried blood spot sample
radioisotope scan showed absent gland, but the USS thyroid
should be performed 7e10 days after the initial sample and
showed normal gland with positive TSH receptor antibodies?
assayed for TSH in duplicate, ideally from different spots.
Please select ONE.
Congenital hypothyroidism results from the inadequate
a. Thyroid agenesis
thyroid hormone production which can occur because of a
b. Biosynthetic defect
defect in the gland, an inborn error of thyroid metabolism or
c. Thyroglobulin synthetic defect
iodine deficiency.
d. Transient hypothyroidism
Thyroid dysgenesis was believed to account for w80% of
e) Ectopic thyroid gland
the cases of congenital hypothyroidism. With recent studies
Question 5 showing a doubling in incidence of congenital hypothyroidism
What is the most appropriate next step for term babies whose as well as an increase in congenital hypothyroidism with
TSH concentration on newborn screening is >6 mU/L? Please thyroid gland-in-situ (GIS), the epidemiology has probably
select ONE. slightly changed. Although the majority of dyshormono-
a) Congenital hypothyroidism is not suspected, and no further genesis has been explained by mutations in genes encoding
action is required components of the thyroid hormone biosynthesis pathway,
b) A repeat request should be issued mutation in only four transcription factors that mediate the
c) The sample should be retested in duplicate from the same normal thyroid development have been identified. These are
card but on a different spot(s) to give a more definitive the following: NKX2-1, PAX8, FOXE1 and HHEX and their
result mutations result in thyroid dysgenesis. Mutations in these
d) Babies should be considered to have a positive screening genes are rare, accounting for <5% of congenital hypothy-
result for congenital hypothyroidism and should be re- roidism. Therefore, known genetic causes account for <20%
ported as ‘congenital hypothyroidism suspected’ of the cases of congenital hypothyroidism collectively. How-
Answers ever, a confirmatory genetic diagnosis can facilitate counsel-
1. d ling as well as management by allowing treatment withdrawal
2. a, c, d, e, f and i in cases of transient hypothyroidism or by identifying certain
3. d genetic aetiologies which present with normal TSH levels in
4. d neonatal life but evolve to significant hypothyroidism later in
5. c childhood. Genetic confirmation of the later cases would allow
close biochemical monitoring and therefore early diagnosis in
Congenital hypothyroidism (CHT) affected siblings.
Congenital hypothyroidism is most commonly detected by If the radioisotope imaging shows no active gland, but the
elevated TSH on newborn screening. USS shows normal thyroid gland and the TSH receptor anti-
TSH analysis should be carried out on a single spot from bodies are positive, then transient hypothyroidism due to
the initial dried blood sample. maternal antibodies is the most likely diagnosis.
Babies whose TSH concentration is <6.0 mU/L in the initial
sample should be considered to have a negative screening Further reading
result for congenital hypothyroidism. They should be reported Ogilvy-Stuart A and Midgley P, Practical Neonatal Endocrinology,
as ‘CHT not suspected’, unless they were born at less than 32 2006
weeks gestation. Jacob H, Peters C. Screening, diagnosis and management of
Samples with TSH >6.0mU/L are retested in duplicate from congenital hypothyroidism: European Society for Paediatric
the same card, but on different spots. Endocrinology Consensus Guideline. Archives of Disease in
Then a plan is made based on the triplicate mean result. Childhood-Education and Practice. 2015 Oct 1; 100 (5):260
Babies where the mean TSH of the three tests is <8.0 mU/L e263. Refrository Partial stl iss
should be considered to have a negative screening result for Schoenmakers N, The genetic basis of congenital hypothyroid-
CHT. ism, The Endocrinologist, Issue 125, Autumn 2017

PAEDIATRICS AND CHILD HEALTH 31:6 269 Ó 2021 Elsevier Ltd. All rights reserved.

You might also like